NUR 205- MedSurg 2- Test 1

Lakukan tugas rumah & ujian kamu dengan baik sekarang menggunakan Quizwiz!

A major role for nursing in the management of glaucoma is health education. Which of the following is the most important teaching point that the nurse should advise the patient of? Participate in the decision-making process. Keep a record of eye pressure measurements. Keep all follow-up appointments. Adhere to the medication regimen.

Adhere to the medication regimen. All of the teaching points are important but the most important is emphasizing the strict adherence to the medication regimen because glaucoma cannot be cured but its progression can be slowed.

A nurse is caring for a patient diagnosed with a hemorrhagic stroke. When creating this patients plan of care, what goal should be prioritized? A) Prevent complications of immobility. B) Maintain and improve cerebral tissue perfusion. C) Relieve anxiety and pain. D) Relieve sensory deprivation.

Ans: B Feedback: Each of the listed goals is appropriate in the care of a patient recovering from a stroke. However, promoting cerebral perfusion is a priority physiologic need, on which the patients survival depends.

The critical care nurse is admitting a patient in myasthenic crisis to the ICU. The nurse should prioritize what nursing action in the immediate care of this patient? A) Suctioning secretions B) Facilitating ABG analysis C) Providing ventilatory assistance D) Administering tube feedings

Ans: C Feedback: Providing ventilatory assistance takes precedence in the immediate management of the patient with myasthenic crisis. It may be necessary to suction secretions and/or provide tube feedings, but they are not the priority for this patient. ABG analysis will be done, but this is not the priority.

The critical care nurse is caring for 25-year-old man admitted to the ICU with a brain abscess. What is a priority nursing responsibility in the care of this patient? A) Maintaining the patients functional independence B) Providing health education C) Monitoring neurologic status closely D) Promoting mobility

Ans: C Feedback: Vigilant neurologic monitoring is a key aspect of caring for a patient who has a brain abscess. This supersedes education, ADLs, and mobility, even though these are all valid and important aspects of nursing care.

A patient is admitted through the ED with suspected St. Louis encephalitis. The unique clinical feature of St. Louis encephalitis will make what nursing action a priority? A) Serial assessments of hemoglobin levels B) Blood glucose monitoring C) Close monitoring of fluid balance D) Assessment of pain along dermatomes

Ans: C Feedback: A unique clinical feature of St. Louis encephalitis is SIADH with hyponatremia. As such, it is important to monitor the patients intake and output closely.

A patient who has been experiencing numerous episodes of unexplained headaches and vomiting has subsequently been referred for testing to rule out a brain tumor. What characteristic of the patient's vomiting is most consistent with a brain tumor? A)The patient's vomiting is accompanied by epistaxis. B)The patient's vomiting does not relieve his nausea. C)The patient's vomiting is unrelated to food intake. D)The patient's emesis is blood-tinged.

Ans: C) The patient's vomiting is unrelated to food intake. Feedback: Vomiting is often unrelated to food intake if caused by a brain tumor. The presence or absence of blood is not related to the possible etiology and vomiting may or may not relieve the patient's nausea.

The nurse is creating a plan of care for a patient who has a recent diagnosis of MS. Which of the following should the nurse include in the patients care plan? A) Encourage patient to void every hour. B) Order a low-residue diet. C) Provide total assistance with all ADLs. D) Instruct the patient on daily muscle stretching.

Ans: D Feedback: A patient diagnosed with MS should be encouraged to increase the fiber in his or her diet and void 30 minutes after drinking to help train the bladder. The patient should participate in daily muscle stretching to help alleviate and relax muscle spasms.

Which client statement would lead the nurse to suspect that the client is experiencing bacterial conjunctivitis? "My eyelids were stuck together this morning." "My eyes hurt when I'm in the bright sunlight." "My eyes feel like they are on fire." "It feels like there is something stuck in my eye."

"My eyelids were stuck together this morning." Burning, a sensation of a foreign body, and pain in bright light (photophobia) are signs and symptoms associated with any type of conjunctivitis. The drainage related to bacterial conjunctivitis is usually present in the morning, and the eyes may be difficult to open because of adhesions caused by the exudate.

A 53-year-old man presents to the emergency department with a chief complaint of inability to form words, and numbness and weakness of the right arm and leg. Where would you locate the site of injury? 1- Left frontoparietal region 2- Right frontoparietal region 3- Left basal ganglia 4- Left temporal region

1

A nurse assesses the patient's level of consciousness using the Glasgow Coma Scale. What score indicates severe impairment of neurologic function? 1- 3 2- 6 3- 9 4- 15

1

A nurse is conducting a health history with a client. During the history, the client tells the nurse, "Things that are far away often look blurred." The nurse inteprets this finding as: 1- myopia 2- hyperopia 3- presbyopia 4- cataract

1

A nurse notices that a client's left upper eyelid is drooping. The nurse has observed: 1- ptosis 2- ptolemy 3- proptosis 4- nystagmus

1

A patient is participating in aural rehabilitation. The nurse understands that this type of training emphasizes which of the following? 1- Listening skills 2- Social skills 3- Occupational skills 4- Functional skills

1

Acoustic neuromas are benign tumors of which of the following cranial nerves? 1- VIII 2- VII 3- VI 4- V

1

The nurse recognizes that rheumatoid arthritis is characterized by: 1- Ulnar deviation 2- Ballottement sign 3- Clonus 4- Fasciculations

1

A home care nurse assesses for disease complications in a client with bone cancer. Which laboratory value may indicate the presence of a disease complication? 1- Potassium level of 6.3 mEq/L 2- Calcium level of 11.6 mg/dl 3- Sodium level of 110 mEq/L 4- Magnesium level of 0.9 mg/dl

2

A patient who has sustained a basal skull fracture is admitted to the neurological unit. The nurse should know that the patient should be observed for: 1- An area of bruising over the mastoid bone 2- Bleeding from the ears 3- An increase in pulse 4- Difficulty sleeping

2

An aging client is brought to the eye clinic by the son. The son states he has seen his parent holding reading materials at an increasing distance to focus properly. What age-related changes does this indicate? 1- Cataract 2- Presbyopia 3- Myopia 4- Macular degeneration

2

A child goes to the school nurse and complains of not being able to hear her teacher. What test could the school nurse perform to make a general estimate of hearing loss? 1- Chiming clock test 2- Rinne test 3- Whisper test 4- Weber test

3

A client presents to the emergency department stating numbness and tingling occurring down the left leg into the left foot. When documenting the experience, which medical terminology would the nurse be most correct to report? 1- Sciatic nerve pain 2- Herniation 3- Paresthesia 4- Paralysis

3

A nurse practitioner checks pupillary response with a penlight. The nurse is aware that the normal pupil size in room lighting is: 1- 1 to 2 mm. 2- 2 to 3 mm. 3- 3 to 5 mm. 4- 4 to 7 mm.

3

Which cells are involved in bone resorption? 1- Chondrocytes 2- Osteoblasts 3- Osteoclasts 4- Osteocytes

3

A client with a spinal cord injury has full head and neck control when the injury is at which level? 1- C1 2- C2 to C3 3- C4 4- C5

4

A nurse is performing a neurologic assessment on a client. The nurse observes the client's tongue for symmetry, tremors, and strength, and assesses the client's speech. Which cranial nerve is the nurse assessing? 1- IV 2- IX 3- VI 4- XII

4

The trochlear nerve controls which function? 1- Movement of the tongue 2- Hearing and equilibrium 3- Visual acuity 4- Eye muscle movement

4

Which symptoms may a client with Ménière disease report before an attack? 1- Nystagmus 2- Low blood pressure 3- Photosensitivity 4- A full feeling in the ear

4

A 72-year-old woman is being treated on a medical unit 1 week after experiencing an ischemic stroke. The effects of the woman's stroke include slurred, partially intelligible speech. However, the patient's comprehension of spoken language remains intact. The nurse should expect to read documentation in the patient's chart of: A) Dysarthria B) Aphasia C) Dysphasia D) Apraxia

A) Dysarthria

A client is DX with an acoustic neuroma. Which manifestations would the nurse except to find? Select all that apply. A) Tinnitus B) Vertigo C) Staggering Gait D) Seizures E) Headache

A) Tinnitus B) Vertigo C) Staggering Gait

Which of the following surgical procedures involves taking a piece of silicone plastic or sponge and sewing it onto the sclera at the site of a retinal tear? Pneumatic retinopexy Pars plana vitrectomy Scleral buckle Phacoemulsification

Scleral buckle The scleral buckle is a procedure in which a piece of silicone plastic or sponge is sewn onto the sclera at the site of the retinal tear. The buckle holds the retina against the sclera until scarring seals the tear. The other surgeries do not use this type of procedure.

Weber test

Test done by placing the stem of a vibrating tuning fork on the midline of the head and having the patient indicate in which ear the tone can be heard.

A patient is suspected of having myositis. The nurse prepares the patient for what procedure that will confirm the diagnosis? Bone scan Computed tomography (CT) Magnetic resonance imaging (MRI) Muscle biopsy

Correct response: Muscle biopsy Explanation: As with other diffuse connective tissue disorders, no single test confirms polymyositis. An electromyogram is performed to rule out degenerative muscle disease. A muscle biopsy may reveal inflammatory infiltrate in the tissue. Serum studies indicate increased muscle enzyme activity.

Nursing assessment for tinnitus, gastric intolerance, and bleeding is important for patient who take which class of medications for a rheumatic disease? Salicylates COX-2 inhibitors Immunosuppressive Antimalarials

Correct response: Salicylates Explanation: Salicylates have anti-inflammatory, analgesic, and antipyretic actions. They should be given with meals to prevent gastric irritation.

When caring for a patient who has had a hemorrhagic stroke, close monitoring of vital signs and neurological status is imperative. What is the earliest sign of deterioration in a patient with a hemorrhagic stroke? - Shortness of breath - Alteration in level of consciousness (LOC) - Headache - Tonic-clonic seizures

Correct response: Alteration in level of consciousness (LOC) Explanation: Alteration in LOC is the earliest sign of deterioration in a patient with a hemorrhagic stroke; these include mild drowsiness, slight slurring of speech, and sluggish papillary reaction.

The nursing educator on a neurological unit is conducting a brief in-service on glial cell tumors. The educator tells the nurses that the most common type of glioma is an astrocytoma and that astrocytomas are graded from I to IV, indicating the degree of malignancy. What is the tumor's grading based on? - Cellular density, cell mitosis, and appearance - Size of cells, number of cells, and appearance - Cell mitosis, size of cells, and appearance - Cellular density, number of cells, and appearance

Correct response: Cellular density, cell mitosis, and appearance Explanation: The grading of a glioma is based on cellular density, cell mitosis, and appearance.

A patient admitted with a stroke is coming to the unit from the emergency department. The nurse assigned to care for the new patient knows that what assessment finding is indicative of a stroke? - Electrolyte imbalance - Difficulty speaking - Increase in heart rate - Facial edema

Correct response: Difficulty speaking Explanation: Difficulty speaking is a classic abnormal finding on a physical assessment that may be associated with a stroke. Tachycardia, edema, and electrolyte imbalances are not common initial presentations of stroke.

A nurse is reading a journal article about spinal cord tumors and metastasis from other primary sites. The nurse demonstrates understanding of the article when identifying which primary sites as commonly metastasizing to the spinal cord? Select all that apply. - Lung - Breast - Gastrointestinal tract - Bladder - Prostate

Correct response: Lung Breast Gastrointestinal tract Explanation: Cancer can spread to the spinal cord from any primary site. However, the three most common cancers that metastasize to the spinal cord are lung, breast, and those of the gastrointestinal tract.

A woman has been brought to the emergency department (ED) by her distraught husband who believes that she has had a stroke. A rapid assessment by the care team confirms that the husband's suspicions are likely accurate, and the woman is being screened for the possible administration of recombinant tissue plasminogen activator (r tPA). Which of the following factors would contraindicate the use of tPA? - The woman has hypertension and type 1 diabetes. - The woman has previously had a stroke. - The woman's stroke has a hemorrhagic etiology. - The woman is older than 80 years of age.

Correct response: The woman's stroke has a hemorrhagic etiology. Explanation: tPA is contraindicated in hemorrhagic stroke because it would greatly exacerbate cerebral bleeding. Older age, previous stroke, hypertension, and diabetes do not necessarily contraindicate the use of tPA.

Which of the following would lead a nurse to suspect that a client has a rotator cuff tear? A Increased ability to stretch arm over the head B Pain worse in the morning C Minimal pain with movement D Difficulty lying on affected side

D

Which term refers to the failure of fragments of a fractured bone to heal together? A Malunion B Dislocation C Subluxation D Nonunion

D

To avoid the side effects of corticosteroids, which medication classification is used as an alternative to treating inflammatory conditions of the eyes? Mydriatics Nonsteroidal anti-inflammatory drugs (NSAIDs) Cycloplegics Miotics

Nonsteroidal anti-inflammatory drugs (NSAIDs) NSAIDs are used as an alternative in controlling inflammatory eye conditions and postoperatively to reduce inflammation. Miotics are used to cause the pupil to constrict. Mydriatics cause the pupil to dilate. Cycloplegics cause paralysis of the iris sphincter.

A client is color blind. The nurse understands that this client has a problem with: rods. lens. aqueous humor. cones.

cones. Cones provide daylight color vision, and their stimulation is interpreted as color. If one or more types of cones are absent or defective, color blindness occurs. Rods are sensitive to low levels of illumination but can't discriminate color. The lens is responsible for focusing images. Aqueous humor is a clear watery fluid and isn't involved with color perception.

A client with a T4-level spinal cord injury (SCI) is experiencing autonomic dysreflexia; his blood pressure is 230/110. The nurse cannot locate the cause and administers antihypertensive medication as ordered. The nurse empties the client's bladder and the symptoms abate. Now, what must the nurse watch for? 1- Rebound hypotension 2- Rebound hypertension 3- Urinary tract infection 4- Spinal shock

1

The upper eyelid normally covers the uppermost portion of the iris and is innervated by which cranial nerve? 1- III 2- I 3- IV 4- II

1

At a certain point, the brain's ability to autoregulate becomes ineffective and decompensation (ischemia and infarction) begins. Which of the following are associated with Cushing's triad? Select all that apply. 1- Bradycardia 2- Hypertension 3- Bradypnea 4- Hypotension 5- Tachycardia

1,2,3

When assessing the pressure of the anterior chamber of the eye, a nurse normally expects to find a pressure of: 20 to 30 mm Hg. 5 to 10 mm Hg. over 30 mm Hg. 10 to 20 mm Hg.

10 to 20 mm Hg. Normally, pressure in the anterior chamber of the eye remains relatively constant at 10 to 20 mm Hg.

A patient has had a stroke and is unable to move the right upper and lower extremity. During assessment the nurse picks up the arm and it is limp and without tone. How would the nurse document this finding? 1- Rigidity 2- Flaccidity 3- Atonic 4- Tetanic

2

The nurse is reading the admission note for a client with a bone fracture that requires surgery. The note indicates the presence of crepitus. The nurse interprets this as being 1- bleeding. 2- a crackling sound. 3- ecchymosis. 4- a closed fracture.

2

A nurse is assisting during a lumbar puncture. How should the nurse position the client for this procedure? 1- Prone, with the head turned to the right 2- Supine, with the knees raised toward the chest 3- Lateral recumbent, with chin resting on flexed knees 4- Lateral, with right leg flexed

3

Which client would the nurse identify as having the greatest risk for osteoporosis? 1- A 40-year-old overweight African American woman 2- A 16-year-old male with a history of asthma 3- A small-framed, thin 45-year-old white woman 4- A 20-year-old male athlete with repeated injuries

3

Which condition occurs when blood collects between the dura mater and arachnoid membrane? 1- Intracerebral hemorrhage 2- Epidural hematoma 3- Extradural hematoma 4- Subdural hematoma

4

Which hormone inhibits bone resorption and increases the deposit of calcium in the bone? 1- Growth hormone 2- Vitamin D 3- Sex hormones 4- Calcitonin

4

Which statement is consistent with acute otitis media? 1- The infection usually lasts more than 6 weeks. 2- It is a relatively uncommon childhood infection. 3- It is usually caused by a fungal infection. 4- Conductive hearing loss may occur.

4

Which term refers to the shaft of the long bone? 1- Epiphysis 2- Lordosis 3- Scoliosis 4- Diaphysis

4

A nursing student is writing a care plan for a newly admitted patient who has been diagnosed with a stroke. What major nursing diagnosis should most likely be included in the patients plan of care? A) Adult failure to thrive B) Post-trauma syndrome C) Hyperthermia D) Disturbed sensory perception

Ans: D Feedback: The patient who has experienced a stroke is at a high risk for disturbed sensory perception. Stroke is associated with multiple other nursing diagnoses, but hyperthermia, adult failure to thrive, and post- trauma syndrome are not among these.

A middle-aged woman has sought care from her primary care provider and undergone diagnostic testing that has resulted in a diagnosis of MS. What sign or symptom is most likely to have prompted the woman to seek care? A) Cognitive declines B) Personality changes C) Contractures D) Difficulty in coordination

Ans: D Feedback: The primary symptoms of MS most commonly reported are fatigue, depression, weakness, numbness, difficulty in coordination, loss of balance, spasticity, and pain. Cognitive changes and contractures usually occur later in the disease.

Which is a hallmark sign of compartment syndrome? A Weeping skin surfaces B Edema C Pain D Motor weakness

C

Which nursing intervention is essential in caring for a client with compartment syndrome? A Starting an I.V. line in the affected extremity in anticipation of venogram studies B Wrapping the affected extremity with a compression dressing to help decrease the swelling C Removing all external sources of pressure, such as clothing and jewelry D Keeping the affected extremity below the level of the heart

C

The clinic nurse caring for a patient with Parkinson's disease notes that the patient has been taking levodopa and carbidopa (Sinemet) for 7 years. What common side effects of Sinemet would the nurse assess this patient for? - Diarrhea - Lactose intolerance - Pruritus - Dyskinesia

Correct response: Dyskinesia Explanation: Within 5 to 10 years of taking levodopa, most patients develop a response to the medication characterized by dyskinesia (abnormal involuntary movements). Another potential complication of long-term dopaminergic medication use is neuroleptic malignant syndrome, characterized by severe rigidity, stupor, and hyperthermia. Side effects of long-term Sinemet therapy are not pruritus, lactose intolerance, or diarrhea.

A patient has been admitted to the intensive care unit (ICU) for the treatment of bacterial meningitis. The ICU nurse is aware of the need for aggressive treatment and vigilant nursing care because meningitis has the potential to cause what sequela? - Hydrocephalus - Glioma - Increased intracranial pressure (ICP) - Cerebrovascular accident (CVA)

Correct response: Increased intracranial pressure (ICP) Explanation: Increased ICP is a significant risk in patients being treated for meningitis. This infection does not cause brain tumors, hydrocephalus, or CVA.

A patient visits a clinic for an eye examination. He describes his visual changes and mentions a specific diagnostic clinical sign of glaucoma. What is that clinical sign? Diminished acuity The presence of halos around lights A significant loss of central vision Pain associated with a purulent discharge

The presence of halos around lights Colored halos around lights is a classic symptom of acute-closure glaucoma.

When conducting an eye exam, the nurse practitioner is aware that a diagnostic clinical manifestation of glaucoma is: Diminished acuity. A significant loss of central vision. The presence of halos around lights. Pain associated with a purulent discharge.

The presence of halos around lights. Most patients are unaware that they have glaucoma until they experience visual changes and vision loss. Usually the patient notices blurred vision and the presence of "halos" around lights.

A client comes to the clinic for an ophthalmologic screening, which will include measurement of intraocular pressure (IOP) with a tonometer. Which statement about this procedure is true? A topical anesthetic will be administered after the examination. The client should wear dark glasses for several hours after the procedure. The client will direct the gaze forward while the physician rests the tonometer on the scleral surface. The tonometer will register the force required to indent or flatten the corneal apex.

The tonometer will register the force required to indent or flatten the corneal apex. The tonometer will register the force required to indent (using Schiotz's tonometer) or flatten (using an applanation tonometer) the corneal apex. This force varies with firmness of the eye, which fluctuates with IOP. Although the client does direct the gaze forward during tonometry, the tonometer rests on the surface of the cornea, not the sclera. Topical anesthetic drops are administered before, not after, the examination. The client should wear dark glasses after pupil dilation, not tonometry, to protect the eyes from light.

A client accidentally splashes chemicals into one eye. The nurse knows that eye irrigation with plain tap water should begin immediately and continue for 15 to 20 minutes. What is the primary purpose of this first aid treatment? To serve as a stopgap measure until help arrives To eliminate the need for medical care To hasten formation of scar tissue To prevent vision loss

To prevent vision loss Prolonged eye irrigation after a chemical burn is the most effective way to prevent formation of permanent scar tissue and thus help prevent vision loss. After a potentially serious eye injury, the victim should always seek medical care. Eye irrigation isn't considered a stopgap measure.

A boy has been brought to the clinic by his mother because of redness, swelling, and tenderness in the conjunctiva of his left eye. The nurse's inspection of the boy's eye also reveals the presence of purulent discharge in his left medial canthus. The nurse should anticipate that the boy will require what intervention? Topical antibiotics Instillation of a hypertonic solution Application of a colloidal silver pad Oral antiviral medication

Topical antibiotics This patient's signs and symptoms are characteristic of bacterial conjunctivitis, which is frequently treated with topical antibiotics. Viral conjunctivitis typically produces watery eye discharge. Hypertonic solutions and colloidal silver are not normally used in the treatment of bacterial conjunctivitis.

A patient is suspected of having retinal detachment. The nurse would expect to prepare the patient for which of the following? Select all that apply. Fluorescein angiography Slit lamp biomicroscopy Indirect ophthalmoscopy Tonometry Visual acuity testing Amsler grid testing

Visual acuity testing Indirect ophthalmoscopy Slit lamp biomicroscopy Fluorescein angiography Testing for retinal detachment includes visual acuity testing, indirect ophthalmoscopy, slit lamp biomicroscopy, and fluorescein angiography. The Amsler grid is used to evaluate for macular degeneration. Tonometry is used to evaluate for glaucoma.

Hypercalcemia is a dangerous complication of bone cancer. Therefore, nursing assessment includes evaluation of symptoms that require immediate treatment. Which of the following are signs/symptoms that are indictors of an elevated serum calcium? Select all that apply. a. Muscle weakness b. Tachycardia c. Anorexia and constipation d. Prolonged ST segment e. Shortened QT interval f. Lack of muscle coordination

a,c,e,f

A client has just been diagnosed with early glaucoma. During a teaching session, the nurse should: demonstrate eyedrop instillation. assess the client's visual acuity. provide instructions on eye patching. teach about intraocular lens cleaning.

demonstrate eyedrop instillation. Eyedrop instillation is a critical component of self-care for a client with glaucoma. After demonstrating eyedrop instillation to the client and family, the nurse should verify their ability to perform this measure properly. An eye patch isn't necessary unless the client has undergone surgery. Visual acuity assessment isn't necessary before discharge. Intraocular lenses aren't implanted in clients with glaucoma.

A client has noticed recently having clearer vision at a distance than up close. What is the term used to describe this client's visual condition? hyperopia astigmatism myopia emmetropia

hyperopia Hyperopia is farsightedness. People who are hyperopic see objects that are far away better than objects that are close.

The nurse is caring for a client who is postoperative from surgery for a brain tumor resection. The client has a visitor at the bedside who lowers the head of the bed below 30 degrees. The nurse assesses the client has decreased level of consciousness. What actions should the nurse take? Select all that apply. - Assess for presence of visual changes - Check for leaks on the surgical site dressing - Check the client's blood pressure - Review chart to check for high white blood cell count - Assess the client for headache

orrect response: Check the client's blood pressure Assess the client for headache Assess for presence of visual changes Explanation: The client is most likely developing increased intracranial pressure (ICP). With suspicion of ICP, the nurse should assess for ominous signs such as hypertension, bradycardia and respiratory depression. These are serious late signs of ICP and constitute an emergency. Headache is thought to be caused by the tumor's invading, compressing, or distorting the pain-sensitive structures or by edema that accompanies the tumor. Thus, headaches are related to intracerebral edema and increasing ICP. Visual changes can result from ICP, which is referred to as papilledema. Papilledema results from edema on the optic nerve due to increased ICP. Leaking at the surgical site do not arise from increase ICP but can be a sign of infection or inflammation and warrant intervention. A high white blood cell count would indicate infection but would not necessarily be linked to ICP.

The nurse realizes that a client understands how to correctly instill ophthalmic medications when the client: wipes the lids and lashes prior to instillation in a direction toward the nose with moistened, soft gauze. allows the tip of the container to touch the eyelid while administering the medication. pulls the tissue near the cheek downward to instill medication. rubs the eye after administering medication.

pulls the tissue near the cheek downward to instill medication. Pull the cheek downward to form a sack in the lower lid. Instill the drops into the conjunctival pocket. The lid and lashes would be wiped in a direction away from the nose to avoid contamination. The tip of the container will be contaminated if it touches eye or eyelid. Do not rub the eye because it may irritate the eye.

After a fall at home, a client hits their head on the corner of a table. Shortly after the accident, the client arrives at the ED, unable to see out of their left eye. The client tells the nurse that symptoms began with seeing spots or moving particles in the field of vision but that there was no pain in the eye. The client is very upset that the vision will not return. What is the most likely cause of this client's symptoms? angle-closure glaucoma eye trauma chalazion retinal detachment

retinal detachment A detached retina is associated with a hole or tear in the retina caused by stretching or degenerative changes. Retinal detachment may follow a sudden blow, penetrating injury, or eye surgery.

When the client tells the nurse that his vision is 20/200 and then asks what that means, the nurse informs the client that a person with 20/200 vision sees an object from 20 feet away just like a person with normal vision. sees an object from 20 feet away that a person with normal vision sees from 200 feet away. sees an object from 200 feet away that a person with normal vision sees from 20 feet away. sees an object from 200 feet away just like a person with normal vision.

sees an object from 20 feet away that a person with normal vision sees from 200 feet away. The fraction 20/20 is considered the standard of normal vision. Most people can see the letters on an eye chart designated as 20/20 from a distance of 20 feet.

A client is prescribed eye drops which cause mydriasis. When evaluating the effect of these drops, the nurse would assess for: reduced conjunctival redness. pupil dilation. reduced intraocular pressure. pupil constriction.

pupil dilation. Mydriatic agents cause pupil dilation; miotic agents cause pupil constriction. Drops used to treat glaucoma lower intraocular pressure. Anti-inflammatory drops aid in reducing redness and irritation of the conjunctiva.

A patient with Bell's palsy asks the nurse why she has to wear an eye shield at night. What would be the nurse's best response? - "A patient with Bell's palsy often can't close the eye, which causes insomnia." - "A patient with Bell's palsy needs an eye shield at night to prevent injury to the eye." - "A patient with Bell's palsy can't blink, so the eye shield keeps the eye moist." - "A patient with Bell's palsy needs extra coverage for their eye."

"A patient with Bell's palsy needs an eye shield at night to prevent injury to the eye." Frequently, the eye does not close completely and the blink reflex is diminished, so the eye is vulnerable to injury from dust and foreign particles. Corneal irritation, corneal abrasion, and ulceration may occur. Distortion of the lower lid alters the proper drainage of tears. To prevent injury, the eye may be covered with a protective shield at night. The eye patch may abrade the cornea, however, because there is some difficulty in keeping the partially paralyzed eyelids closed. Eye ointment may be applied at bedtime to promote adherence of the eyelids to prevent injury during sleep. Artificial tears are typically instilled to maintain eye lubrication. The patient can be taught to close the paralyzed eyelid manually before going to sleep. Wrap-around sunglasses or goggles may be worn during the day to decrease normal evaporation from the eye.

A client has undergone a cataract extraction with an intraocular lens implant. After providing discharge instructions, the nurse determines that the client needs additional teaching based on which statement? "My vision will be back to normal immediately after surgery." "I should wear the eye patch for the first 24 hours after surgery." "I might feel a scratchy sensation for the first couple of days." " I need to wear sunglasses if I'm outside."

"My vision will be back to normal immediately after surgery." After surgery, the client's vision gradually improves as the eye heals. Although clients with intraocular implants have functional vision on the first day after surgery, vision is stabilized when the eye is completely healed, usually within 6 to 12 weeks. To prevent accidental rubbing or poking of the eye, the client wears a protective eye patch for 24 hours after surgery, followed by eyeglasses worn during the day and a metal shield worn at night for 1 to 4 weeks. Sunglasses should be worn while outdoors during the day because the eye is sensitive to light. Slight morning discharge, some redness, and a scratchy feeling may be expected for a few days.

A nurse is discussing vision loss with a group of participants at a community-based health fair. The nurse knows that many individuals are unaware of the deleterious effects of glaucoma and is briefly explaining the disease to the group. What should the nurse teach these participants about the signs and symptoms of glaucoma? "Glaucoma causes excess tear production as the high-pressure fluid in the eye is shunted away." "Unfortunately, many people have no idea they have glaucoma until their vision is affected." "Many people with glaucoma get itchy, reddened eyes but attribute it to a cold or allergies." "Glaucoma causes significant pain when it first occurs but many people neglect to seek care or treatment."

"Unfortunately, many people have no idea they have glaucoma until their vision is affected." Glaucoma is often called the "silent thief of sight" because most patients are unaware that they have the disease until they have experienced visual changes and vision loss. Pain is a late sign, and it does not result in increased tear production. The disease does not normally cause itchy, reddened eyes.

A client with a spinal cord injury is to receive Lovenox (enoxaparin) 50 mg subcutaneously twice a day. The medication is supplied in vials containing 80 mg per 0.8 mL. How many mL will constitute the correct dose? Enter the correct number ONLY.

0.5 mL

A 36-year-old male patient is preparing for discharge from the hospital to a rehabilitative facility 4 weeks after he suffered a spinal cord injury (SCI) during a workplace accident. The hospital nurse should be aware that the primary focus of this coming phase of the patient's recovery will be: 1- Providing him with the skills to perform as many activities of daily living (ADLs) as possible 2- Ensuring that he adheres to the prescribed treatment regimen before being discharged home 3- Helping him establish therapeutic relationships with people who have had similar injuries 4- Allowing him to receive care in a setting that is less institutional than a hospital

1

A 77-year-old female patient who is recovering in the hospital from a total knee replacement has rung her call bell and told the nurse that she needs pain medication. When assessing the patient's pain, what principle should the nurse bear in mind? 1- Older adults tend to have a blunted pain sensation, so complaints should be followed-up promptly. 2- Older adults frequently confuse pain with other tactile sensations. 3- Pain in older adults is often unrelated to physical harm or pathophysiological processes. 4- The sensation of pain increases with age, so older adults typically feel more pain for a longer period than younger patients.

1

A client comes to the emergency department, reporting that a bee has flown into his ear and is stuck. The client reports a significant amount of pain. Which of the following would be most appropriate to use to remove the bee? 1- Mineral oil 2- Irrigation 3- Hair pin 4- Tweezers

1

A client comes to the walk-in clinic complaining of a "bug in my ear." What action should be taken when there is an insect in the ear? 1- Instillation of mineral oil 2- Instillation of carbamide peroxide 3- Instillation of hot water 4- Use of a small forceps

1

A client has a history of hearing loss and is returning for an annual hearing examination. During client education, the nurse explains that hearing involves which areas of the ear? 1- all sections 2- outer section 3- middle section 4- inner section

1

A client has been diagnosed with otosclerosis. The nurse explains to the client that this is a common cause of hearing impairment among adults and is the result of a bony overgrowth of the: 1- stapes 2- labyrinth 3- tympanic membrane 4- incus

1

A client has sustained a traumatic brain injury with involvement of the hypothalamus. The nurse is concerned about the development of diabetes insipidus. Which of the following would be an appropriate nursing intervention to monitor for early signs of diabetes insipidus? 1- Take daily weights. 2- Reposition the client frequently. 3- Assess for pupillary response frequently. 4- Assess vital signs frequently.

1

A client has undergone a myringotomy. The nurse interprets this as which of the following? 1- Incision of the eardrum 2- Placement of ventilation tubes 3- Surgical reconstruction of the eardrum 4- Reconstruction of the middle ear bones

1

A client is being prepared for a cochlear implant. Which client statement would alert the nurse to the need for additional teaching? 1- "I'm going to be able to hear normally again." 2- "I'll have a small incision behind my ear." 3- "I'll wear an external transmitter and microphone." 4- "I'll be able to hear medium and loud sounds for once."

1

A client is being treated for increased intracranial pressure (ICP). The nurse should ensure that the client does not develop hypothermia because: 1- shivering in hypothermia can increase ICP. 2- hypothermia is indicative of severe meningitis. 3- hypothermia is indicative of malaria. 4- hypothermia can cause death to the client.

1

A client is having a routine eye examination. The procedure being performed is done by using an instrument to indent or flatten the surface of the eye. This is known as ________ and it is routinely done to test for ________. 1- tonometry; intraocular pressure 2- retinoscopy; detached retina 3- tonometry; macular degeneration 4- retinoscopy; cataracts

1

A client is scheduled to undergo an electromyography. When performed, what will this test evaluate? 1- Muscle weakness 2- Muscle composition 3- Bone density 4- Metastatic bone lesions

1

A client presents to the emergency department status post-seizure. The physician wants to know what the pressure is in the client's head. What test might be ordered on this client? 1- Lumbar puncture 2- Echoencephalography 3- Nerve conduction studies 4- EMG

1

A client suffered a closed head injury in a motor vehicle collision, and an ICP monitor was inserted. In the occurrence of increased ICP, what physiologic function contributes to the increase in intracranial pressure? 1- vasodilation 2- vasoconstriction 3- hypertension 4- increased PaO

1

A client suffered trauma to the sclera and is being treated for a subsequent infection. During client education, the nurse indicates where the sclera is attached. Which structure would not be included? 1- eyelids 2- cornea 3- iris 4- pupil

1

A client undergoing mastoid surgery asks the nurse about the pain following the surgery. Which response by the nurse is appropriate? 1- "Usually the incisional pain is mild and controlled by the prescribed medication for the first 24 hours." 2- "The incisional pain usually last 3 weeks. The doctor will make sure you have enough pain medications." 3- "Most client report a sharp shooting pain for 1 to 2 months following the surgery from the eustachian tube opening." 4- "Usually there is a constant throbbing pain for the first week. Most client report no pain with the use of the pain medications."

1

A client was hit in the head with a ball and knocked unconscious. Upon arrival at the emergency department and subsequent diagnostic tests, it was determined that the client suffered a subdural hematoma. The client is becoming increasingly symptomatic. How would the nurse expect this subdural hematoma to be classified? 1- acute 2- chronic 3- subacute 4- intracerebral

1

A client who has a lengthy history of progressive hearing loss is very forthright about the condition, and the nurse wants to develop a communication strategy for this client's hospital stay. Which communication strategy has proven to be the most effective? 1- the one the client will use 2- speech reading 3- signing 4- writing

1

A client with a traumatic brain injury has already displayed early signs of increasing intracranial pressure (ICP). Which of the following would be considered late signs of increasing ICP? 1- Decerebrate posturing and loss of corneal reflex 2- Loss of gag reflex and mental confusion 3- Complaints of headache and lack of pupillary response 4- Mental confusion and pupillary changes

1

A client with a traumatic brain injury has developed increased intracranial pressure resulting in dibetes insipidus. While assessing the client, the nurse expects which of the following findings? 1- Excessive urine output and decreased urine osmolality 2- Oliguria and decreased urine osmolality 3- Oliguria and serum hyperosmolarity 4- Excessive urine output and serum hypo-osmolarity

1

A client with multiple sclerosis is being seen by a neuroophthalmologist for a routine eye exam. The nurse explains to the client that during the examination, the client will be asked to maintain a fixed gaze on a stationary point while an object is moved from a point on the side, where it can't be seen, toward the center. The client will indicate when the object becomes visible The nurse further explains that the test being performed is called a: 1- perimetry test 2- color vision test 3- slit-lamp examination 4- retinal angiography

1

A client's spouse relates how the client reported a severe headache and then was unable to talk or move their right arm and leg. After diagnostics are completed and the client is admitted to the hospital, when would basic rehabilitation begin? 1- immediately 2- in 2 to 3 days 3- after 1 week 4- upon transfer to a rehabilitation unit

1

A dietary modification for a patient with Ménière's disease would be: 1- A decrease in sodium intake to 2,000 mg daily. 2- Fluid restriction to 2 L/day. 3- An increase in calcium to 1 g/day. 4- An increase in vitamin C to 1.5 g/day.

1

A female patient's severe ear infection has resulted in tympanic membrane perforation. The patient's care provider has concluded that surgical repair is unnecessary because the membrane is healing spontaneously. What health education should the nurse emphasize to this patient? 1- It is important to keep the ear canal dry until the membrane has healed. 2- Half-strength hydrogen peroxide should be instilled into the ear once a day. 3- The ear should be gently cleaned with a cotton swab once a day. 4- Purulent drainage should be expected for the first week after the injury.

1

A has experienced increasing pain and progressing inflammation of the hands and feet. The rheumatologist has prescribed NSAID use to treat the condition. What client education is most important for the nurse to address with the use of these medications? 1- common adverse effects 2- dietary restrictions 3- activity restrictions 4- loading-dose schedule

1

A neurological nurse is conducting a scheduled assessment of a patient who is receiving care on the unit. The nurse is aware of the need to conduct a vigilant assessment of the patient's level of consciousness (LOC). How should the nurse best gauge a patient's LOC? 1- By assessing according to the Glasgow Coma Scale (GCS) 2- By eliciting the patient's response to a question requiring judgment 3- By engaging the patient in a conversation, if possible 4- By observing the patient's interactions with caregivers

1

A nurse in a rehabilitation facility is coordinating the discharge of a client who is tetraplegic. The client, who is married and has two children in high school, is being discharged to home and will require much assistance. Who would the discharge planner recognize as being the most important member of this client's care team? 1- spouse 2- chaplin 3- home care nurse 4- physical therapist

1

A nurse is caring for a client with lower back pain who is scheduled for myelography using metrizamide (a water-soluble contrast dye). After the test, the nurse should place the client in which position? 1- Head of the bed elevated 45 degrees 2- Prone 3- Supine with feet raised 4- Supine with the head lower than the trunk

1

A nurse is conducting a neurological assessment of a patient who has just been admitted to the unit. In preparation for assessing the patient for pronator drift, what instructions should the nurse provide to the patient? 1- "Please hold your arms straight out with your palms pointing up to the ceiling." 2- "Please close your eyes and then walk a few steps with one foot directly in front of the other." 3- "Please close your eyes and then touch the tip of your nose with one index finger and then the other." 4- "Please lift one leg a few inches off the bed and hold it as still as possible."

1

A nurse is continually monitoring a client with a traumatic brain injury for signs of increasing intracranial pressure. The cranial vault contains brain tissue, blood, and cerebrospinal fluid; an increase in any of the components causes a change in the volume of the others. This hypothesis is called which of the following? 1- Monro-Kellie 2- Cushing's 3- Dawn phenomenon 4- Hashimoto's disease

1

A nurse is performing a neurologic assessment on a client with a stroke and cannot elicit a gag reflex. This deficit is related to cranial nerve (CN) X, the vagus nerve. What will the nurse consider a priority nursing diagnosis? 1- Risk for aspiration 2- Risk for falls 3- Risk for impaired skin integrity 4- Decreased intracranial adaptive capacity

1

A nurse is performing a neurologic assessment on the client and notes a positive Romberg test. This test for balance is related to which of the following cranial nerves? 1- VIII 2- X 3- III 4- VII

1

A nurse is performing an eye examination. Which question would not be included in the examination? 1- "Are you able to raise both eyebrows?" 2- Have you experienced blurred, double, or distorted vision?" 3- "Do any family members have any eye conditions?" 4- "What medications are you taking?"

1

A nurse is preparing a plan of care for a client with otitis externa. Based on the typical assessment findings, which of the following would the nurse most likely identify as the priority nursing diagnosis? 1- Acute pain related to inflammation 2- Risk for infection related to drainage from the ear canal 3- Disturbed sensory perception: auditory related to sensorineural hearing loss. 4- Hyperthermia related to elevated temperature secondary to infection

1

A nurse practitioner explains to a patient recently diagnosed with noise-induced hearing loss that the basic altered physiology in the ear is the result of: 1- Damage to the hair cells inside the cochlea. 2- Stenosis of the semicircular canals. 3- Hardening of the tympanic membrane. 4- Malfunctioning of the incus and the staples.

1

A nurse practitioner who works with many older adult patients integrates screening for osteoporosis into health promotion activities. What screening tool is most clinically useful when screening for osteoporosis? 1- Hip bone mineral density (BMD) testing 2- Assessment of serum calcium levels 3- Bone biopsy 4- Assessment of 25-hydroxy-vitamin D levels

1

A nurse who oversees care in a long-term care facility is aware that a high percentage of residents have osteoporosis, and that residents who do not have the disease must be assessed and monitored closely for this health problem. Which of the following older adults most clearly exemplifies the risk factors for osteoporosis? 1- A Caucasian woman who has low body mass index 2- An African American woman who is slightly obese 3- An Asian man whose mobility is limited to a wheelchair 4- A Caucasian man who has led a sedentary lifestyle

1

A nurse who provides care in an extended-care setting works primarily with adults aged 75 and older. When planning care and implementing safety measures for this population, the nurse should be cognizant of which of the following normal, age-related changes to the musculoskeletal system? 1- Decreased bone mass 2- Presence of osteochondromas 3- Decreased collagen in muscles 4- Increased elasticity of tendons

1

A nurse who provides care in an ophthalmologist's office has received training on the technique for assessing patients' intraocular pressure (IOP). This nurse will possess the skills necessary to perform what type of diagnostic testing? 1- Tonometry 2- Direct ophthalmoscopy 3- Perimetry testing 4- Slit-lamp examination

1

A nurse working on a medical-surgical floor walks into a patient's room to find the patient with an altered level of consciousness (LOC). Which of the following nursing diagnoses would be the first priority for the plan of care? 1- Ineffective airway clearance related to altered LOC 2- Risk of injury related to decreased LOC 3- Deficient fluid volume related to inability to take fluids by mouth 4- Risk for impaired skin integrity related to prolonged immobility

1

A nursing instructor is teaching pre-nursing students in a pathophysiology class. What would the instructor teach the students about Ménière's disease? 1- It is referred to as endolymphatic hydrops. 2- It originates in the middle ear. 3- It is referred to as lymphatic hydrops. 4- It originates in the outer ear.

1

A patient admitted for the treatment of a nondepressed skull fracture has been leaking clear fluid from his nose, and glucose testing confirms that it is cerebrospinal fluid (CSF). This development necessitates what nursing action? 1- Elevating the head of the bed to 30 degrees 2- Performing gentle nasal suctioning at 20 to 30 mm Hg 3- Insertion of a nasogastric (NG) tube to low suction 4- Positioning the patient side-lying

1

A patient arrives to have an MRI done in the outpatient department. What information provided by the patient warrants further assessment to prevent complications related to the MRI? 1- "I am trying to quit smoking and have a patch on." 2- "I have been trying to get an appointment for so long." 3- "I have not had anything to eat or drink since 3 hours ago." 4- "My legs go numb sometimes when I sit too long."

1

A patient comes to the clinic and informs the nurse of numbness, tingling, and a burning sensation in the arm from the elbow down to the fingers. What type of symptom would this be documented as? 1- Paresthesia 2- Flaccidity 3- Atonia 4- Effusion

1

A patient comes to the emergency department with a large scalp laceration after being struck in the head with a glass bottle. After assessment of the patient, what does the nurse do before the physician sutures the wound? 1- Irrigates the wound to remove debris 2- Administers an oral analgesic for pain 3- Administers acetaminophen (Tylenol) for headache 4- Shaves the hair around the wound

1

A patient has just had an arthroscopy. What is a nursing intervention that is necessary for the nurse to implement following an arthroscopy? 1- Wrap the joint in compression dressing. 2- Flex and lower the joint. 3- Extend and lower the joint. 4- Apply heat.

1

A patient is to have an angiography done using fluorescein as a contrast agent to determine if the patient has macular edema. What laboratory work should the nurse monitor prior to the angiography? 1- BUN and creatinine 2- AST and ALT 3- Hemoglobin and hematocrit 4- Platelet count

1

A patient sustained a head injury during a fall and has changes in personality and affect. What part of the brain does the nurse recognize has been affected in this injury? 1- Frontal lobe 2- Parietal lobe 3- Occipital lobe 4- Temporal lobe

1

A patient with significant and increasing sensorineural hearing loss has opted not to have surgery and has chosen instead to focus on aural rehabilitation with an emphasis on auditory training. What will be the primary focus of the patient's rehabilitation? 1- Developing effective skills and strategies for listening 2- Learning to use sign language effectively 3- Developing coping techniques for dealing with a lack of communication 4- Developing nonverbal communication techniques

1

A young baseball player was fielding a ground ball when it bounced and struck the player in the left eye, leaving a large ecchymosis and edema. In client education, after applying an ice pack, the nurse explains the functions of the various structures of the eye. What glands, contained in the eyelids, produce tears? 1- lacrimal 2- sebaceous 3- sweat 4- olfactory

1

A young man has presented for care because he claims that he has experienced hearing loss that he attributes to a summer job at a gravel crusher. On questioning, the clinician learns that the man was provided with hearing protection but was "hit and miss" with using it. What should the clinician teach this patient about sensorineural hearing loss? 1- "Hearing loss like you've experienced is permanent because of the destruction of important cells in your ears." 2- "You'll likely find that your hearing will improve over the next several years." 3- "As long as you're very careful to avoid loud noises, your hearing will probably recover in the following few months." 4- "Unfortunately, your hearing loss is likely to get progressively worse because of the process that this damage started."

1

An 87-year-old client is receiving a hearing test because she has perceived a change in her hearing. During your client education session, you indicate the lowest level of sound that individuals may normally hear. What is that level? 1- 20 dB 2- 10 dB 3- 5 dB 4- 25 dB

1

An 88-year-old client has a history of hearing loss and is returning for his annual hearing examination. During your client education session, you explain to the client that hearing involves which areas of the ear? 1- All sections of the ear are involved in perceiving sound. 2- The outer section of the ear is involved in perceiving sound. 3- The middle section of the ear is involved in perceiving sound. 4- The inner section of the ear is involved in perceiving sound.

1

An instructor is describing the process of bone development. Which of the following would the instructor describe as being responsible for the process of ossification? 1- Osteoblasts 2- Cortical bone 3- Osteoclasts 4- Cancellous bone

1

An older adult patient has noticed a significant amount of vision loss in the last few years. What does the nurse recognize as the most common cause of visual loss in older adults? 1- Macular degeneration 2- Ocular trauma 3- Retinal vascular disease 4- Uveitis

1

Choose the correct statement about the endosteum, a significant component of the skeletal system: 1- Covers the marrow cavity of long bones 2- Supports the attachment of tendons to bones 3- Contains blood vessels and lymphatics 4- Facilitates bone growth

1

Cranial nerve IX is also known as which of the following? 1- Glossopharyngeal 2- Vagus 3- Spinal accessory 4- Hypoglossal

1

During a neurological physical examination, a patient tells the nurse that she is having difficulty raising her right eyelid. The nurse knows that this may be an indication of problems with which cranial nerve? 1- III 2- IV 3- VI 4- VII

1

During a routine physical examination of a client, the nurse observes a flexion deformity of the promixal interphalangeal (PIP) joint of two toes on the right foot. How would the nurse document this finding? 1- Hammer toe 2- Mallet toe 3- Hallux valgus 4- Bunion

1

If untreated, squamous cell carcinoma of the external ear can spread through the temporal bone, causing which effect? 1- Facial nerve paralysis 2- Nystagmus 3- Motor impairment 4- Diplopia

1

Instructions for the client with low back pain include that, when lifting, the client should 1- avoid overreaching. 2- place the load away from the body. 3- use a narrow base of support. 4- bend the knees and loosen the abdominal muscles.

1

Morton neuroma is exhibited by which clinical manifestation? 1- Swelling of the third (lateral) branch of the median plantar nerve 2- High arm and a fixed equinus deformity 3- Diminishment of the longitudinal arch of the foot 4- Inflammation of the foot-supporting fascia

1

Morton neuroma is exhibited by which clinical manifestation? 1- Swelling of the third (lateral) branch of the median plantar nerve 2- High arm and a fixed equinus deformity 3- Longitudinal arch of the foot is diminished 4- Inflammation of the foot-supporting fascia

1

Skull sutures are an example of which type of joint? 1- Synarthrosis 2- Amphiarthrosis 3- Diarthrosis 4- Aponeuroses

1

The critical care nurse is giving end-of-shift report on a client she is caring for. The nurse uses the Glasgow Coma Scale (GCS) to assess the level of consciousness (LOC) of a female client and reports to the oncoming nurse that the client has an LOC of 6. What does an LOC score of 6 in a client indicate? 1- Comatose 2- Somnolence 3- Stupor 4- Normal

1

The homecare nurse is evaluating the musculoskeletal system of a geriatric client whose previous assessment was within normal limits. The nurse initiates a call to the health care provider and/or emergency services when which change is found? 1- Decreased right-sided muscle strength 2- Decreased flexibility 3- Decreased agility 4- Increased joint stiffness

1

The initial sign of increasing intracranial pressure (ICP) includes 1- decreased level of consciousness. 2- herniation. 3- vomiting. 4- headache.

1

The nurse has completed evaluating the client's cranial nerves. The nurse documents impairment of the right cervical nerves (CN IX and CN X). Based on these findings, the nurse should instruct the client to 1- refrain from eating or drinking for now. 2- have their spouse bring in the client's glasses. 3- wear any hearing aids while in the hospital. 4- use the walker when walking.

1

The nurse is assessing an older client's vision. The nurse integrates knowledge of which of the following during the assessment? 1- The power of the lens to accommodate will be decreased. 2- Increased fat will be around the orbit. 3- The skin around the eyes will be more elastic. 4- The depth of the eyeball will be increased, leading to myopia.

1

The nurse is caring for a 35-year-old man diagnosed with a back strain. What would be a priority point of discussion with this patient? 1- Avoid lifting heavy weights without assistance. 2- Focus on using back muscles during lifting. 3- Lift objects while holding the object away from the body. 4- Tighten the abdominal muscles and lock the knees during the lifting of an object.

1

The nurse is caring for a client diagnosed with Guillain-Barre syndrome. His spouse asks about recovery rates. The nurse can correctly relate which of the following? 1- Approximately 60% to 75% of clients recover completely. 2- Only a very small percentage (5% to 8%) of clients recover completely. 3- Usually 100% of clients recover completely. 4- No one with Guillain-Barre syndrome recovers completely.

1

The nurse is caring for a client experiencing hearing loss. The nurse uses the otoscope to assess the ear canal and tympanic membrane and notes a significant accumulation of cerumen. Which documentation of hearing loss type would be most accurate? 1- Conductive 2- Mixed 3- Central 4- Sensorineural

1

The nurse is caring for a client who has sustained a spinal cord injury (SCI) at C5 and has developed a paralytic ileus. The nurse will prepare the client for which of the following procedures? 1- Insertion of a nasogastric tube 2- A large volume enema 3- Digital stimulation 4- Bowel surgery

1

The nurse is caring for a client with a head injury. The client is experiencing CSF rhinorrhea. Which order should the nurse question? 1- Insertion of a nasogastric (NG) tube 2- Urine testing for acetone 3- Serum sodium concentration testing 4- Out of bed to the chair three times a day

1

The nurse is caring for a client with traumatic brain injury (TBI). Which clinical finding, observed during the reassessment of the client, causes the nurse the most concern? 1- Temperature increase from 98.0°F to 99.6°F 2- Urinary output increase from 40 to 55 mL/hr 3- Heart rate decrease from 100 to 90 bpm 4- Pulse oximetry decrease from 99% to 97% room air

1

The nurse is caring for a patient with Ménière's disease who is hospitalized with severe vertigo. What medication does the nurse anticipate administering to shorten the attack? 1- Meclizine (Antivert) 2- Furosemide (Lasix) 3- Cortisporin otic solution 4- Gentamicin (Garamycin) intravenously

1

The nurse is evaluating the client while taking the color vision test. Which response would the nurse anticipate when caring for a client with normal color vision? 1- The nurse would anticipate the client identifying numbers and shapes. 2- The nurse would anticipate a cross-eyed appearance. 3- The nurse would anticipate responding to the color names in the pictures. 4- The nurse would anticipate no differentiation in between colors.

1

The nurse is instructing a client's family members on the most incapacitating symptom of Ménière's disease. Which nursing instruction associated with the symptom is most helpful? 1- Assist the client when ambulating. 2- Keep a bucket beside the bed. 3- Ensure low lighting in the room. 4- Sit in front of the client when speaking.

1

The nurse is performing a neurological assessment. What will this assessment include? 1- Ask the client to plantar flex the toes. 2- Observe for capillary refill of the great toe. 3- Palpate the dorsalis pedis pulse. 4- Inspect the foot for edema.

1

The nurse is preparing the client for computed tomography. Which information should be given by the nurse? 1- "You must remain very still during the procedure." 2- "A small bit of tissue will be removed and sent to the lab." 3- "Fluid will be removed from you affected joint." 4- "A radioisotope will be given through an IV."

1

The nurse received the report from a previous shift. One of her clients was reported to have a history of basilar skull fracture with otorrhea. What assessment finding does the nurse anticipate? 1- The client has cerebral spinal fluid (CSF) leaking from the ear. 2- The client has ecchymosis in the periorbital region. 3- The client has an elevated temperature. 4- The client has serous drainage from the nose.

1

The nursing instructor is discussing the difference between sensorineural and conductive hearing loss with his class. The discussion turns to evaluation for determining what kind of hearing loss a patient has. What Weber test result would indicate the presence of a sensorineural loss? 1- The sound is heard better in the ear in which he has better hearing. 2- The sound is heard equally in both ears. 3- The sound is heard better in the ear in which he has poorer hearing. 4- The sound is heard longer in the ear in which he has better hearing.

1

The parent of a young client with severe hearing loss is quite concerned about the child's future independence because of impaired hearing. Which type of hearing loss is usually irreversible? 1- sensorineural 2- conductive 3- tinnitus 4- noise exposure

1

The physician suggests that a client use meclizine as treatment for his motion sickness. The nurse explains the rationale for this drug based on an understanding of which of the following as the drug's action? 1- Blocks conduction of the vestibular pathways 2- Antagonizes the response of histamine 3- Depresses the central nervous system 4- Lowers endolymphatic system pressure

1

There are thousands of components of the musculoskeletal system that facilitate mobility and independent function. The function of skeletal muscle is promoting: 1- movement of skeletal bones. 2- organ function. 3- involuntary function. 4- All options are correct.

1

What part of the brain controls and coordinates muscle movement? 1- Cerebellum 2- Cerebrum 3- Midbrain 4- Brain stem

1

When performing a postoperative assessment on a client who has undergone surgery to manage increased intracranial pressure (ICP), a nurse notes an ICP reading of 0 mm Hg. Which action should the nurse perform first? 1- Check the equipment. 2- Contact the physician to review the care plan. 3- Continue the assessment because no actions are indicated at this time. 4- Document the reading because it reflects that the treatment has been effective.

1

When the nurse observes that the patient has extension and external rotation of the arms and wrists, and extension, plantar flexion, and internal rotation of the feet, she records the patient's posturing as which of the following? 1- Decerebrate 2- Normal 3- Flaccid 4- Decorticate

1

Which Glasgow Coma Scale score is indicative of a severe head injury? 1- 7 2- 9 3- 11 4- 13

1

Which are characteristics of autonomic dysreflexia? 1- severe hypertension, slow heart rate, pounding headache, sweating 2- severe hypotension, tachycardia, nausea, flushed skin 3- severe hypertension, tachycardia, blurred vision, dry skin 4- severe hypotension, slow heart rate, anxiety, dry skin

1

Which condition refers to hearing loss associated with degenerative changes? 1- Presbycusis 2- Myopia 3- Strabismus 4- Presbyopia

1

Which instructions regarding swimming should the nurse give to a client who is recovering from otitis externa? Select all that apply. 1- Wear soft plastic earplugs. 2- Wear a scarf. 3- Avoid cold water. 4- Insert a loose cotton earplug in the external ear.

1

Which lobe of the brain is responsible for concentration and abstract thought? 1- Frontal 2- Parietal 3- Temporal 4- Occipital

1

Which lobe of the brain is responsible for spatial relationships? 1- Parietal 2- Temporal 3- Occipital 4- Frontal

1

Which medication directly inhibits osteoclasts, thereby reducing bone loss and increasing BMD? 1- Calcitonin (Miacalcin) 2- Raloxifene (Evista) 3- Teriparatide (Forteo) 4- Vitamin D

1

Which medication taken by the client in the previous 24 hours would be of greatest concern to the nurse caring for a client undergoing a bone biopsy? 1- aspirin 2- furosemide 3- digoxin 4- NPH insulin

1

Which nerve is assessed when the nurse asks the client to spread all fingers? 1- Ulnar 2- Peroneal 3- Radial 4- Median

1

Which nursing action is most important in caring for the client following an arthrogram? 1- Apply ice to the joint. 2- Keep the joint below the level of the heart. 3- Administer morphine sulfate. 4- Assist the client with passive range of motion.

1

Which nursing intervention should be included during the assessment of a client with an eye disorder? 1- Check the extraocular muscles by instructing the client to keep his or her head still when following an object. 2- Examine the retina with a direct ophthalmoscope. 3- Use a tonometer to indent or flatten the surface of the eye. 4- Instruct the client to stare at the central fixation spot on an Amsler grid and report if he or she sees any distortion of the squares.

1

Which of the following brain structures is responsible for equilibrium? 1- Cerebellum 2- Brainstem 3- Thalamus 4- Hypothalamus

1

Which of the following inhibits bone resorption and promotes bone formation? 1- Calcitonin 2- Estrogen 3- Parathyroid hormone 4- Corticosteroids

1

Which of the following is a sympathetic nervous system effect? 1- Decreased peristalsis 2- Decreased blood pressure 3- Constricted pupils 4- Constricted bronchioles

1

Which of the following is an example of a gliding joint? 1- Carpal bones in the wrist 2- Knee 3- Hip 4- Joint at base of thumb

1

Which of the following is an involuntary rhythmic movement of the eyes that is also associated with vestibular dysfunction? 1- Nystagmus 2- Vertigo 3- Tinnitus 4- Presbycusis

1

Which of the following is the final stage of fracture repair? 1- Remodeling 2- Cartilage calcification 3- Cartilage removal 4- Angiogenesis

1

Which of the following is the main refracting surface of the eye? 1- Cornea 2- Iris 3- Pupil 4- Conjunctiva

1

Which of the following is the most common site of joint effusion? 1- Knee 2- Elbow 3- Hip 4- Shoulder

1

Which of the following is the priority nursing diagnosis for the client preparing for a bone marrow biopsy? 1- Deficient knowledge: procedure 2- Acute pain 3- Risk for infection 4- Risk for ineffective peripheral tissue perfusion

1

Which of the following occurs when there is deviation from perfect ocular alignment? 1- Strabismus 2- Ptosis 3- Chemosis 4- Nystagmus

1

Which of the following terms is used to describe rapid, jerky, involuntary, purposeless movements of the extremities? 1- Chorea 2- Bradykinesia 3- Spondylosis 4- Dyskinesia

1

Which of the following tests uses a tuning fork between two positions to assess hearing? 1- Rinne 2- Whisper 3- Watch tick 4- Weber

1

Which one of the following patients is most likely to be experiencing a health problem that may be a result of vestibular dysfunction? 1- A 78-year-old woman who requires open reduction and internal fixation of her left hip following a fall 2- An 80-year-old man whose recent declines in cognition have prompted his move to a long-term care facility 3- A woman whose progressive weakness and loss of coordination are attributable to multiple sclerosis 4- A man who has been hospitalized with signs and symptoms of malnutrition and fluid volume deficit

1

Which precautions should the nurse take when a client is at risk of injury secondary to vertigo and probable imbalance? 1- Grasp the siderails when rising to a standing position. 2- Keep his or her eyes closed. 3- Refrain from looking at one place. 4- Immobilize the head to reduce the risk of falling

1

Which statement describes benign paroxysmal positional vertigo (BPPV)? 1- The vertigo is usually accompanied by nausea and vomiting. 2- The onset of BPPV is gradual. 3- BPPV is caused by tympanic membrane rupture. 4- BPPV is stimulated by the use of certain medications, such as acetaminophen.

1

Which term describes the fibrous connective tissues that cover the brain and spinal cord? 1- Meninges 2- Dura mater 3- Arachnoid mater 4- Pia mater

1

Which term refers to a disease of a nerve root? 1- Radiculopathy 2- Involucrum 3- Sequestrum 4- Contracture

1

Which term refers to the shifting of brain tissue from an area of high pressure to an area of low pressure? 1- Herniation 2- Autoregulation 3- Cushing's response 4- Monro-Kellie hypothesis

1

Which terms refers to the progressive hearing loss associated with aging? 1- Presbycusis 2- Exostoses 3- Otalgia 4- Sensorineural hearing loss

1

While doing an initial assessment on a patient, the patient tells the nurse that she has bone pain. The nurse asks the patient to describe the characteristics of the pain. Which of the following are typical characteristics of bone pain? 1- Dull, deep ache 2- Soreness or aching that may include cramping 3- Sharp and piercing pain that is relieved by immobilization 4- Spastic or sharp pain that radiates

1

While snowboarding, a fell and sustained a blow to the head, resulting in a loss of consciousness. The client regained consciousness within an hour after arrival at the ED, was admitted for 24-hour observation, and was discharged without neurologic impairment. What would the nurse expect this client's diagnosis to be? 1- concussion 2- laceration 3- contusion 4- skull fracture

1

You are the clinic nurse in an ophthalmic clinic. When assessing clients, which client has an abnormal intraocular pressure (IOP)? 1- A client with an IOP of 8 mm Hg 2- A client with an IOP of 15 mm Hg 3- A client with an IOP of 19 mm Hg 4- A client with an IOP of 21 mm Hg

1

During an assessment using the positions test, what does an asymmetrical movement indicate? Select all that apply. 1- Weakness in the extraocular muscles 2- Dysfunction of the cranial nerve 3- Cataract 4- Glaucoma

1,2

The nurse is educating a group of people newly diagnosed with migraine headaches. What information should the nurse include in the educational session? Select all that apply. 1- Keep a food diary. 2- Maintain a headache diary. 3- Sleep no more than 5 hours at a time. 4- Exercise in a dark room. 5- Use St. John's Wort.

1,2

A nurse caring for a patient with head trauma will be monitoring the patient for Cushing's triad. What will the nurse recognize as the symptoms associated with Cushing's triad? Select all that apply. 1- Bradycardia 2- Bradypnea 3- Hypertension 4- Tachycardia

1,2,3

Cerebrospinal fluid (CSF) studies would indicate which of the following in a patient suspected of having meningitis? Select all that apply. 1- Decreased glucose 2- Increased protein 3- Increased white blood cells 4- Decreased protein 5- Increased glucose

1,2,3

Loud, persistent noise has been found to have which of the following effects on the body? Select all that apply. 1- Constriction of peripheral blood vessels 2- Increased blood pressure 3- Increased heart rate 4- Decreased gastrointestinal motility 5- Dilation of peripheral blood vessels

1,2,3

Red bone marrow produces which of the following? Select all that apply. 1- Platelets 2- White blood cells (WBCs) 3- Red blood cells (RBCs) 4- Estrogen 5- Corticosteroids

1,2,3

The nurse is performing an assessment for a patient who may have peripheral neurovascular dysfunction. What signs does the patient present with that indicate circulation is impaired? (Select all that apply.) 1- Pale, cyanotic, or mottled color 2- Cool temperature of the extremity 3- More than 3-second capillary refill 4- Tenting skin turgor 5- Limited range of motion

1,2,3

The nurse is reviewing the audiology consultation on a woman who has been experiencing progressive hearing loss. To accurately interpret these diagnostic findings, the nurse should be aware of what important characteristics of sound? Select all that apply. 1- Frequency 2- Pitch 3- Intensity 4- Character 5- Familiarity

1,2,3

Which of the following describes vertigo? Select all that apply. 1- Misperception of motion 2- Spinning sensation 3- Objects are moving around him or her 4- Syncope 5- Fainting

1,2,3

The nurse is providing care for a male patient who has undergone knee arthroplasty. As part of the nurse's morning assessment, the nurse is assessing for peripheral neurovascular dysfunction distal to the surgical site. When performing this assessment, what parameters should the nurse assess and document? Select all that apply. 1- The color of the patient's lower leg and foot 2- The patient's ability to move his foot 3- The patient's sensation in his foot and lower leg 4- The temperature of the patient's foot and lower leg 5- The presence or absence of hair on the patient's foot and lower leg

1,2,3,4

Which are risk factors for spinal cord injury (SCI)? Select all that apply. 1- Young age 2- Female gender 3- Alcohol use 4- Drug abuse 5- Caucasian ethnicity

1,3,4

A client with a spinal cord injury is to receive methylprednisolone sodium succinate 100 mg intravenously twice a day. The medication is supplied in vials containing 125 mg per 2 mL. How many mL will constitute the correct dose? Enter the correct number ONLY.

1.6

A 70-year-old patient with severe presbycusis is weighing the possibility of getting hearing aids. The nurse who is contributing to this patient's care should be aware that the patient's potential benefit of using hearing aids will depend upon which of the following factors? 1- The patient's ability to supplement the hearing aid with speech reading 2- The patient's ability to adequately discriminate sounds 3- The patient's level of motivation to hear more clearly 4- The patient's level of personal hygiene

2

A client comes to the clinic complaining of low back pain radiating down the left leg. After diagnostic studies rule out any pathology, the health care provider orders a serotonin-norepinephrine reuptake inhibitor (SNRI). Which medication does the nurse anticipate educating the client about? 1- Amitriptyline 2- Duloxetine 3- Gabapentin 4- Cyclobenzaprine

2

A client experiences a musculoskeletal injury that involves the structure that connects a muscle to the bone. The nurse understands that this injury involves which structure? 1- Ligament 2- Tendon 3- Cartilage 4- Joint

2

A client has been diagnosed with a frontal lobe brain abscess. Which nursing intervention is appropriate? 1- Assess for facial weakness. 2- Initiate seizure precautions. 3- Assess visual acuity. 4- Ensure that client takes nothing by mouth.

2

A client has been treated for migraine headaches for several months and comes to the clinic stating he is getting no better. The nurse is talking with the client and hears an audible click when the client is moving his jaw. What does the nurse suspect may be happening? 1- Trigeminal neuralgia 2- Temporomandibular disorder 3- Loose teeth 4- Dislocated jaw

2

A client in the surgical intensive care unit has skeletal tongs in place to stabilize a cervical fracture. Protocol dictates that pin care should be performed each shift. When providing pin care for the client, which finding should the nurse report to the physician? 1- Crust around the pin insertion site 2- A small amount of yellow drainage at the left pin insertion site 3- A slight reddening of the skin surrounding the insertion site 4- Pain at the insertion site

2

A client is admitted to the hospital after sustaining a closed head injury in a skiing accident. The physician ordered neurologic assessments to be performed every 2 hours. The client's neurologic assessments have been unchanged since admission, and the client is complaining of a headache. Which intervention by the nurse is best? 1- Administer codeine 30 mg by mouth as ordered and continue neurologic assessments as ordered. 2- Assess the client's neurologic status for subtle changes, administer acetaminophen, and then reassess the client in 30 minutes. 3- Reassure the client that a headache is expected and will go away without treatment. 4- Notify the physician; a headache is an early sign of worsening neurologic status.

2

A client is diagnosed with a bone tumor. What result would the nurse expect the blood tests to reveal? 1- Decreased alkaline phosphatase 2- Decreased serum calcium level 3- Decreased serum phosphorus 4- Decreased red blood cell count

2

A client is experiencing muscle weakness in the upper extremities. The client raises an arm above the head but then loses the ability to maintain the position. Muscular dystrophy is suspected. Which diagnostic test would evaluate muscle weakness or deterioration? 1- A serum calcium test 2- An electromyography 3- An arthroscopy 4- A magnetic resonance imaging (MRI)

2

A client is receiving long-term intravenous therapy with gentamicin for a chronic wound infection. Which of the following would be most important for the nurse to ensure? 1- Obtain specimens for wound cultures daily. 2- Arrange for audiograms twice a week. 3- Monitor complete blood counts every other day. 4- Assess intravenous access site daily.

2

A client is waiting in a triage area to learn the medical status of family members following a motor vehicle accident. The client is pacing, taking deep breaths, and handwringing. Considering the effects in the body systems, the nurse anticipates that the liver will: 1- cease function and shunt blood to the heart and lungs. 2- convert glycogen to glucose for immediate use. 3- produce a toxic byproduct in relation to stress. 4- maintain a basal rate of functioning.

2

A client undergoes an invasive joint examination of the knee. What will the nurse closely monitor the client for? 1- Lack of sleep and appetite 2- Serous drainage 3- Signs of depression 4- Signs of shock

2

A client visits the health care provider for a routine checkup. The history reveals that the client was diagnosed with a spinal curvature. Which region of the spine should the nurse assess for complications? 1- Cervical 2- Thoracic 3- Lumbar 4- Sacral

2

A client who has been fitted with a hearing aid comes for a follow-up evaluation. During the visit, the client states, "I've noticed that I still don't seem to hear well enough. The hearing aid doesn't seem to make the sounds louder." Which of the following might the nurse determine as the possible cause? 1- Ear mold is loose. 2- Client has cerumen in the ear. 3- The mold is not properly fitted. 4- The client has an ear infection.

2

A client who is a competitive swimmer is diagnosed with swimmer's ear (otitis externa). Which instruction would be least appropriate for the nurse to include when teaching the client about this condition? 1- Wear ear plugs when swimming, showering, or washing the hair. 2- Use a hair dryer set on high to dry the ear canal after swimming. 3- Instill alcohol drops into the external ear after swimming. 4- Avoid scratching or exposing the ear to scratching or trauma.

2

A client with Meniere's disease has a nursing diagnosis of risk for injury related to gait disturbances and vertigo. Which of the following would be most appropriate to include in this client's plan of care? 1- Moving the head from side-to-side when vertigo occurs 2- Sitting down at the first sign of feeling dizzy 3- Closing the eyes when lying down during an episode of vertigo 4- Performing self-care activities when the vertigo first starts

2

A client with a traumatic brain injury has developed increased intracranial pressure resulting in diabetes insipidus. The expected treatment would consist of which of the following? 1- Fluid restriction 2- Vasopressin therapy 3- Hypertonic saline solution 4- Diet containing extra sodium

2

A client with otosclerosis comes to the clinic for a follow-up. When assessing this client's hearing, which type of hearing loss would the nurse expect to find? 1- sensorineural 2- conductive 3- no hearing loss 4- loss of high frequency sounds

2

A client you are caring for has a hearing loss. The client tells you he is self-conscious about his hearing loss. What advice should the nurse give a self-conscious client with hearing loss to protect his self-esteem? 1- Pretend to follow conversations by nodding the head. 2- Be forthright and inform others about the hearing deficit. 3- Follow lip movements closely. 4- Avoid excess socializing.

2

A critical care nurse is documenting her assessment of a client she is caring for. The client is status post-resection of a brain tumor. The nurse documents that the client is flaccid on the left. What does this mean? 1- The client has an abnormal posture response to stimuli. 2- The client is not responding to stimuli. 3- The client is hyperresponsive on the left. 4- The client is hyporesponsive on the left.

2

A fall during a rock climbing expedition this morning has caused a 28-year-old woman to develop an epidural hematoma. Immediate treatment is being organized by the emergency department team because this woman faces a risk of serious neurological damage as a result of: 1- Decreased intravascular volume 2- Increased intracranial pressure (ICP) 3- Ischemic cerebrovascular accident (CVA) 4- Brain tissue necrosis

2

A female patient with a profound visual deficit is unable to read the largest E on the Snellen chart. How should the nurse proceed with assessment of the patient's visual acuity? 1- Document the fact that the patient does not have functional vision. 2- Hold up a random number of fingers in front of the patient's eyes and ask her to count them. 3- Give the patient a hand-held version of the Snellen chart and have her hold it at the furthest distance at which she can read it. 4- Ask the patient to identify the contents of a picture in a book or magazine.

2

A high school soccer player sustained five concussions before she was told that she should never play contact sports again. After her last injury, she began experiencing episodes of double vision. She was told that she had most likely incurred damage to which cranial nerve? 1- V (Trigeminal) 2- VI (Abducens) 3- VII (Facial) 4- IV (Trochlear)

2

A middle-aged woman has scheduled an appointment with her nurse practitioner because she has been experiencing intractable muscle weakness in recent weeks. Which of the following characteristics of the patient's weakness should cause the nurse to suspect a neurological etiology? 1- The patient's weakness is most severe in the early morning. 2- The weakness is primarily on the left side of the patient's body. 3- The weakness is not relieved by increasing her food intake. 4- The patient's weakness began around the time of her husband's death.

2

A mother brings her daughter to the clinic for an evaluation because the child is complaining of ear pain. Which of the following would lead the nurse to suspect that the child is experiencing otitis externa and not otitis media? 1- Fever 2- Aural tenderness 3- Bulging eardrum 4- Ear drainage

2

A nures is performing an otoscopic examination on a client. Which finding would the nurse document as abnormal? 1- Umbo in the center of the tympanic membrane 2- External auditory canal erythema 3- Tympanic membrane pearly gray 4- Manubrium superior to the umbo

2

A nurse conducted a history and physical for a newly admitted patient who states, "My arms are too short. I have to hold my book at a distance to read." The nurse knows that the patient is most likely experiencing: 1- Opacity in the lens. 2- Loss of accommodative power in the lens. 3- Shrinkage of the vitreous body. 4- Decreased eye muscle tone.

2

A nurse is caring for a client with deteriorating neurologic status. The nurse is performing an assessment at the beginning of the shift that reveals a falling blood pressure and heart rate, and the client makes no motor response to stimuli. Which documentation of neuromuscular status is most appropriate? 1- Abnormal posture 2- Flaccidity 3- Weak muscular tone 4- Decorticate posturing

2

A nurse is planning discharge teaching regarding exercise for a client at risk for osteoporosis. Which of the following exercises would be appropriate? 1- Yoga 2- Walking 3- Bicycling 4- Swimming

2

A nurse is taking a newly admitted patient's health history, and the patient states that she has had ongoing problems with a "pinched nerve." In addition to the potential for pain, the nurse should consider the fact that physical pressure that is placed on nerves can often lead to what? 1- Crepitus 2- Paresthesias 3- Muscle hypertrophy 4- Osteoporosis

2

A nurse is working in a neurologist's office. The physician orders a Romberg test. The nurse should have the client: 1- touch his or her nose with one finger. 2- close his or her eyes and stand erect. 3- close his or her eyes and discriminate between dull and sharp. 4- close his or her eyes and jump on one foot.

2

A nurse notices a client lying on the floor at the bottom of the stairs. The client is alert and oriented and denies pain other than in the arm, which is swollen and appears deformed. After calling for help, what should the nurse do? 1- Place the client in a sitting position. 2- Immobilize the client's arm. 3- Help the client walk to the nearest nurses' station. 4- Raise the client's arm above the heart.

2

A nurse working on a medical-surgical floor walks into a patient's room to find the patient with an altered level of consciousness (LOC). Which of the following actions would be the first priority? 1- Positioning to prevent complications 2- Maintenance of a patent airway 3- Assessment of pupillary light reflexes 4- Determination of the cause

2

A nursing educator is talking with nurses about the effects of the aging process and neurologic changes. What would the educator identify as a normal neurological change that accompanies the aging process? 1- Hyperactive deep tendon reflexes 2- Reduction in cerebral blood flow (CBF) 3- Increased cerebral metabolism 4- Hypersensitivity to painful stimuli

2

A nursing instructor is preparing a class on air and bone conduction of sound. When describing the transmission of sounds by air conduction, which of the following would the instructor include? 1- Vibrations bypass the tympanic membrane to reach the inner ear. 2- Sounds travel through the external and middle ears. 3- It is a less efficient pathway for sound. 4- Vibration of the tympanic membrane diverts sound away from the ossicles.

2

A patient has been brought to the emergency department (ED) with signs and symptoms of a stroke and a stat computed tomography (CT) head scan has been ordered. The ED nurse should know that the image that results from CT indicates distinguishing differences based on which of the following variables? 1- Proximity to the CT scanner 2- Variations in tissue density 3- Metabolic activity 4- Oxygen consumption

2

A patient has been referred to a specialist by his primary care provider because he is suspected to have an acoustic neuroma. What signs and symptoms most likely prompted the care provider's referral? 1- A history of clear discharge from one of the patient's ears 2- Unilateral tinnitus and hearing loss 3- Vertigo that is unresponsive to medication 4- Chronic tympanic perforation

2

A patient has come to the office for her first eye examination and is diagnosed as having a visual acuity of 20/40. The patient asks the nurse what these numbers mean. What is a correct response by the nurse? 1- "A person whose vision is 20/40 can see an object from 40 feet away that a person with 20/20 vision can see from 20 feet away." 2- "A person whose vision is 20/40 can see an object from 20 feet away that a person with 20/20 vision can see from 40 feet away." 3- "A person whose vision is 20/40 can see an object from 40 inches away that a person with 20/20 vision can see from 20 inches away." 4- "A person whose vision is 20/40 can see an object from 20 inches away that a person with 20/20 vision can see from 40 inches away."

2

A patient is brought to the trauma center by ambulance after sustaining a high cervical spinal cord injury 1½ hours ago. What medication does the nurse know will be given to prevent further spinal cord damage? 1- Furosemide (Lasix) 2- Methylprednisolone (Solu-Medrol) 3- Cyclobenzaprine (Flexeril) 4- Hydralazine hydrochloride (Apresoline)

2

A patient is scheduled to have an electronystagmography as part of a diagnostic workup for Ménière's disease. What question is it most important for the nurse to ask the patient in preparation for this test? 1- "Have you ever experienced claustrophobia or feelings of anxiety while in enclosed spaces?" 2- "Do you currently take sedatives, tranquilizers, or antihistamines on a regular basis?" 3- "Do you have a history of falls or problems with loss of balance?" 4- "Do you have a history of either high or low blood pressure?"

2

A patient with a neurological disorder is being assessed by the nurse. The nurse assesses the patient's biceps reflex as diminished. The nurse would be correct in documenting this response as what? 1- 0 2- 1+ 3- 2+ 4- 3+

2

A patient with a primary diagnosis of colorectal cancer has developed bone metastases in recent weeks. When reviewing this patient's daily blood work, the nurse should be aware that secondary bone tumors create a significant risk of what electrolyte imbalance? 1- Hypokalemia 2- Hypercalcemia 3- Hyponatremia 4- Hyperchloremia

2

A patient with amyotrophic lateral sclerosis (ALS) asks if the nurse has heard of a drug that will prolong the patient's life. The nurse knows that there is a medication that may prolong the life by 3 to 6 months. To which medication is the patient referring? 1- Baclofen 2- Riluzole 3- Dantrolene sodium 4- Diazepam

2

A patient with spinal cord injury has a nursing diagnosis of altered mobility. Which of the following would be included as an appropriate nursing intervention to prevent deep vein thrombosis (DVT) from occurring? 1- Placing the patient on a fluid restriction 2- Applying thigh-high elastic stockings 3- Administering an antifibrinolytic agent 4- Assisting the patient with passive range of motion exercises

2

After thorough diagnosis and screening, a 9-year-old boy has been scheduled for a cochlear implant. The nurse who will be contributing to this child's care should recognize that: 1- The boy's hearing deficit will be considered normal within 24 to 48 hours after surgery. 2- A lengthy period of learning may be required before he can accurately interpret sounds. 3- The boy will achieve hearing through bone conduction. 4- The cochlear implant will provide a beneficial, but temporary, solution to the boy's hearing loss.

2

As part of an admission assessment, the nurse is including questions that pertain to the patient's vision. Before conducting a physical assessment, the nurse is seeking to identify factors that have the potential to influence the patient's vision. Which of the following assessment questions most directly addresses these factors? 1- "Are you a smoker?" 2- "What medications do you take?" 3- "Do you exercise regularly?" 4- "What is your occupation?"

2

Cerebral edema peaks at which time point after intracranial surgery? 1- 12 hours 2- 24 hours 3- 48 hours 4- 72 hours

2

During a routine physical examination on an older female client, a nurse notes that the client is 5 feet, 3/8 inches (1.6 m) tall. The client states, "How is that possible? I was always 5 feet and 1/2? (1.7 m) tall." Which statement is the best response by the nurse? 1- "After age 40, height may show a gradual decrease as a result of spinal compression" 2- "After menopause, the body's bone density declines, resulting in a gradual loss of height." 3- "There may be some slight discrepancy between the measuring tools used." 4- "The posture begins to stoop after middle age."

2

Episodes of orthostatic hypotension occur in the first 2 weeks after a spinal cord injury. Compare the two blood pressure measurement for each answer. The blood pressure reading obtained when the patient was sitting, is in the left column for comparison. Which of the following shows the blood pressure measurement indicative of orthostatic hypotension? 1- 140/110 130/110 2- 140/100 120/90 3- 130/90 125/85 4- 130/80 120/80

2

High doses of which medication can produce bilateral tinnitus? 1- Meclizine 2- Aspirin 3- Promethazine 4- Dimenhydrinate

2

Loud, persistent noise has what effect on the body? 1- Dilation of peripheral blood vessels 2- Increased blood pressure 3- Decreased heart rate 4- Decreased gastrointestinal activity

2

Lower motor neuron lesions cause 1- increased muscle tone. 2- flaccid muscles. 3- no muscle atrophy. 4- hyperactive and abnormal reflexes.

2

Pressure ulcers may begin within hours of an acute spinal cord injury (SCI) and may cause delay of rehabilitation, adding to the cost of hospitalization. The most effective approach is prevention. Which of the following nursing interventions will most protect the client against pressure ulcers? 1- Continuous use of an indwelling catheter 2- Meticulous cleanliness 3- Avoidance of all lotions and lubricants 4- Allowing the client to choose the position of comfort

2

The Glasgow Coma Scale is a common screening tool used for patients with a head injury. During the physical exam, the nurse documents that the patient is able to spontaneously open her eyes, obey verbal commands, and is oriented. The nurse records the highest score of: 1- 20 2- 15 3- 10 4- 5

2

The client is having a Weber test. During a Weber test, where should the tuning fork be placed? 1- On the mastoid process behind the ear 2- In the midline of the client's skull or in the center of the forehead 3- Near the external meatus of each ear 4- Under the bridge of the nose

2

The diagnosis of multiple sclerosis is based on which test? 1- CSF electrophoresis 2- Magnetic resonance imaging 3- Evoked potential studies 4- Neuropsychological testing

2

The nures is assessing a client using an Amsler Grid. The nurse is assessing for which of the following? 1- Visual acuity 2- Macular problems 3- Intraocular pressure 4- Visual field

2

The nurse identifies the nursing diagnosis of deficient knowledge related to a new hearing aid for a client. After teaching a client about caring for his new hearing aid, the nurse determines that the outcome has been achieved when the client states which of the following? 1- "I need to wipe the ear mold daily with a moist washcloth." 2- "I need to keep my ear canal clean and dry." 3- "I should wash the receiver with soap and water once a week." 4- "I should insert the ear mold when it is wet."

2

The nurse in the neurologic ICU is caring for a client who sustained a severe brain injury. Which nursing measures will the nurse implement to help control intracranial pressure (ICP)? 1- Position the client in the supine position 2- Maintain cerebral perfusion pressure from 50 to 70 mm Hg 3- Restrain the client, as indicated 4- Administer enemas, as needed

2

The nurse is assessing a client's hearing using the Rinne test. When providing instruction to elicit client feedback, which instruction is essential? 1- Raise your hand when you hear the vibration. 2- Raise your hand when you no longer hear sound. 3- Raise your hand when the vibration exceeds the sound. 4- Raise your hand when the sound exceeds the vibration.

2

The nurse is assessing the pupils of a patient who has had a head injury. What does the nurse recognize as a parasympathetic effect? 1- Dilated pupils 2- Constricted pupils 3- One pupil is dilated and the opposite pupil is normal 4- Roth's spots

2

The nurse is assisting the client in planning care during exacerbations of Ménière's disease. Which diet would the nurse identify as appropriate at this time? 1- A high-protein diet 2- A low-sodium diet 3- A low-fat diet 4- A calorie-controlled diet

2

The nurse is caring for a patient who was involved in a motor vehicle accident and sustained a head injury. When assessing deep tendon reflexes (DTR), the nurse observes diminished or hypoactive reflexes. How will the nurse document this finding? 1- 0 2- 1+ 3- 2+ 4- 3+

2

The nurse is conducting a musculoskeletal assessment of a client in a nursing home. The client is unable to dorsiflex the right foot or extend the toes. The nurse evaluates this finding as an injury to which nerve? 1- Sciatic 2- Peroneal 3- Femoral 4- Achilles

2

The nurse is doing an initial assessment on a patient recently admitted to the unit with a diagnosis of cerebrovascular accident (CVA). The patient has difficulty copying a figure that the nurse has drawn. The nurse uses this technique to assess for what type of aphasia? 1- Auditory-receptive 2- Visual-receptive 3- Expressive speaking 4- Expressive writing

2

The nurse is educating a client with low back pain on proper lifting techniques. The nurse recognizes that the education was effective when the client 1- reaches over the head with the arms fully extended. 2- places the load close to the body. 3- uses a narrow base of support. 4- bends at the hips and tightens the abdominal muscles.

2

The nurse is explaining metastatic rhabdomyosarcoma to a group of parents with children diagnosed with the disease. The most common site of metastasis in clients diagnosed with rhabdomyosarcoma is the 1- brain. 2- lung. 3- bone. 4- lymph nodes.

2

The nurse is performing a detailed mental status assessment of an older adult patient who has a diagnosis of mild Alzheimer's disease. What assessment most accurately gauges the patient's abstract reasoning? 1- "What city and state are we in right now?" 2- "What would you do if you found a stamped envelope on the street?" 3- "If you divide 16 by four and then double it, what do you get?" 4- "How do you believe that Alzheimer's disease is affecting you?"

2

The nurse is performing a neurologic assessment on a client diagnosed with a stroke and cannot elicit a gag reflex. This deficit is related to which of the following cranial nerves? 1- VIII 2- X 3- III 4- VII

2

The nurse is reviewing the results of a client's audiometric testing. The nurse determines that the client has a mild hearing loss based on the result signifying which loss of decibels? 1- 20 2- 30 3- 45 4- 80

2

The nurse is teaching a class on diseases of the ear. What would the nurse teach the class is the most characteristic symptom of otosclerosis? 1- The client being distressed in the mornings 2- A progressive, bilateral loss of hearing 3- A red and swollen ear drum 4- The client describing a recent upper respiratory infection

2

The nurse obtains a Snellen eye chart when assessing cranial nerve function. Which cranial nerve is the nurse testing when using the chart? 1- CN I 2- CN II 3- CN III 4- CN IV

2

The nurse on a cruise ship is assessing clients for motion sickness. Which of the following is a common misconception? 1- Repeated motion is the cause. 2- Once symptoms occur, they will always be present. 3- Medications help the symptoms. 4- Pallor and diaphoresis is a first symptom.

2

The nurse teaches the client with a high risk for osteoporosis about risk-lowering strategies, including which action? 1- Increase fiber in the diet 2- Walk or perform weight-bearing exercises outdoors 3- Reduce stress 4- Decrease the intake of vitamin A and D

2

The nursing instructor is teaching about hematomas to a pre-nursing pathophysiology class. What would the nursing instructor describe as an arterial bleed with rapid neurologic deterioration? 1- Extradural hematoma 2- Epidural hematoma 3- Subdural hematoma 4- Intracranial hematoma

2

The staff educator is orientating a nurse new to the neurological ICU when a patient with a T2 spinal cord injury is admitted. The patient is soon exhibiting manifestations of neurogenic shock. What sign or symptom is consistent with this diagnosis? 1- Increased cardiac biomarkers 2- Hypotension 3- Tachycardia 4- Excessive sweating

2

There are 12 pairs of cranial nerves. Only three are sensory. Select the cranial nerve that is affected with decreased visual fields. 1- Cranial nerve I 2- Cranial nerve II 3- Cranial nerve III 4- Cranial nerve IV

2

There are several types of hearing loss. Which type of hearing loss benefits most from the use of a hearing aid? 1- Sensorineural 2- Conductive 3- Genetic 4- Acquired

2

Three hours after injuring the spinal cord at the C6 level, a client receives high doses of methylprednisolone sodium succinate (Solu-Medrol) to suppress breakdown of the neurologic tissue membrane at the injury site. To help prevent adverse effects of this drug, the nurse expects the physician to order: 1- naloxone (Narcan). 2- famotidine (Pepcid). 3- nitroglycerin (Nitro-Bid). 4- atracurium (Tracrium).

2

To avoid the side effects of corticosteroids, which medication classification is used as an alternative in treating inflammatory conditions of the eyes? 1- Miotics 2- Nonsteroidal anti-inflammatory drugs (NSAIDs) 3- Mydriatics 4- Cycloplegics

2

What inner ear structure is affected when a patient is diagnosed with otosclerosis? 1- Malleus 2- Stapes 3- Incus 4- Tympanic membrane

2

What is the term for a rhythmic contraction of a muscle? 1- Atrophy 2- Clonus 3- Hypertrophy 4- Crepitus

2

Which body movement involves moving toward the midline? 1- Pronation 2- Adduction 3- Abduction 4- Eversion

2

Which cerebral lobes is the largest and controls abstract thought? 1- Temporal 2- Frontal 3- Parietal 4- Occipital

2

Which diagnostic test distinguishes between conductive and sensorineural hearing loss? 1- Whisper test 2- Rinne test 3- Audiometry 4- Weber test

2

Which is a correct rationale for encouraging a client with otitis externa to eat soft foods? 1- Chewy foods, such as red meat, may react with prescribed analgesics and antibiotics. 2- Chewing may cause discomfort. 3- Chewing may lead to further complications, such as otitis media. 4- Chewing may cause excessive drainage.

2

Which is the most common cause of visual loss in people older than 65 years of age? 1- Glaucoma 2- Macular degeneration 3- Cataracts 4- Retinal detachment

2

Which medication classification is prescribed when allergy is a factor causing the skin disorder? 1- Corticosteroids 2- Antihistamines 3- Antibiotics 4- Local anesthetics

2

Which nursing goal is a priority when caring for a client newly diagnosed with vertigo? 1- Client will maintain therapeutic medication schedule. 2- Client will remain safe while ambulating in the home. 3- Client will have a caretaker with him or her in the home. 4- Client will close eyes as needed to reduce symptoms.

2

Which nursing suggestion would be most helpful to the client with recurrent otitis externa? 1- Use a cotton applicator to ensure that the ear canal is dry. 2- Place ear plugs into the ears before swimming 3- Flush the ear with hydrogen peroxide 4- Avoid lying on the side of the affected ear

2

Which precaution should the nurse take when a client is at risk of injury secondary to vertigo and probable imbalance? 1- Recommend that the client keep his or her eyes closed 2- Have the client wait for help before moving 3- Restrict the client from focusing on one spot 4- Encourage the client to move the head slowly

2

Which surgical procedure involves flattening the anterior curvature of the cornea by removing a stromal lamella layer? 1- Photorefractive keratectomy (PRK) 2- Laser-assisted stromal in situ keratomileusis (LASIK) 3- Keratoconus 4- Keratoplasty

2

Which term refers to mature compact bone structures that form concentric rings of bone matrix? 1- Endosteum 2- Lamellae 3- Trabecula 4- Cancellous bone

2

Which term refers to muscular hypertonicity in a weak muscle, with increased resistance to stretch? 1- Akathisia 2- Spasticity 3- Ataxia 4- Myoclonus

2

Which term refers to the inability to coordinate muscle movements, resulting difficulty walking? 1- Agnosia 2- Ataxia 3- Spasticity 4- Rigidity

2

Which term refers to the inability to coordinate muscle movements, resulting in difficulty walking? 1- Agnosia 2- Ataxia 3- Spasticity 4- Rigidity

2

You are teaching a class on diseases of the ear. What would you teach the class is the most characteristic symptom of otosclerosis? 1- The client being distressed in the mornings 2- A progressive, bilateral loss of hearing 3- A red and swollen ear drum 4- The client describing a history of having had a recent upper respiratory infection

2

Age-related changes in the neurologic system must be carefully assessed. Which of the following changes does the nurse expect to find in some degree depending on the patient's age and medical condition? Select all that apply. 1- Hyper-reactive deep tendon reflexes 2- Decreased muscle mass 3- Increased sensitivity to heat and cold 4- Stage IV sleep is prolonged 5- Increased sensitivity of taste buds 6- Reduced papillary responses

2,3,6

A nurse practitioner examines a patient and documents a best corrected visual acuity (BCVA) ratio in his better eye that qualifies him for government financial assistance based on the definition of legal blindness. What is that ratio? 20/160 20/120 20/140 20/200

20/200 Legal blindness is a condition of impaired vision in which a person has a BCVA that does not exceed 20/200 in the better eye or whose widest visual field diameter is 20 degrees or less.

A nurse is reviewing the medical records of several clients who have come to the clinic. Each of the clients wears corrective lenses. For the client with which corrected visual acuity would the nurse need to include additional devices and strategies to address low vision? 20/40 20/60 20/80 20/20

20/80 Low vision is defined as a best corrected visual acuity (BCVA) of 20/70 to 20/200. Low vision is a general term describing visual impairment that requires clients to use devices and strategies in addition to corrective lenses to perform visual tasks.

A 10-year-old boy who was brought to the emergency room after a skiing accident is diagnosed with a fracture of the distal end of the femur. Why is this type of fracture significant? 1- Osteoblast formation will stop during the time needed for fracture healing. 2- Red blood cell production will be temporarily reduced because of the damage to the medullar cavity. 3- Potential growth problems may result from damage to the epiphyseal plate. 4- Periosteal blood vessels will be damaged, thus compromising blood flow to the compact bone.

3

A 22-year-old man is being closely monitored in the neurological ICU after suffering a basal skull fracture during an assault. The nurse's hourly assessment reveals the presence of a new blood stain on the patient's pillow that is surrounded by a stain that is pale yellow in color. The nurse should follow up this finding promptly because it is suggestive of: 1- Increasing intracranial pressure (ICP) 2- An epidural hematoma 3- Leakage of cerebrospinal fluid (CSF) 4- Meningitis

3

A 30-year-old primiparous woman has been admitted in early labor. The obstetrical nurse has read on the patient's prenatal record that she has a history of seizures. The nurse should understand that seizures most often occur as a result of: 1- Benign spinal cord lesions 2- Cranial nerve deficits 3- Abnormal activity in the cerebral cortex 4- Faulty integration of sensory impulses by the thalamus

3

A 48-year-old woman has been diagnosed with Ménière's disease after several serious bouts of vertigo. The nurse is reviewing nutritional guidelines relevant to the management of the disease and should recommend which of the following? 1- A low-carbohydrate, high-protein diet 2- Eating several small meals each day rather than three larger meals 3- Limiting the amount of salt and sugar in the diet 4- A gluten-free diet emphasizing leafy green vegetables

3

A 56-year-old patient has come to the clinic for his routine eye examination and is told he needs bifocals. The patient asks the nurse what in his eye has changed so that he needs bifocals. What should the nurse respond? 1- "You know, you are getting older now, and we change as we get older." 2- "The parts of our eyes age, just like the rest of us." 3- "There is a gradual thickening of the lens of the eye, and it can limit accommodation of the eye." 4- "The eye gets shorter, back to front, as we age, and it changes how we see things."

3

A client comes to the eye clinic for a routine check-up. The client tells the nurse he thinks he is color blind. What screening test does the nurse know will be performed on this client to assess for color blindness? 1- Rosenbaum 2- Jaeger 3- Ishihara 4- Snellen

3

A client diagnosed with carpal tunnel syndrome (CTS) asks the nurse about numbness in the fingers and pain in the wrist. What is the best response by the nurse? 1- "CTS is a neuropathy that is characterized by bursitis and tendinitis." 2- "CTS is a neuropathy that is characterized by flexion contracture of the fourth and fifth fingers." 3- "CTS is a neuropathy that is characterized by compression of the median nerve at the wrist." 4- "CTS is a neuropathy that is characterized by pannus formation in the shoulder."

3

A client has been referred to an ophthalmologist for suspected macular degeneration. The nurse knows to prepare what test for the physician to give the client? 1- Ishihara polychromatic plates 2- Visual field 3- Amsler grid 4- Slit lamp

3

A client has undergone a mastoidectomy. The nurse teaches the client about possible complications, instructing the client to notify the physician immediately if which of the following occur? 1- Aural fullnesss 2- Serosanguinous drainage 3- Slurred speech 4- Intermittent sharp, shooting pain

3

A client in the emergency department has a suspected neurologic disorder. To assess gait, the nurse asks the client to take a few steps; with each step, the client's feet make a half circle. To document the client's gait, the nurse should use which term? 1- Ataxic 2- Dystrophic 3- Helicopod 4- Steppage

3

A client is diagnosed with otitis externa. Which instruction is most appropriate for the nurse to give? 1- Keep the ear covered with a scarf. 2- Avoid using cold water when shampooing hair. 3- Avoid swimming for 7 to 10 days. 4- Keep a dry cotton ball in the ear canal.

3

A client is recovering from a fractured hip. What would the nurse suggest that the client increase intake of to facilitate calcium absorption from food and supplements? 1- Amino acids 2- Vitamin B6 3- Vitamin D 4- Dairy products

3

A client preparing to undergo a lumbar puncture states he doesn't think he will be able to get comfortable with his knees drawn up to his abdomen and his chin touching his chest. He asks if he can lie on his left side. Which statement is the best response by the nurse? 1- "Lying on your left side will be fine during the procedure." 2- "There's no other option but to assume the knee-chest position." 3- "Although the required position may not be comfortable, it will make the procedure safer and easier to perform." 4- "I'll report your concerns to the physician."

3

A client who was found unconscious at home is brought to the hospital by a rescue squad. In the intensive care unit, the nurse checks the client's oculocephalic (doll's eye) response by: 1- introducing ice water into the external auditory canal. 2- touching the cornea with a wisp of cotton. 3- turning the client's head suddenly while holding the eyelids open. 4- shining a bright light into the pupil.

3

A client with a concussion is discharged after the assessment. Which instruction should the nurse give the client's family? 1- Have the client avoid physical exertion 2- Emphasize complete bed rest 3- Look for signs of increased intracranial pressure 4- Look for a halo sign

3

A client with neurologic infection develops cerebral edema from syndrome of inappropriate antidiuretic hormone (SIADH). Which is an important nursing action for this client? 1- Maintaining adequate hydration 2- Administering prescribed antipyretics 3- Restricting fluid intake and hydration 4- Hyperoxygenation before and after tracheal suctioning

3

A nurse has assessed a patient's orientation during the initial head-to-toe assessment near the beginning of a shift. The patient is able to state his own full name and knows the name of the hospital but is unable to identify the month and the year. How should the nurse best document this assessment finding? 1- "Patient is demonstrating signs of decreased neurological function." 2- "Patient is disoriented." 3- "Patient is oriented to person and place, but unable to state month and year." 4- "Patient is oriented to person, oriented to place, but not oriented to time."

3

A nurse is assessing reflexes in a patient with hyperactive reflexes. When the patient's foot is abruptly dorsiflexed, it continues to "beat" two to three times before setting into a resting position. How would the nurse document this finding? 1- Rigidity 2- Flaccidity 3- Clonus 4- Ataxia

3

A nurse is caring for a client with an undiagnosed bone disease. When instructing on the normal process to maintain bone tissue, which process transforms osteoblasts into mature bone cells? 1- Remodeling 2- Resorption 3- Ossification and calcification 4- Epiphyses and diaphysis formation

3

A nurse is preparing to perform the whisper test to assess a client's gross auditory acuity. Which of the following would be most appropriate for the nurse to do? 1- Stand at a position diagonal to the client. 2- Have the client use a finger to occlude the ear to be tested. 3- Stand about 1 to 2 feet away from the ear to be tested. 4- Speak a phrase in a low normal tone of voice.

3

A nurse is providing education about migraine headaches to a community group. The cause of migraines has not been clearly demonstrated, but is related to vascular disturbances. A member of the group asks about familial tendencies. The nurse's correct reply will be which of the following? 1- "There is a very weak familial tendency." 2- "No familial tendency has been demonstrated." 3- "There is a strong familial tendency." 4- "Only secondary migraine headaches show a familial tendency."

3

A nurse is teaching a client about preventing osteoporosis. Which teaching point is correct? 1- Obtaining an X-ray of the bones every 3 years is recommended to detect bone loss. 2- To prevent fractures, the client should avoid strenuous exercise. 3- The recommended daily allowance of calcium may be found in a wide variety of foods. 4- Obtaining the recommended daily allowance of calcium requires taking a calcium supplement.

3

A nurse is working on a neurological unit with a nursing student who asks the difference between primary and secondary headaches. The nurse's correct response will include which of the following statements? 1- "A secondary headache is one for which no organic cause can be identified." 2- "A secondary headache is located in the frontal area." 3- "A secondary headache is associated with an organic cause, such as a brain tumor." 4- "A migraine headache is an example of a secondary headache."

3

A nurse who coordinates care in a long-term care facility is aware that almost 100% of the residents wear glasses and that many of them require surgical corrective procedures. Which of the following phenomena is considered to be an age-related change to vision? 1- Beginning at around age 40, rods gradually replace cones. 2- The optic nerve (CN II) becomes progressively demyelinated with age. 3- The lens of the eye becomes more opaque with age. 4- Intraocular pressure gradually increases, peaking around age 75.

3

A patient comes to the clinic with a suspected eye infection. The nurse recognizes that the patient most likely has conjunctivitis, as evidenced by what symptom? 1- Blurred vision 2- Elevated IOP 3- A mucopurulent ocular discharge 4- Severe pain

3

A patient comes to the emergency department with severe pain in the face that was stimulated by brushing the teeth. What cranial nerve does the nurse understand can cause this type of pain? 1- III 2- IV 3- V 4- VI

3

A patient has been diagnosed with a deficiency of the major neurotransmitter acetylcholine. Based on this information, the nurse knows to assess the patient for complications associated with: 1- Fine movements. 2- Sleep patterns. 3- Heart rate and rhythm. 4- Emotional balance.

3

A patient has been diagnosed with a loss of hearing related to damage of the end organ for hearing. What term is used to describe this condition? 1- Exostoses 2- Otalgia 3- Sensorineural hearing loss 4- Presbycusis

3

A patient is scheduled to have an auditory brain stem response in 2 days. What does the nurse instruct the patient to do in preparation for the test? 1- Shave several areas on the scalp where the electrodes will be placed. 2- Do not eat or drink 8 hours prior to testing. 3- Wash and rinse hair before test but do not apply any other hair products. 4- Omit daily medications prior to testing.

3

A patient presents at the clinic complaining of back pain that goes all the way down the back of her leg to her foot. What is this type of pain referred to as? 1- Bursitis 2- Radiculopathy 3- Sciatica 4- Tendonitis

3

A patient sustained a head trauma in a diving accident and has a cerebral hemorrhage located within the brain. What type of hematoma is this classified as? 1- An epidural hematoma 2- An extradural hematoma 3- An intracerebral hematoma 4- A subdural hematoma

3

A patient tells the nurse, "I was working out and lifting weights and now that I have stopped, I am flabby and my muscles have gone!" What is the best response by the nurse? 1- "While you are lifting weights, endorphins are released, creating increase in muscle mass, but if the muscles are not used they will atrophy." 2- "The muscle mass has decreased from the lack of calcium in the cells." 3- "Your muscles were in a state of hypertrophy from the weight lifting but it will persist only if the exercise is continued." 4- "Once you stop exercising, the contraction of the muscle does not regain its strength."

3

A patient with multiple sclerosis is admitted to the unit. When the nurse assesses the patient, he notes that the patient is demonstrating involuntary rhythmic eye movements. What term will the nurse use when documenting these involuntary oscillations of the eye? 1- Vertigo 2- Tinnitus 3- Nystagmus 4- Dizziness

3

A pediatric nurse is caring for a child who has been brought to the clinic with otitis externa. What assessment finding is characteristic of otitis externa? 1- Tophi on the pinna and ear lobe 2- Dark yellow cerumen in the external auditory canal 3- Pain on manipulation of the auricle 4- Air bubbles visible in the middle ear

3

A primary nursing assessment for a patient who has sustained a fracture involving the basilar skull is inspection for: 1- Leakage of CSF from the nose. 2- Ecchymosis of the mastoid process of the temporal bone. 3- Leakage of CSF from the ear. 4- Vomiting and headaches due to increased intracranial pressure.

3

An older adult patient has been admitted to a subacute geriatric medicine unit with a diagnosis of failure to thrive. The nurse is conducting a comprehensive assessment that focuses on the patient's musculoskeletal system. During this assessment, what will be the nurse's primary focus? 1- Risk factors for common diseases of the musculoskeletal system 2- The presence or absence of musculoskeletal diagnoses 3- The patient's level of function and activities of daily living 4- The integrity of the patient's integumentary system

3

An osmotic diuretic such as mannitol is given to the client with increased intracranial pressure (ICP) to 1- control fever. 2- control shivering. 3- dehydrate the brain and reduce cerebral edema. 4- reduce cellular metabolic demand.

3

During a Tensilon test to determine if a patient has myasthenia gravis, the patient complains of cramping and becomes diaphoretic. Vital signs are BP 130/78, HR 42, and respiration 18. What intervention should the nurse prepare to do? 1- Place the patient in the supine position. 2- Administer diphenhydramine (Benadryl) for the allergic reaction. 3- Administer atropine to control the side effects of edrophonium. 4- Call the rapid response team because the patient is preparing to arrest.

3

During a routine eye examination, a patient complains that she is unable to read road signs at a distance when driving her car. What should the patient be assessed for? 1- Astigmatism 2- Anisometropia 3- Myopia 4- Presbyopia

3

Health teaching for a patient who suffers from motion sickness would include recommending the use of which one of the following over-the-counter drugs? 1- Scopolamine 2- Phenergan 3- Dramamine 4- Ephedrine

3

Nursing students are reviewing the structure and function of the ears in preparation for class the next day. The students demonstrate understanding of the information when they describe which of the following as a middle ear structure? 1- Auricle 2- Membranous labyrinth 3- Ossicles 4- Organ of Corti

3

Parathormone regulates the concentration of calcium in the blood by: 1- Stimulating osteoblastic activity. 2- Stimulating the liver to produce an insulin-like growth factor. 3- Promoting the movement of calcium from the bone. 4- Increasing the release of thyroid hormones.

3

Tap water irrigation of an older adult's ear canal has failed to dislodge the impacted cerumen that has been causing the patient to experience otalgia. In preparation for another attempt at relieving the patient's problem, what action should the nurse take? 1- Instill a hypertonic saline solution for 30 minutes prior to irrigating the patient's ear. 2- Attempt irrigation using a 10 mL syringe and higher pressure. 3- Put a small amount of mineral oil in the patient's ear before attempting removal. 4- Use a flexible suction catheter and 100 to 120 mm Hg of wall suction to remove the cerumen.

3

The client presents to the emergency department with fever, chills, restlessness, and limited movement of a fractured jaw. The nurse interprets these findings as indicating which of the following complications? 1- Avascular necrosis 2- Fat embolism 3- Osteomyelitis 4- Compartment syndrome

3

The emergency department nurse is caring for a patient who has been brought in by ambulance after sustaining a fall at home. The patient is exhibiting an altered level of consciousness. Following a skull X-ray, the patient is diagnosed with a basilar skull fracture. Which sign should alert the nurse to this type of fracture? 1- Babinski sign 2- Kernig's sign 3- Battle's sign 4- Brudzinski's sign

3

The human body is designed to protect its vital parts. The nurse is aware that a fracture of what type of bone may interfere with the protection of vital organs? 1- Long bones 2- Short bones 3- Flat bones 4- Irregular bones

3

The most important nursing priority of treatment for a patient with an altered LOC is to: 1- Stabilize the blood pressure and heart rate to ensure adequate perfusion of the brain. 2- Prevent dehydration and renal failure by inserting an IV line for fluids and medications. 3- Maintain a clear airway to ensure adequate ventilation. 4- Position the patient to prevent injury and ensure dignity.

3

The nurse assesses soft subcutaneous nodules along the line of the tendons in a patient's hand and wrist. What does this finding indicate to the nurse? 1- The patient has osteoarthritis. 2- The patient has lupus erythematosus. 3- The patient has rheumatoid arthritis. 4- The patient has neurofibromatosis.

3

The nurse caring for a client with Ménière's disease makes which primary assessments? 1- Ability to function 2- Social support 3- Gross hearing 4- Level of disability

3

The nurse has educated a patient with low back pain about techniques to relieve the back pain and prevent further complications. What statement by the patient shows understanding of the education the nurse provided? 1- "I will lie prone with my legs slightly elevated." 2- "I will bend at the waist when I am lifting objects from the floor." 3- "I will avoid prolonged sitting or walking." 4- "Instead of turning around to grasp an object, I will twist at the waist."

3

The nurse is caring for a client following a spinal cord injury who has a halo device in place. The client is preparing for discharge. Which statement by the client indicates the need for further instruction? 1- "I will change the vest liner periodically." 2- "If a pin becomes detached, I'll notify the surgeon." 3- "I can apply powder under the liner to help with sweating." 4- "I'll check under the liner for blisters and redness."

3

The nurse is caring for a client immediately following a spinal cord injury (SCI). Which is an acute complication of SCI? 1- Cardiogenic shock 2- Tetraplegia 3- Spinal shock 4- Paraplegia

3

The nurse is caring for a client ordered for multiple eye screening. Following which procedure will the nurse instruct the client on a yellow coloring to the skin and urine as being normal? 1- Ultrasonography 2- Retinal Imaging 3- Retinal Angiography 4- Retinoscopy

3

The nurse is caring for a client with chronic migraines who is prescribed medication. What drug-related instructions should the nurse give the client? 1- Take medication just before going to bed at night. 2- Take medication only when migraine is intense. 3- Take medication as soon as symptoms of the migraine begin. 4- Take medication only during the morning when it's calm and quiet.

3

The nurse is caring for a patient in the emergency department with a diagnosed epidural hematoma. What procedure will the nurse prepare the patient for? 1- Hypophysectomy 2- Application of Halo traction 3- Burr holes 4- Insertion of Crutchfield tongs

3

The nurse is caring for a patient with a bone tumor. The nurse provides education that teaches the patient to implement measures to reduce the risk of pathologic fractures. What intervention will assist the patient in fracture prevention? 1- Teaching the patient to achieve maximum weight-bearing capabilities 2- Maintaining strict bed rest 3- Supporting the affected extremity with external supports (splints) 4- Limiting the patient's reliance on assistive devices

3

The nurse is conducting a physical assessment of a male patient who has been admitted to the hospital unit. The nursing documentation on the unit specifies an assessment of the patient's direct and consensual pupillary response. How should the nurse assess the patient's consensual pupillary response? 1- Ask the patient to follow the movement of pen from several feet away to near the tip of the patient's nose 2- Ask the patient to look straight ahead while bringing a penlight in from the periphery of the patient's vision 3- Shine a penlight in one of the patient's eyes while observing the response of the opposite eye 4- Shine a penlight in the patient's eye while asking him to identify a common object with the other eye

3

The nurse is discussing conservative management of tendonitis with a patient. Which of the following is likely the most effective approach to managing tendonitis? 1- Weight reduction 2- Stress reduction 3- Intermittent application of ice and heat 4- Range-of-motion (ROM) exercise of the affected joint

3

The nurse is educating the patient with low back pain about the proper way to lift objects. What muscle should the nurse encourage the patient to maximize? 1- Gastrocnemius 2- Latissimus dorsi 3- Quadriceps 4- Rectus abdominis

3

The nurse is instructing the client with dried cerumen blocking the ear canal on potential methods to reduce symptoms. Which at-home methods of cerumen removal are discouraged? 1- Instilling 1 to 2 drops of half-strength peroxide in the ear 2- Using warm glycerin or mineral oil to soften the cerumen 3- Removing the cerumen by means of a cotton tip applicator 4- Irrigating the ear with warm water and a rubber-bulb syringe

3

The nurse is performing a musculoskeletal assessment of a client in a nursing home who had a stroke 2 years ago and who has right-sided hemiplegia. The nurse notes that the girth of the client's right calf is 2 inches less in diameter than the left calf. The nurse attributes the decreased girth to 1- edema in left lower extremity. 2- increased use of left calf muscle. 3- atrophy of right calf muscle. 4- bruising in right lower extremity.

3

The nurse is performing a neurovascular assessment of a client's injured extremity. Which would the nurse report? 1- Positive distal pulses 2- Capillary refill of 3 seconds 3- Dusky or mottled skin color 4- Skin warm to touch

3

The nurse is performing an assessment of the visual fields for a patient with glaucoma. When assessing the visual fields in acute glaucoma, what would the nurse expect to find? 1- Clear cornea 2- Constricted pupil 3- Marked blurring of vision 4- Watery ocular discharge

3

The nurse notes that the client's left great toe deviates laterally. This finding would be recognized as which condition? 1- Hammertoe 2- Pes cavus 3- Hallux valgus 4- Flatfoot

3

What kind of otitis media is a pathogen-free fluid behind the tympanic membrane, resulting from irritation associated with respiratory allergies and enlarged adenoids? 1- Purulent otitis media 2- Infectious otitis media 3- Serous otitis media 4- Sterile otitis media

3

When performing a physical assessment of a client, the client reports numbness, tingling, and pain when the nurse percusses lightly over the median nerve. The nurse recognizes that this finding is consistent with: 1- Morton's neuroma 2- Dupuytren's contracture 3- Carpal tunnel syndrome 4- Impingement syndrome

3

When preparing a teaching plan for a client diagnosed with otitis externa, the nurse instructs the client to avoid any water sport for which duration? 1- 3 to 5 days 2- 5 to 7 days 3- 7 to 10 days 4- 10 to 14 days

3

When undergoing testing of visual acuity with a Snellen chart, the client can read the line labeled 20/50 but misses three letters on the line. The nurse documents this finding as which of the following? 1- 20/20 + 30 2- 20/20/50 3- 20/50-3 4- 20/50

3

Which is useful in identifying acute or chronic tears of the joint capsule or supporting ligaments of the knee, shoulder, ankle, hip, or wrist? 1- Meniscography 2- Bone densitometry 3- Arthrography 4- EMG

3

Which of the following factors would the nurse need to keep in mind about the focus of the initial history when assessing a new client with a musculoskeletal problem? 1- Client's age 2- Client's lifestyle 3- Any chronic disorder or recent injury 4- Duration and location of discomfort or pain

3

Which of the following nursing interventions would be most likely to facilitate effective communication with a hearing impaired patient? 1- Ask the patient to repeat what was said in order to evaluate understanding. 2- Stand directly in front of the patient to facilitate lip reading. 3- Reduce environmental noise and distractions before communicating. 4- Raise the voice to project sound at a higher frequency.

3

Which test uses a tuning fork shifted between two positions to assess hearing? 1- Whisper 2- Watch tick 3- Rinne 4- Weber

3

A 22-year-old college basketball player is undergoing diagnostic testing following a knee injury, and his primary care provider has ordered arthrography. What teaching should the nurse provide in preparation for this procedure? 1- "A computer will create a three-dimensional image of your knee based on magnetic readings." 2- "You'll be given some anesthetic and then several tissue samples will be taken from your knee joint." 3- "You'll be given a small dose of radioactive solution, and the doctor will observe its distribution in your knee." 4- "The doctor will inject some contrast solution into your joint and take a series of X-rays."

4

A 49-year-old man with a history of poorly controlled type 1 diabetes has developed osteomyelitis adjacent to a chronic diabetic ulcer on his great toe. The patient has been informed that medical treatment for osteomyelitis requires a longer course of antibiotics than most other infections because: 1- Osteomyelitis is usually caused by simultaneous infection with several microorganisms, which must be treated sequentially. 2- Osteomyelitis requires treatment with topical antibiotics rather than IV antibiotics, necessitating a longer course of treatment. 3- Osteomyelitis is usually the result of fungal infection rather than bacterial infection. 4- Osteomyelitis involves the active infection of bone tissue, which is largely avascular.

4

A client arrives at the orthopedic physician's office stating knee pain sustained while playing soccer. A history and physical assessment is completed. The knee appears reddened with edema. Which other diagnostic testing would the nurse anticipate? 1- A bone densitometry 2- A bone scan 3- An arthrocentesis 4- An arthroscopy

4

A client comes to the walk-in clinic complaining of an earache. The cause is found to be impacted cerumen. The client asks the nurse what he can do at home to soften hardened cerumen. What should the nurse recommend to a client to soften hardened cerumen? 1- Avoid harsh sunlight. 2- Increase intake of red meat. 3- Increase intake of beta-carotene. 4- Take nonprescription preparations.

4

A client has been diagnosed with a concussion and is to be released from the emergency department. The nurse teaches the family or friends who will be caring for the client to contact the physician or return to the ED if the client 1- reports a headache. 2- reports generalized weakness. 3- sleeps for short periods of time. 4- vomits.

4

A client has sustained a traumatic brain injury. Which of the following is the priority nursing diagnosis for this client? 1- Deficient fluid balance related to decreased level of consciousness and hormonal dysfunction 2- Ineffective cerebral tissue perfusion related to increased intracranial pressure 3- Disturbed thought processes related to brain injury 4- Ineffective airway clearance related to brain injury

4

A client is scheduled for an EEG. The client asks about any diet-related prerequisites before the EEG. Which diet-related advice should the nurse provide to the client? 1- Avoid eating food at least 8 hours before the test. 2- Include an increased amount of minerals in the diet. 3- Decrease the amount of minerals in the diet. 4- Avoid taking sedative drugs or drinks that contain caffeine for at least 8 hours before the test.

4

A client is seen in the emergency room for a knee injury that happened during a basketball game. Diagnostic tests reveal torn cords of fibrous connective tissue that connect muscles to bones. What type of tear has this client sustained? 1- Fascia 2- Ligament 3- Bursa 4- Tendon

4

A client is transferred to the intensive care unit after evacuation of a subdural hematoma. Which nursing intervention reduces the client's risk of increased intracranial pressure (ICP)? 1- Encouraging oral fluid intake 2- Suctioning the client once each shift 3- Elevating the head of the bed 90 degrees 4- Administering a stool softener as ordered

4

A client is weak and drowsy after a lumbar puncture. The nurse caring for the client knows that what priority nursing intervention should be provided after a lumbar puncture? 1- Administer antihistamines to the client. 2- Provide adequate caffeine-rich drinks to the client. 3- Assess the level of consciousness (LOC) and the pupil response of the client. 4- Position the client flat for at least 3 hours.

4

A client with chronic osteomyelitis has undergone 6 weeks of antibiotic therapy. The wound appearance has not improved. What action would the nurse anticipate to promote healing? 1- Wound packing 2- Wound irrigation 3- Vitamin supplements 4- Surgical debridement

4

A client with low back pain is being seen in the clinic. In planning care, which teaching point should the nurse include? 1- Sleep on the stomach to alleviate pressure on the back. 2- A soft mattress is most supportive by conforming to the body. 3- Avoid twisting and flexion activities. 4- Use the large muscles of the leg when lifting items.

4

A client with osteoporosis is prescribed a selective estrogen receptor modifier (SERM) as treatment. The nurse would identify which drug as belonging to this class? 1- Alendronate (Fosamax) 2- Calcium gluconate 3- Tamoxifen (Nolvadex) 4- Raloxifene (Evista)

4

A client's vision is assessed at 20/200. The client asks what that means. Which is the mostappropriate response by the nurse? 1- "You see an object from 200 feet away that a person with normal vision sees from 20 feet away." 2- "You see an object from 20 feet away just like a person with normal vision." 3- "You see an object from 200 feet away just like a person with normal vision." 4- "You see an object from 20 feet away that a person with normal vision sees from 200 feet away."

4

A group of students are studying for an examination on joints. The students demonstrate understanding of the material when they identify which of the following as an example of a synarthrodial joint? 1- Between the vertebrae 2- In the fingers 3- At the hip 4- Skull at the temporal and occipital bones

4

A medical nurse has provided care for many patients who have bone disorders, and the nurse integrates health education around the maintenance of bone health into care. What should the nurse recommend to patients in an effort to promote bone health? 1- Moderate activity restriction for older adults 2- Daily use of low-dose ASA 3- A low-fat, low-carbohydrate diet 4- Adequate dietary intake of calcium

4

A nurse conducts the Romberg test by asking the client to stand with the feet close together and the eyes closed. As a result of this posture, the client suddenly sways to one side and is about to fall when the nurse intervenes and prevents the client from being injured. In which way should the nurse interpret the client's result? 1- Positive Romberg test, indicating a problem with level of consciousness 2- Negative Romberg test, indicating a problem with body mass 3- Negative Romberg test, indicating a problem with vision 4- Positive Romberg test, indicating a problem with equilibrium

4

A nurse is asked to assess a patient's need for a hearing aid. The nurse knows that a general guideline to determine need would be a hearing loss of: 1- 10 dB in the range of 200 to 1,000 Hz. 2- 20 dB in the range of 300 to 1,000 Hz. 3- 30 dB in the range of 400 to 1,500 Hz. 4- 40 dB in the range of 500 to 2,000 Hz.

4

A nurse is assisting with the clinical examination for determination of brain death for a client, related to potential organ donation. All 50 states in the United States recognize uniform criteria for brain death. The nurse is aware that the three cardinal signs of brain death on clinical examination are all of the following except: 1- Coma 2- Absence of brain stem reflexes 3- Apnea 4- Glasgow Coma Scale of 6

4

A nurse is aware of the significant potential benefits of cochlear implants for certain individuals. Which one of the following individuals is the most appropriate candidate for a cochlear implant? 1- An individual who has not yet experienced hearing loss but who possesses multiple risk factors 2- An individual who has been diagnosed with presbycusis 3- An individual who has longstanding and severe conductive hearing loss 4- An individual who has profound, bilateral, sensorineural hearing loss

4

A nurse is caring for a client who's experiencing septic arthritis. This client has a history of immunosuppressive therapy and the immune system is currently depressed. Which assignment is the most appropriate for the nurse caring for this client? 1- The nurse caring for this client is also caring for four other immunosuppressed clients on the medical floor. 2- The nurse caring for this client is also caring for four clients receiving chemotherapy for cancer treatment on the oncology floor. 3- The nurse caring for this client is also caring for two other immunosuppressed clients on the medical intensive care unit. 4- The nurse is caring for this client on the intensive care unit.

4

A nurse is caring for a client with an injury to the central nervous system. When caring for a client with a spinal cord insult slowing transmission of the motor neurons, the nurse would anticipate a delayed reaction in: 1- identification of information due to slowed passages of information to brain. 2- cognitive ability to understand relayed information. 3- processing information transferred from the environment. 4- response due to interrupted impulses from the central nervous system

4

A nurse is completing a neurological assessment and determines that the client has significant visual deficits. Considering the functions of the lobes of the brain, which area will most likely contain the neurologic deficit? 1- frontal 2- parietal 3- temporal 4- occipital

4

A nurse is conducting an assessment of a patient's musculoskeletal system and assessing for joint function. How should the nurse best perform this assessment? 1- Ask the patient to clearly describe any recent changes in mobility or flexibility. 2- Assess the patient's expectations around function and mobility. 3- Palpate and percuss each of the patient's joints bilaterally. 4- Assess the range of motion (ROM) of each of the patient's joints.

4

A nurse is performing discharge teaching for an elderly client with osteoporosis. Which instruction about taking a calcium supplement should the nurse include? 1- Take weekly on the same day and at the same time. 2- Remain in an upright position 30 minutes after taking the supplement. 3- Take the supplement on an empty stomach with a full glass of water. 4- Take the supplement with meals or with orange juice.

4

A nurse is preparing a presentation for a local community about hearing loss and prevention. Which of the following would the nurse integrate into the presentation as the most effective preventive measure? 1- Maintaining daily hygiene for the ears 2- Having yearly audiometric testing 3- Obtaining prompt treatment for ear infections 4- Wearing ear protection when exposed to noise

4

A nurse knows that a person with a 3-week-old femur fracture is at the stage where angiogenesis is occurring. What are the characterisitics of this stage? 1- The formation of a hematoma and fibrin. 2- Inflammation and the stimulation of osteoblasts and osteoclasts. 3- Cartilage cells forming matrix villa that regulate calcification of the cartilage. 4- New capillaries producing a bridge between the fractured bones.

4

A nurse observes an abnormal posture response in an unconscious patient. She documents "extension and outward rotation of the upper extremities and plantar flexion of the feet." She is aware that this posture is a clinical indicator of which of the following? 1- A brain lesion that causes a spontaneous response that changes with electrical activity in the brain 2- Cerebral hemisphere pathology that will cause alterations in flaccidity and contraction of motor responses 3- Decorticate positioning indicating damage to the upper midbrain 4- Decerebrate positioning implying severe dysfunction and brain pathology

4

A nurse practitioner in an emergency room receives a telephone call from a mother whose 4-year-old child has a mosquito stuck in his external ear canal. Which of the following is the best information the nurse could give the mother? 1- Irrigate the ear canal with warm water to flush out the insect. 2- Use an aural suction cup to pull out the insect. 3- Insert a cotton-tipped applicator (e.g., Q-tip) to trap the insect and slowly pull the applicator backward. 4- Instill a few drops of warmed mineral oil to cover the insect.

4

A nurse who is responsible for planning many of the aspects of care on a subacute geriatric unit is aware of the high incidence and prevalence of vestibular disorders among older adults. In individuals with these disorders, the nurse should identify what consequent nursing diagnosis? 1- Risk for impaired tissue integrity related to vestibular disorder 2- Risk for chronic confusion related to vestibular disorder 3- Risk for aspiration related to vestibular disorder 4- Risk for falls related to vestibular disorder

4

A patient had hand surgery to correct a Dupuytren's contracture. What nursing intervention is a priority postoperatively? 1- Changing the dressing 2- Applying a cock-up splint and immobilization 3- Having the patient exercise the fingers to avoid future contractures 4- Performing hourly neurovascular assessments for the first 24 hours

4

A patient is admitted to the emergency room with a fractured skull sustained in a motorcycle accident. The nurse notes fluid leaking from the patient's ears. The nurse knows this is a probable sign of which type of skull fracture? 1- Simple 2- Comminuted 3- Depressed 4- Basilar

4

A patient is admitted to the emergency room with a skull fracture. The nurse notes a blood stain, surrounded by a yellowish ring, on the linens on the stretcher. The patient's respiratory system was stabilized at the site of the accident. Which of the following nursing interventions describes the immediate nursing action that needs to be taken? 1- Tell the patient not to blow his nose. 2- Put a note about avoiding the use of a suction catheter at the head of the bed. 3- Test the fluid leaking from the patient's ears for glucose. 4- Elevate the head of the bed 30 degrees.

4

A patient is being tested for a gag reflex. When the nurse places the tongue blade to the back of the throat, there is no response elicited. What dysfunction does the nurse determine the patient has? 1- Dysfunction of the spinal accessory nerve 2- Dysfunction of the acoustic nerve 3- Dysfunction of the facial nerve 4- Dysfunction of the vagus nerve

4

A patient is having an eye examination. The ophthalmologist is looking inside the patient's eye with a hand-held instrument. What instrument is being used to perform this procedure? 1- Slit-lamp examination 2- An indirect ophthalmoscope 3- An Ishihara plate 4- A direct ophthalmoscope

4

A patient stepped on an acorn while walking barefoot in the backyard and developed an infection progressing to osteomyelitis. What microorganism does the nurse understand is most often the cause of the development of osteomyelitis? 1- Proteus 2- Pseudomonas 3- Salmonella 4- Staphylococcus aureus

4

A patient who has been diagnosed with colon cancer is scheduled to undergo positron emission tomography (PET) to search for metastases. In preparation for this diagnostic procedure, what teaching point should the nurse provide to the patient? 1- "Sound waves will be refracted throughout your body and a computer will analyze them." 2- "A series of X-rays will be taken that will be combined to create a three-dimensional image your body." 3- "It's very important that all metal objects be removed from your body before the test." 4- "You'll be given a radioactive substance that will be measured during the test."

4

A patient who has suffered a stroke is unable to maintain respiration and is intubated and placed on mechanical ventilator support. What portion of the brain is most likely responsible for the inability to breathe? 1- Frontal lobe 2- Occipital lobe 3- Parietal lobe 4- Brain stem

4

After a person experiences a closure of the epiphyses, which statement is true? 1- The bone grows in length but not thickness. 2- The bone increases in thickness and is remodeled. 3- Both bone length and thickness continue to increase. 4- No further increase in bone length occurs.

4

After several weeks of antibiotic therapy for the treatment of osteomyelitis, a patient is preparing for discharge. When providing health education related to self-care, the nurse should emphasize which of the following topics? 1- The need to avoid ASA and anticoagulants 2- The need to resume normal physical activity as soon as possible 3- The importance of maintaining a healthy diet 4- The importance of adhering to further antibiotic treatment

4

An elderly patient has come to the clinic for a regular check-up. While reviewing a patient's history, the nurse notes that the patient has an increased thoracic curvature of the spine. What term describes this assessment finding? 1- Scoliosis 2- Epiphyses 3- Lordosis 4- Kyphosis

4

An emergency department nurse is assessing an 80-year-old patient who has presented with a complaint of abdominal pain. The nurse performs a rapid inspection of the patient and notes multiple significant findings, including kyphosis. The nurse should understand that this assessment finding is suggestive of what musculoskeletal disease? 1- Malignant bone neoplasm 2- Paget's disease 3- Muscular dystrophy 4- Osteoporosis

4

Audiometry confirms a client's chronic progressive hearing loss. Further investigation reveals ankylosis of the stapes in the oval window, a condition that prevents sound transmission. This type of hearing loss is called: 1- functional hearing loss. 2- fluctuating hearing loss. 3- sensorineural hearing loss. 4- conductive hearing loss.

4

Autonomic dysreflexia is an acute emergency that occurs with spinal cord injury as a result of exaggerated autonomic responses to stimuli. Which of the following is the initial nursing intervention to treat this condition? 1- Examine the skin for any area of pressure or irritation. 2- Examine the rectum for a fecal mass. 3- Empty the bladder immediately. 4- Raise the head of the bed and place the patient in a sitting position.

4

In chronic osteomyelitis, antibiotics are adjunctive therapy in which situation? 1- Wound packing 2- Wound irrigation 3- Vitamin supplements 4- Surgical debridement

4

Localized rapid bone turnover, most commonly affecting the skull, femur, tibia, pelvic bones, and vertebrae, is characterized by which bone disorder? 1- Osteomalacia 2- Ganglion 3- Osteomyelitis 4- Paget disease

4

The client diagnosed with osteosarcoma is scheduled for a surgical amputation. Which nursing diagnosis would be a priority for this client compared with other surgical clients? 1- Inadequate nutrition 2- Impaired physical mobility 3- Risk for infection 4- Disturbed body image

4

The nurse at the eye clinic is caring for a patient with suspected glaucoma. What complaint would be significant for a diagnosis of glaucoma? 1- A significant loss of central vision 2- Diminished acuity 3- Pain associated with a purulent discharge 4- The presence of halos around lights

4

The nurse in the emergency department is caring for a patient brought in by the rescue squad after falling from a second-story window. The nurse assesses ecchymosis over the mastoid and clear fluid from the ears. What type of skull fracture is this indicative of? 1- Occipital skull fracture 2- Temporal skull fracture 3- Frontal skull fracture 4- Basilar skull fracture

4

The nurse is admitting a patient to the unit who is diagnosed with a lower motor neuron lesion. What clinical manifestations would this patient most likely exhibit? 1- Increased muscle tone 2- No muscle atrophy 3- Hyperactive and abnormal reflexes 4- Absent or decreased reflexes

4

The nurse is assessing the throat of a client with throat pain. In asking the client to stick out the tongue, the nurse is also assessing which cranial nerve? 1- Cranial nerve I 2- Cranial nerve V 3- Cranial nerve XI 4- Cranial nerve XII

4

The nurse is caring for a client following foot surgery. Which nursing intervention is most important for the nurse to include in the nursing care plan? 1- Examine surgical dressing every hour. 2- Administer pain medication per client request. 3- Monitor vital signs every 4 hours. 4- Perform neuromuscular assessment every hour.

4

The nurse is doing discharge teaching with a client newly diagnosed with Ménière's disease. Why would the nurse advise a low-sodium diet to this client? 1- To minimize the adverse effects of drug therapy 2- To reduce the magnitude of the hearing deficit 3- To minimize the risk of a tumor that involves the vestibulocochlear nerve 4- To reduce the production of fluid in the inner ear

4

The nurse is giving a visual field examination to a 55-year-old male client. The client asks what this test is for. What would be the nurse's best answer? 1- "This test measures visual acuity." 2- "This test measures how well your eyes move." 3- "This test is to see how well your eyes are aging." 4- "This test measures peripheral vision and detects gaps in the visual field."

4

The nurse is performing a musculoskeletal assessment on a patient with arthritis. During passive range-of-motion exercises, the nurse hears grating and cracking sounds. This assessment finding is referred to as what? 1- Fasciculations 2- Clonus 3- Effusion 4- Crepitus

4

The nurse is performing an assessment on an older adult patient and observes the patient has an increased forward curvature of the thoracic spine. What does the nurse understand this common finding is known as? 1- Lordosis 2- Scoliosis 3- Osteoporosis 4- Kyphosis

4

The nurse is planning discharge instructions for the client with osteomyelitis. What instructions should the nurse include in the discharge teaching? 1- "Use your continuous passive motion machine (CPM) 2 hours each day." 2- "You need to perform weight-bearing exercises twice a week." 3- "You need to limit the amount of protein and calcium in your diet." 4- "You will receive IV antibiotics for 3 to 6 weeks."

4

The nurse is reporting on the results of client blood work to the oncoming nurse. Upon reviewing the data, it is noted that the client has an elevated uric acid level. Which inflammatory process would the nurse screen for on shift rounds? 1- Rheumatoid arthritis 2- Lupus erythematosus 3- Osteoporosis 4- Gout

4

The nurse is reviewing the medication administration record of the client. Which of the following medications would lead the nurse to suspect that the client is at risk for osteoporosis? 1- penicillamine (Cuprimine) 2- methotrexate (Rheumatrex) 3- plicamycin (Mithracin) 4- raloxifene (Evista)

4

The nurse is working in the emergency department when a physician asks for help as the client is performing a Romberg test. In which position would the nurse stand to be most helpful? 1- The nurse would stand directly in front of the client. 2- The nurse would stand between the client and physician. 3- The nurse would stand across the room but in direct alignment from the client. 4- The nurse would stand laterally to the client, opposite side to where the physician is standing.

4

The nurse is working in the triage section of a walk-in clinic. Which triad of common symptoms, when placed together, indicate Ménière's disease? 1- Blurred vision, vertigo, nausea 2- Syncope, vertigo, ear pain 3- Disorientation, vertigo, nausea 4- Hearing loss, vertigo, tinnitus

4

The nurse working in the emergency department receives a call from the x-ray department communicating that the client the nurse is caring for has a fracture in the shaft of the tibia. The nurse tells the physician that the client's fracture is in the 1- epiphysis. 2- lordosis. 3- scoliosis. 4- diaphysis.

4

The physician's office nurse is caring for a client who has a history of a cerebral aneurysm. Which diagnostic test does the nurse anticipate to monitor the status of the aneurysm? 1- Myelogram 2- Electroencephalogram 3- Echoencephalography 4- Cerebral angiography

4

The pre-nursing class is learning about the nervous system in their anatomy class. What part of the nervous system would the students learn is responsible for digesting food and eliminating body waste? 1- Central 2- Sympathetic 3- Peripheral 4- Parasympathetic

4

To assess a client's cranial nerve function, a nurse should assess: 1- hand grip. 2- orientation to person, time, and place. 3- arm drifting. 4- gag reflex.

4

What clinical manifestation would the nurse expect to find in a client who has had osteoporosis for several years? 1- Bone spurs 2- Diarrhea 3- Increased heel pain 4- Decreased height

4

What does the nurse recognize as the earliest sign of serious impairment of brain circulation related to increasing ICP? 1- A bounding pulse 2- Bradycardia 3- Hypertension 4- Lethargy and stupor

4

When caring for a client after ear surgery, what is an important aspect of nursing care? 1- Assess social support. 2- Feed small frequent meals to minimize nausea. 3- Fit for a hearing aid. 4- Validate client's feelings of discomfort.

4

When caring for a client who is post-intracranial surgery what is the most important parameter to monitor? 1- Extreme thirst 2- Intake and output 3- Nutritional status 4- Body temperature

4

When caring for a client who is post-intracranial surgery, what is the most important parameter to monitor? 1- Signs of infection 2- Intake and output 3- Nutritional status 4- Body temperature

4

When discussing diseases of the middle ear, the nursing instructor distinguishes the different types of otitis media. What generally causes purulent otitis media? 1- Irritation associated with respiratory allergies and enlarged adenoids 2- Bronchial tree 3- Outer ear 4- Upper respiratory infections

4

When learning about the nervous system, students learn that which nervous system regulates the expenditure of energy? 1- Parasympathetic 2- Central 3- Peripheral 4- Sympathetic

4

Which assessment findings would the nurse expect in a client diagnosed with osteomyelitis? 1- Thrombocytopenia and ecchymosis 2- Pruritus and uremic frost 3- Petechiae over the chest and abnormal ABGs 4- Leukocytosis and localized bone pain

4

Which is a neurovascular problem caused by pressure within a muscle area that increases to such an extent that microcirculation diminishes? 1- Remodeling 2- Hypertrophy 3- Fasciculation 4- Compartment syndrome

4

Which laboratory study indicates the rate of bone turnover? 1- Urine calcium 2- Serum calcium 3- Serum phosphorous 4- Serum osteocalcin

4

Which neurotransmitter demonstrates inhibitory action, helps control mood and sleep, and inhibits pain pathways? 1- Enkephalin 2- Norepinephrine 3- Acetylcholine 4- Serotonin

4

Which nursing diagnosis takes highest priority for a client admitted for evaluation for Ménière's disease? 1- Acute pain related to vertigo 2- Imbalanced nutrition: Less than body requirements related to nausea and vomiting 3- Risk for deficient fluid volume related to vomiting 4- Risk for injury related to vertigo

4

Which of the following is the most important nursing diagnosis for an elderly patient diagnosed with osteoporosis? 1- Deficient knowledge about osteoporosis and the treatment regimen 2- Acute pain related to fracture and muscle spasm 3- Risk for constipation related to immobility 4- Risk for injury related to fractures due to osteoporosis

4

Which safety action will the nurse implement for a client receiving oxygen therapy who is undergoing magnetic resonance imaging (MRI)? 1- Securely fasten the client's portable oxygen tank to the bottom of the MRI table after the client has been positioned on the table. 2- Check the client's oxygen saturation level using a pulse oximeter after the client has been placed on the MRI table. 3- Note that no special safety actions need to be taken. 4- Ensure that no client care equipment containing metal enters the room where the MRI table is located.

4

Which should be included in the teaching plan for a client diagnosed with plantar fasciitis? 1- Plantar fasciitis presents as acute-onset pain localized to the ball of the foot that occurs when pressure is placed on it and diminishes when pressure is released. 2- The pain of plantar fasciitis diminishes with warm water soaks. 3- Complications of plantar fasciitis include neuromuscular damage and decreased ankle range of motion. 4- Management of plantar fasciitis includes stretching exercises.

4

Which term refers to a flexion deformity caused by a slowly progressive contracture of the palmar fascia? 1- Callus 2- Hammertoe 3- Hallux valgus 4- Dupuytren contracture

4

At what rate (in drops per minute) should a nurse start an IV infusion if the order is for 1 g of vancomycin (Vancocin) to be given in 180 ml of dextrose 5% in water over 60 minutes? The tubing delivers 15 drops/ml. Enter the correct number only.

45

A patient is recovering in the hospital following a total hip replacement that was performed 2 days ago. In an effort to prevent the common complications associated with the surgical procedure, the nurse should implement which of the following interventions, as ordered? A Application of sequential compression devices B Intermittent urinary catheterization to prevent urinary retention C Provision of a low-fiber, high-calorie diet D Passive range-of-motion (ROM) exercises with the affected leg

A

The nurse is caring for a client who was involved in an automobile accident and sustained multiple trauma. The client has a Volkmann's contracture to the right hand. What objective data does the nurse document related to this finding? A Clawlike deformity of the right hand without ability to extend fingers B Nodules on the knuckles of the third and fourth finger C Extension of the fingers of the right hand D Dislocation of the fingers

A A Volkmann's contracture is a claw like deformity of the hand resulting from obstructed arterial blood flow to the forearm and hand. The client is unable to extend the fingers and complains of unrelenting pain, particularly if attempting to stretch the hand

An adult is swinging a small child by the arms, and the child screams and grabs his left arm. It is determined in the emergency department that the radial head is partially dislocated. What is this partially dislocated radial head documented as? A Subluxation B Volkmann's contracture C Compartment syndrome D Sprain

A A partial dislocation is referred to as a subluxation. A Volkmann's contracture is a claw like deformity that results from compartment syndrome or obstructed arterial blood flow to the forearm and hand. Compartment syndrome is a condition in which a structure such as a tendon or nerve is constricted in a confined space and affects nerve innervation, leading to subsequent palsy. A sprain is injury to the ligaments surrounding the joint.

After surgery to treat a hip fracture, a client returns from the postanesthesia care unit to the medical-surgical unit. Postoperatively, how should the nurse position the client? A With the leg on the affected side abducted B With the affected hip rotated externally C With the leg on the affected side adducted D With the affected hip flexed acutely

A The nurse must keep the leg on the affected side abducted at all times after hip surgery to prevent accidental dislodgment of the affected hip joint. Placing a pillow or an A-frame between the legs helps maintain abduction and reminds the client not to cross the legs. The nurse should avoid acutely flexing the client's affected hip (for example, by elevating the head of the bed excessively), adducting the leg on the affected side (such as by moving it toward the midline), or externally rotating the affected hip (such as by removing support along the outer side of the leg) because these positions may cause dislocation of the injured hip joint.

A 77-year-old man is recovering in the hospital after a recent femoral fracture and has rung his call light. The nurse has entered the room to find the patient in distress, clutching his chest while struggling to say, "I can't breathe." The nurse should take prompt action based on the knowledge that this patient may be experiencing what complication of lower extremity fractures? A Thromboembolism B Acute respiratory distress syndrome (ARDS) C Ischemic stroke D Unstable angina

A Venous thromboemboli, including deep vein thrombosis (DVT) and pulmonary emboli (PE), are associated with reduced skeletal muscle contractions and bed rest. Patients with fractures of the lower extremities and pelvis are at high risk for venous thromboemboli. The most frequent signs are sudden onset shortness of breath, restlessness, increased respiratory rate, tachycardia, chest pain, and low-grade temperature. Angina, ARDS and stroke are not common complications of skeletal fractures.

Nurses are performing hourly neurological assessments of a patient who is postoperative day 1 following spinal surgery that was performed to treat spinal cord compression (SCC). The nurse's most recent assessment shows a marked reduction in motor and sensory function. How should the nurse best follow-up this assessment finding? A) Inform the care provider promptly about this sudden change. B) Reassess the patient in 15 minutes. C) Contact the anesthesiologist to discuss possible residual effects of anesthesia. D) Document these findings and have a colleague confirm the assessment.

A) Inform the care provider promptly about this sudden change. Sudden onset of neuro deficit is an ominous sign and may be due to vertebral collapse.

After suffering a fall, an 81-year-old woman with Alzheimer's disease (AD) is being treated in the hospital. Which of the following measures should be implemented in an effort to support the patient's cognitive function? A) Maintain consistent, predictable routines whenever possible. B) Provide an engaging, high-stimulation environment. C) Establish clear consequences for aggressive behavior. D) Ensure that the patient has a different care provider each day.

A) Maintain consistent, predictable routines whenever possible. - A calm, predictable environment helps people with AD interpret their surroundings and activities. Environmental stimuli are limited, and a regular routine is established. A quiet, pleasant manner of speaking, clear and simple explanations, and use of memory aids and cues help minimize confusion and disorientation and give patients a sense of security. Behavioral problems, such as agitation and psychosis, can be managed by behavioral and psychosocial therapies. Providing an environment that is controlled and stable can help with behavior problems.

A community health nurse is performing a home visit to a patient with amyotrophic lateral sclerosis (ALS). The nurse should prioritize assessments related to which of the following? A) Respiratory function B) Potential skin breakdown C) Cardiac function D) Cognition

A) Respiratory function - The most common reasons for hospitalization are dehydration and malnutrition, pneumonia, and respiratory failure; recognizing these problems at an early stage in the illness allows for the development of preventive strategies. In about 25% of patients, weakness starts in the muscles supplied by the cranial nerves, and difficulty in talking, swallowing, and, ultimately, breathing occurs.

A patient has been admitted to the neurological unit from the PACU after successful neurosurgery to remove a brain tumor. The nurse's admitting assessment reveals that the patient's gag reflex has not yet fully returned. The nurse should recognize that this assessment finding has the potential to cause which of the following nursing diagnoses? A)Risk for aspiration B)Impaired spontaneous ventilation C)Dysfunctional ventilator weaning response D)Imbalanced nutrition: less than body requirements

A)Risk for aspiration

On ocular examination, the health care provider notes severely elevated IOP, corneal edema, and a pupil that is fixed in a semi-dilated position. The nurse knows that these clinical signs are diagnostic of the type of glaucoma known as: Chronic open-angle. Acute angle-closure. Chronic angle-closure. Normal tension.

Acute angle-closure. Acute angle-closure glaucoma is characterized by the symptoms listed, as well as by being rapidly progressive and accompanied by pain.

A patient diagnosed with Bells palsy is being cared for on an outpatient basis. During health education, the nurse should promote which of the following actions? A) Applying a protective eye shield at night B) Chewing on the affected side to prevent unilateral neglect C) Avoiding the use of analgesics whenever possible D) Avoiding brushing the teeth

Ans: A Feedback: Corneal irritation and ulceration may occur if the eye is unprotected. While paralysis lasts, the involved eye must be protected. The patient should be encouraged to eat on the unaffected side, due to swallowing difficulties. Analgesics are used to control the facial pain. The patient should continue to provide self-care including oral hygiene.

When preparing to discharge a patient home, the nurse has met with the family and warned them that the patient may exhibit unexpected emotional responses. The nurse should teach the family that these responses are typically a result of what cause? A) Frustration around changes in function and communication B) Unmet physiologic needs C) Changes in brain activity during sleep and wakefulness D) Temporary changes in metabolism

Ans: A Feedback: Emotional problems associated with stroke are often related to the new challenges around ADLs and communication. These challenges are more likely than metabolic changes, unmet physiologic needs, or changes in brain activity, each of which should be ruled out.

The nurse is assessing a patient with a suspected stroke. What assessment finding is most suggestive of a stroke? A) Facial droop B) Dysrhythmias C) Periorbital edema D) Projectile vomiting

Ans: A Feedback: Facial drooping or asymmetry is a classic abnormal finding on a physical assessment that may be associated with a stroke. Facial edema is not suggestive of a stroke and patients less commonly experience dysrhythmias or vomiting.

The nurse is working with a patient who is newly diagnosed with MS. What basic information should the nurse provide to the patient? A) MS is a progressive demyelinating disease of the nervous system. B) MS usually occurs more frequently in men. C) MS typically has an acute onset. D) MS is sometimes caused by a bacterial infection.

Ans: A Feedback: MS is a chronic, degenerative, progressive disease of the central nervous system, characterized by the occurrence of small patches of demyelination in the brain and spinal cord. The cause of MS is not known, and the disease affects twice as many women as men.

A patient who suffered an ischemic stroke now has disturbed sensory perception. What principle should guide the nurses care of this patient? A) The patient should be approached on the side where visual perception is intact. B) Attention to the affected side should be minimized in order to decrease anxiety. C) The patient should avoid turning in the direction of the defective visual field to minimize shoulder subluxation. D) The patient should be approached on the opposite side of where the visual perception is intact to promote recovery.

Ans: A Feedback: Patients with decreased field of vision should first be approached on the side where visual perception is intact. All visual stimuli should be placed on this side. The patient can and should be taught to turn the head in the direction of the defective visual field to compensate for this loss. The nurse should constantly remind the patient of the other side of the body and should later stand at a position that encourages the patient to move or turn to visualize who and what is in the room.

You are the clinic nurse caring for a patient with a recent diagnosis of myasthenia gravis. The patient has begun treatment with pyridostigmine bromide (Mestinon). What change in status would most clearly suggest a therapeutic benefit of this medication? A) Increased muscle strength B) Decreased pain C) Improved GI function D) Improved cognition

Ans: A Feedback: The goal of treatment using pyridostigmine bromide is improvement of muscle strength and control of fatigue. The drug is not intended to treat pain, or cognitive or GI functions.

The patient has been diagnosed with aphasia after suffering a stroke. What can the nurse do to best make the patients atmosphere more conducive to communication? A) Provide a board of commonly used needs and phrases. B) Have the patient speak to loved ones on the phone daily. C) Help the patient complete his or her sentences. D) Speak in a loud and deliberate voice to the patient.

Ans: A Feedback: The inability to talk on the telephone or answer a question or exclusion from conversation causes anger, frustration, fear of the future, and hopelessness. A common pitfall is for the nurse or other health care team member to complete the thoughts or sentences of the patient. This should be avoided because it may cause the patient to feel more frustrated at not being allowed to speak and may deter efforts to practice putting thoughts together and completing a sentence. The patient may also benefit from a communication board, which has pictures of commonly requested needs and phrases. The board may be translated into several languages.

A patient is brought by ambulance to the ED after suffering what the family thinks is a stroke. The nurse caring for this patient is aware that an absolute contraindication for thrombolytic therapy is what? A) Evidence of hemorrhagic stroke B) Blood pressure of 180/110 mm Hg C) Evidence of stroke evolution D) Previous thrombolytic therapy within the past 12 months

Ans: A Feedback: Thrombolytic therapy would exacerbate a hemorrhagic stroke with potentially fatal consequences. Stroke evolution, high BP, or previous thrombolytic therapy does not contraindicate its safe and effective use.

A nurse is assessing a patient with an acoustic neuroma who has been recently admitted to an oncology unit. What symptoms is the nurse likely to find during the initial assessment? A)Loss of hearing, tinnitus, and vertigo B)Loss of vision, change in mental status, and hyperthermia C)Loss of hearing, increased sodium retention, and hypertension D)Loss of vision, headache, and tachycardia

Ans: A Feedback: An acoustic neuroma is a tumor of the eighth cranial nerve, the cranial nerve most responsible for hearing and balance. The patient with an acoustic neuroma usually experiences loss of hearing, tinnitus, and episodes of vertigo and staggering gait. Acoustic neuromas do not cause loss of vision, increased sodium retention, or tachycardia.

The nurse is planning the care of a patient who has been recently diagnosed with a cerebellar tumor. Due to the location of this patient's tumor, the nurse should implement measures to prevent what complication? A)Falls B)Audio hallucinations C)Respiratory depression D)Labile BP

Ans: A) Falls Feedback: A cerebellar tumor causes dizziness, an ataxic or staggering gait with a tendency to fall toward the side of the lesion, and marked muscle incoordination. Because of this, the patient faces a high risk of falls. Hallucinations and unstable vital signs are not closely associated with cerebellar tumors.

A male patient presents at the free clinic with complaints of impotency. Upon physical examination, the nurse practitioner notes the presence of hypogonadism. What diagnosis should the nurse suspect? A)Prolactinoma B)Angioma C)Glioma D)Adrenocorticotropic hormone (ACTH)-producing adenoma

Ans: A)Prolactinoma Feedback: Male patients with prolactinomas may present with impotence and hypogonadism. An ACTH-producing adenoma would cause acromegaly. The scenario contains insufficient information to know if the tumor is an angioma, glioma, or neuroma.

A patient with an inoperable brain tumor has been told that he has a short life expectancy. On what aspects of assessment and care should the home health nurse focus? Select all that apply. A)Pain control B)Management of treatment complications C)Interpretation of diagnostic tests D)Assistance with self-care E)Administration of treatments

Ans: A, B, D, E A)Pain control B)Management of treatment complications D)Assistance with self-care E)Administration of treatments Feedback: Home care needs and interventions focus on four major areas: palliation of symptoms and pain control, assistance in self-care, control of treatment complications, and administration of specific forms of treatment, such as parenteral nutrition. Interpretation of diagnostic tests is normally beyond the purview of the nurse.

The nurse is developing a plan of care for a patient newly diagnosed with Bells palsy. The nurses plan of care should address what characteristic manifestation of this disease? A) Tinnitus B) Facial paralysis C) Pain at the base of the tongue D) Diplopia

Ans: B Feedback: Bells palsy is characterized by facial dysfunction, weakness, and paralysis. It does not result in diplopia, pain at the base of the tongue, or tinnitus

A preceptor is discussing stroke with a new nurse on the unit. The preceptor would tell the new nurse which cardiac dysrhythmia is associated with cardiogenic embolic strokes? A) Ventricular tachycardia B) Atrial fibrillation C) Supraventricular tachycardia D) Bundle branch block

Ans: B Feedback: Cardiogenic embolic strokes are associated with cardiac dysrhythmias, usually atrial fibrillation. The other listed dysrhythmias are less commonly associated with this type of stroke.

A patient with possible bacterial meningitis is admitted to the ICU. What assessment finding would the nurse expect for a patient with this diagnosis? A) Pain upon ankle dorsiflexion of the foot B) Neck flexion produces flexion of knees and hips C) Inability to stand with eyes closed and arms extended without swaying D) Numbness and tingling in the lower extremities

Ans: B Feedback: Clinical manifestations of bacterial meningitis include a positive Brudzinskis sign. Neck flexion producing flexion of knees and hips correlates with a positive Brudzinskis sign. Positive Homans sign (pain upon dorsiflexion of the foot) and negative Rombergs sign (inability to stand with eyes closed and arms extended) are not expected assessment findings for the patient with bacterial meningitis. Peripheral neuropathy manifests as numbness and tingling in the lower extremities. Again, this would not be an initial assessment to rule out bacterial meningitis.

A patient with MS has developed dysphagia as a result of cranial nerve dysfunction. What nursing action should the nurse consequently perform? A) Arrange for the patient to receive a low residue diet. B) Position the patient upright during feeding. C) Suction the patient following each meal. D) Withhold liquids until the patient has finished eating.

Ans: B Feedback: Correct, upright positioning is necessary to prevent aspiration in the patient with dysphagia. There is no need for a low-residue diet and suctioning should not be performed unless there is an apparent need. Liquids do not need to be withheld during meals in order to prevent aspiration.

A patient has recently begun mobilizing during the recovery from an ischemic stroke. To protect the patients safety during mobilization, the nurse should perform what action? A) Support the patients full body weight with a waist belt during ambulation. B) Have a colleague follow the patient closely with a wheelchair. C) Avoid mobilizing the patient in the early morning or late evening. D) Ensure that the patients family members do not participate in mobilization.

Ans: B Feedback: During mobilization, a chair or wheelchair should be readily available in case the patient suddenly becomes fatigued or feels dizzy. The family should be encouraged to participate, as appropriate, and the nurse should not have to support the patients full body weight. Morning and evening activity are not necessarily problematic.

When caring for a patient who has had a stroke, a priority is reduction of ICP. What patient position is most consistent with this goal? A) Head turned slightly to the right side B) Elevation of the head of the bed C) Position changes every 15 minutes while awake D) Extension of the neck

Ans: B Feedback: Elevation of the head of the bed promotes venous drainage and lowers ICP, the nurse should avoid flexing or extending the neck or turning the head side to side. The head should be in a neutral midline position. Excessively frequent position changes are unnecessary.

A family member brings the patient to the clinic for a follow-up visit after a stroke. The family member asks the nurse what he can do to decrease his chance of having another stroke. What would be the nurses best answer? A) Have your heart checked regularly. B) Stop smoking as soon as possible. C) Get medication to bring down your sodium levels. D) Eat a nutritious diet.

Ans: B Feedback: Smoking is a modifiable and highly significant risk factor for stroke. The significance of smoking, and the potential benefits of quitting, exceed the roles of sodium, diet, and regular medical assessments.

A 25-year-old female patient with brain metastases is considering her life expectancy after her most recent meeting with her oncologist. Based on the fact that the patient is not receiving treatment for her brain metastases, what is the nurse's most appropriate action? A)Promoting the patient's functional status and ADLs B)Ensuring that the patient receives adequate palliative care C)Ensuring that the family does not tell the patient that her condition is terminal D)Promoting adherence to the prescribed medication regimen

Ans: B) Ensuring that the patient receives adequate palliative care Feedback: Patients with intracerebral metastases who are not treated have a steady downhill course with a limited survival time, whereas those who are treated may survive for slightly longer periods, but for most cure is not possible. Palliative care is thus necessary. This is a priority over promotion of function and the family should not normally withhold information from the patient. Adherence to medications such as analgesics is important, but palliative care is a high priority.

A patient has been admitted to the neurologic unit for the treatment of a newly diagnosed brain tumor. The patient has just exhibited seizure activity for the first time. What is the nurse's priority response to this event? A)Identify the triggers that precipitated the seizure. B)Implement precautions to ensure the patient's safety. C)Teach the patient's family about the relationship between brain tumors and seizure activity. D)Ensure that the patient is housed in a private room.

Ans: B)Implement precautions to ensure the patient's safety. Feedback: Patients with seizures are carefully monitored and protected from injury. Patient safety is a priority over health education, even though this is appropriate and necessary. Specific triggers may or may not be evident; identifying these is not the highest priority. A private room is preferable, but not absolutely necessary.

A patient diagnosed with a pituitary adenoma has arrived on the neurologic unit. When planning the patient's care, the nurse should be aware that the effects of the tumor will primarily depend on what variable? A)Whether the tumor utilizes aerobic or anaerobic respiration B)The specific hormones secreted by the tumor C)The patient's pre-existing health status D)Whether the tumor is primary or the result of metastasis

Ans: B)The specific hormones secreted by the tumor Feedback: Functioning pituitary tumors can produce one or more hormones normally produced by the anterior pituitary and the effects of the tumor depend largely on the identity of these hormones. This variable is more significant than the patient's health status or whether the tumor is primary versus secondary. Anaerobic and aerobic respiration is not relevant.

A patient is being admitted to the neurologic ICU with suspected herpes simplex virus encephalitis. What nursing action best addresses the patients complaints of headache? A) Initiating a patient-controlled analgesia (PCA) of morphine sulfate B) Administering hydromorphone (Dilaudid) IV as needed C) Dimming the lights and reducing stimulation D) Distracting the patient with activity

Ans: C Feedback: Comfort measures to reduce headache include dimming the lights, limiting noise and visitors, grouping nursing interventions, and administering analgesic agents. Opioid analgesic medications may mask neurologic symptoms, therefore, they are used cautiously. Non-opioid analgesics may be preferred. Distraction is unlikely to be effective, and may exacerbate the patients pain.

The nurse is discharging home a patient who suffered a stroke. He has a flaccid right arm and leg and is experiencing problems with urinary incontinence. The nurse makes a referral to a home health nurse because of an awareness of what common patient response to a change in body image? A) Denial B) Fear C) Depression D) Disassociation

Ans: C Feedback: Depression is a common and serious problem in the patient who has had a stroke. It can result from a profound disruption in his or her life and changes in total function, leaving the patient with a loss of independence. The nurse needs to encourage the patient to verbalize feelings to assess the effect of the stroke on self-esteem. Denial, fear, and disassociation are not the most common patient response to a change in body image, although each can occur in some patients.

4. A patient with Guillain-Barr syndrome has experienced a sharp decline in vital capacity. What is the nurses most appropriate action? A) Administer bronchodilators as ordered. B) Remind the patient of the importance of deep breathing and coughing exercises. C) Prepare to assist with intubation. D) Administer supplementary oxygen by nasal cannula.

Ans: C Feedback: For the patient with Guillain-Barr syndrome, mechanical ventilation is required if the vital capacity falls, making spontaneous breathing impossible and tissue oxygenation inadequate. Each of the other listed actions is likely insufficient to meet the patients oxygenation needs.

The nurse is reviewing the medication administration record of a female patient who possesses numerous risk factors for stroke. Which of the womans medications carries the greatest potential for reducing her risk of stroke? A) Naproxen 250 PO b.i.d. B) Calcium carbonate 1,000 mg PO b.i.d. C) Aspirin 81 mg PO o.d. D) Lorazepam 1 mg SL b.i.d. PRN

Ans: C Feedback: Research findings suggest that low-dose aspirin may lower the risk of stroke in women who are at risk. Naproxen, lorazepam, and calcium supplements do not have this effect.

The pathophysiology of an ischemic stroke involves the ischemic cascade, which includes the following steps: 1. Change in pH 2. Blood flow decreases 3. A switch to anaerobic respiration 4. Membrane pumps fail 5. Cells cease to function 6. Lactic acid is generated Put these steps in order in which they occur. A) 635241 B) 352416 C) 236145 D) 162534

Ans: C Feedback: The ischemic cascade begins when cerebral blood flow decreases to less than 25 mL per 100 g of blood per minute. At this point, neurons are no longer able to maintain aerobic respiration. The mitochondria must then switch to anaerobic respiration, which generates large amounts of lactic acid, causing a change in the pH. This switch to the less efficient anaerobic respiration also renders the neuron incapable of producing sufficient quantities of adenosine triphosphate (ATP) to fuel the depolarization processes. The membrane pumps that maintain electrolyte balances begin to fail, and the cells cease to function.

The nurse is preparing health education for a patient who is being discharged after hospitalization for a hemorrhagic stroke. What content should the nurse include in this education? A) Mild, intermittent seizures can be expected. B) Take ibuprofen for complaints of a serious headache. C) Take antihypertensive medication as ordered. D) Drowsiness is normal for the first week after discharge.

Ans: C Feedback: The patient and family are provided with information that will enable them to cooperate with the care and restrictions required during the acute phase of hemorrhagic stroke and to prepare the patient to return home. Patient and family teaching includes information about the causes of hemorrhagic stroke and its possible consequences. Symptoms of hydrocephalus include gradual onset of drowsiness and behavioral changes. Hypertension is the most serious risk factor, suggesting that appropriate antihypertensive treatment is essential for a patient being discharged. Seizure activity is not normal, complaints of a serious headache should be reported to the physician before any medication is taken. Drowsiness is not normal or expected.

A patient with a brain tumor has begun to exhibit signs of cachexia. What subsequent assessment should the nurse prioritize? A)Assessment of peripheral nervous function B)Assessment of cranial nerve function C)Assessment of nutritional status D)Assessment of respiratory status

Ans: C) Assessment of nutritional status Feedback: Cachexia is a wasting syndrome of weight loss, muscle atrophy, fatigue, weakness, and significant loss of appetite. Consequently, nutritional assessment is paramount.

A patient diagnosed with a cerebral aneurysm reports a severe headache to the nurse. What action is a priority for the nurse? A) Sit with the patient for a few minutes. B) Administer an analgesic. C) Inform the nurse-manager. D) Call the physician immediately.

Ans: D Feedback: A headache may be an indication that the aneurysm is leaking. The nurse should notify the physician immediately. The physician will decide whether administration of an analgesic is indicated. Informing the nurse-manager is not necessary. Sitting with the patient is appropriate, once the physician has been notified of the change in the patients condition.

A nurse in the ICU is providing care for a patient who has been admitted with a hemorrhagic stroke. The nurse is performing frequent neurologic assessments and observes that the patient is becoming progressively more drowsy over the course of the day. What is the nurses best response to this assessment finding? A) Report this finding to the physician as an indication of decreased metabolism. B) Provide more stimulation to the patient and monitor the patient closely. C) Recognize this as the expected clinical course of a hemorrhagic stroke. D) Report this to the physician as a possible sign of clinical deterioration.

Ans: D Feedback: Alteration in LOC often is the earliest sign of deterioration in a patient with a hemorrhagic stroke. Drowsiness and slight slurring of speech may be early signs that the LOC is deteriorating. This finding is unlikely to be the result of metabolic changes and it is not expected. Stimulating a patient with an acute stroke is usually contraindicated.

The nurse caring for a patient in ICU diagnosed with Guillain-Barr syndrome should prioritize monitoring for what potential complication? A) Impaired skin integrity B) Cognitive deficits C) Hemorrhage D) Autonomic dysfunction

Ans: D Feedback: Based on the assessment data, potential complications that may develop include respiratory failure and autonomic dysfunction. Skin breakdown, decreased cognition, and hemorrhage are not complications of Guillain-Barr syndrome.

What should be included in the patients care plan when establishing an exercise program for a patient affected by a stroke? A) Schedule passive range of motion every other day. B) Keep activity limited, as the patient may be over stimulated. C) Have the patient perform active range-of-motion (ROM) exercises once a day. D) Exercise the affected extremities passively four or five times a day.

Ans: D Feedback: The affected extremities are exercised passively and put through a full ROM four or five times a day to maintain joint mobility, regain motor control, prevent development of a contracture in the paralyzed extremity, prevent further deterioration of the neuromuscular system, and enhance circulation. Active ROM exercises should ideally be performed more than once per day.

A female patient is diagnosed with a right-sided stroke. The patient is now experiencing hemianopsia. How might the nurse help the patient manage her potential sensory and perceptional difficulties? A) Keep the lighting in the patients room low. B) Place the patients clock on the affected side. C) Approach the patient on the side where vision is impaired. D) Place the patients extremities where she can see them.

Ans: D Feedback: The patient with homonymous hemianopsia (loss of half of the visual field) turns away from the affected side of the body and tends to neglect that side and the space on that side, this is called amorphosynthesis. In such instances, the patient cannot see food on half of the tray, and only half of the room is visible. It is important for the nurse to remind the patient constantly of the other side of the body, to maintain alignment of the extremities, and if possible, to place the extremities where the patient can see them. Patients with a decreased field of vision should be approached on the side where visual perception is intact. All visual stimuli (clock, calendar, and television) should be placed on this side. The patient can be taught to turn the head in the direction of the defective visual field to compensate for this loss. Increasing the natural or artificial lighting in the room and providing eyeglasses are important in increasing vision. There is no reason to keep the lights dim.

While assessing the patient at the beginning of the shift, the nurse inspects a surgical dressing covering the operative site after the patients' cervical diskectomy. The nurse notes that the drainage is 75% saturated with serosanguineous discharge. What is the nurse's most appropriate action? A)Page the physician and report this sign of infection. B)Reinforce the dressing and reassess in 1 to 2 hours. C)Reposition the patient to prevent further hemorrhage. D)Inform the surgeon of the possibility of a dural leak.

Ans: D)Inform the surgeon of the possibility of a dural leak. Feedback: After a cervical diskectomy, the nurse will monitor the operative site and dressing covering this site. Serosanguineous drainage may indicate a dural leak. This constitutes a risk for meningitis, but is not a direct sign of infection. This should be reported to the surgeon, not just reinforced and observed.

A patient, diagnosed with cancer of the lung, has just been told he has metastases to the brain. What change in health status would the nurse attribute to the patient's metastatic brain disease? A)Chronic pain B)Respiratory distress C)Fixed pupils D)Personality changes

Ans: D)Personality changes Feedback: Neurologic signs and symptoms include headache, gait disturbances, visual impairment, personality changes, altered mentation (memory loss and confusion), focal weakness, paralysis, aphasia, and seizures. Pain, respiratory distress, and fixed pupils are not among the more common neurologic signs and symptoms of metastatic brain disease.

The nurse is demonstrating how to perform punctal occlusion. Which activities does the nurse perform? Applies firm pressure to the upper and lower eyelids at the outer edges to keep eyelids in approximation Applies gentle pressure bilaterally on the bridge of the nose to the inner canthus of each eye Applies gentle pressure to the upper eyelid to keep the lid open while telling the client to gaze upward Holds down the lower lid of the eye by applying pressure on the eyeball and the cheekbone

Applies gentle pressure bilaterally on the bridge of the nose to the inner canthus of each eye Punctal occlusion is done by applying gentle pressure to the inner canthus of each eye for 1 to 2 minutes immediately after eye drops are instilled. The nurse does not apply pressure to the eyeball when administering medications. The lower eyelid is held down to expose the conjunctival sac. The other action described will not aid in the retention or absorption of medication.

A client who is blind is awaiting elective surgery. What should the nurse do to promote this client's control over their hospital environment? Open all containers without prompting to be helpful. Keep personal care items where the nurse knows their location. Arrange the meal tray in a way that is easiest for the nurse to assist the client. Ask the client where to store his or her self-care items.

Ask the client where to store his or her self-care items. Ask the client's preference for where to store hygiene articles and other objects needed for self-care. Involving the client promotes his or her control over the environment. Personal care items should be kept in the same location at all times to provide the client with the ability to locate toiletries easily. At mealtime, describe where food is on the plate using the positions on the face of a clock. This measure assists the client to identify the location of food. Allow the client to open containers and offer help if needed. Having a choice facilitates independence.

Following cataract removal, discharge instructions will be provided to the client. Which of the following instructions is most important? Only sleep on back. Avoid any activity that can increase intraocular pressure. Avoid washing face and eyes for first 24 hours. Apply protective patch to both eyes at bedtime.

Avoid any activity that can increase intraocular pressure. For approximately 1 week, the client should avoid any activity that can cause an increase in intraocular pressure. Clients may sleep on back or unaffected side. Clients may use a clean damp cloth to remove eye discharge and wash face. An eye shield is often ordered for the first 24 hours and during the night to prevent rubbing or trauma to the operative eye.

An 80-year-old woman has just undergone cataract surgery at an outpatient surgical center and is being prepared for discharge home. When providing relevant health education, the nurse should teach the patient to: Remain on bed rest for the next 2 to 3 days, if possible Irrigate her eye with normal saline solution every 4 to 6 hours Avoid bending over for an extended period until the eye has healed Expect significant pain for the next 24 to 48 hours

Avoid bending over for an extended period until the eye has healed After cataract surgery, the patient should avoid bending or stooping for an extended period. Bed rest is not necessary, and irrigation is contraindicated. Pain is an abnormal finding that would warrant prompt follow-up.

A client has been in a motor vehicle collision. Radiographs indicate a fractured humerus; the client is awaiting the casting of the upper extremity and admission to the orthopedic unit. What is the primary treatment for musculoskeletal trauma? A external rotation B immobilization C surgical repair D enhancing complications

B

The nurse is caring for a patient who sustained an open fracture of the right femur in an automobile accident. What does the nurse understand is the most serious complication of an open fracture? A Muscle atrophy caused by loss of supporting bone structure B Infection C Nerve damage D Necrosis of adjacent soft tissue caused by blood loss

B

When the client who has experienced trauma to an extremity reports severe burning pain, vasomotor changes, and muscles spasms in the injured extremity, the nurse recognizes that the client is likely demonstrating signs of A heterotrophic ossification. B complex regional pain syndrome. C avascular necrosis of bone. D a reaction to an internal fixation device.

B

Which nursing diagnosis takes highest priority for a client with a compound fracture? A Imbalanced nutrition: Less than body requirements related to immobility B Risk for infection related to effects of trauma C Impaired physical mobility related to trauma D Activity intolerance related to weight-bearing limitations

B

A client with metastatic bone cancer sustained a left hip fracture without injury. What type of fracture does the nurse understand occurs without trauma or fall? A Impacted fracture B Pathologic fracture C Compound fracture D Transverse fracture

B A pathologic fracture is a fracture that occurs through an area of diseased bone and can occur without trauma or a fall. An impacted fracture is a fracture in which a bone fragment is driven into another bone fragment. A transverse fracture is a fracture straight across the bone. A compound fracture is a fracture in which damage also involves the skin or mucous membranes.

A client who has sustained a fracture reports an increase in pain and decreased function of the affected extremity. What will the nurse suspect? A Infection B Avascular necrosis C Hypovolemic shock D Pulmonary embolism

B Avascular necrosis refers to the death of the bone from insufficient blood supply, typically manifested by complaints of increased pain and decreased function. Fever or redness, purulent drainage, and swelling of the site would suggest infection. Respiratory distress would suggest a pulmonary embolism. Changes in vital signs, level of consciousness, and signs and symptoms of fluid loss would suggest hypovolemic shock.

A client who suffers an injury in a local high school hockey game presents with left shoulder pain. The client cannot move the left arm, and the left shoulder is lower than the right shoulder. The nurse recognizes the client most likely has a: A Cervical injury B Dislocated shoulder C Clavicle fracture D Dislocated elbow

B Clinical manifestations of a dislocated shoulder include pain, lack of motion, feeling of an empty shoulder socket, and uneven posture

Two days after application of a cast to treat a fractured femur, the client reports severe, deep, and constant pain in the leg. What willl the nurse suspect? A Chronic venous insufficiency. B Compartment syndrome. C Phlebitis. D Infection.

B Compartment syndrome refers to the compression of nerves, blood vessels, and muscle within a closed space. This leads to tissue death from lack of oxygenation

A nurse suspects that a client with a recent fracture has compartment syndrome. Assessment findings may include: A body-wide decrease in bone mass. B inability to perform active movement and pain with passive movement. C inability to perform passive movement and pain with active movement. D a growth in and around the bone tissue.

B With compartment syndrome, the client can't perform active movement, and pain occurs with passive movement. A body-wide decrease in bone mass is seen in osteoporosis. A growth in and around the bone tissue may indicate a bone tumor.

A patient's brain tumor has been manifested by seizures that have become more frequent and severe in recent days. The patient has been scheduled for neurosurgery but the nurse is nonetheless making changes to the patient's preoperative nursing care plan. The patient's seizures should cause the nurse to prioritize which of the following nursing diagnoses? A) Acute pain related to seizure activity B) Risk for injury related to seizures C) Impaired memory related to seizure activity D) Imbalanced nutrition: less than body requirements related to seizures

B) Risk for injury related to seizures

A 77-year-old man is receiving care in the neurological intensive care unit following an ischemic stroke. The patient is breathing spontaneously but has not yet regained consciousness. How should the nurses in the neuro ICU best position this patient? A) In the high Fowler's position with a pillow under his knees B) Side-lying with the head of the bed slightly elevated C) Supine with pillows supporting his extremities D) In the semi-Fowler's position

B) Side-lying with the head of the bed slightly elevated

Disturbed by the high incidence and poor outcomes of stroke in the community, a public health nurse is planning a health promotion campaign that is specifically focused on stroke. Which of the following proposed outcomes would most directly address an identified public awareness need? A)"Participants will describe the factors that affect cerebral blood flow." B)"Participants will state the most common signs and symptoms of stroke." C) "Participants will state the common treatment modalities for different types of stroke." D) "Participants will describe the relationship between psychological stress and stroke."

B)"Participants will state the most common signs and symptoms of stroke."

A patient who has been diagnosed with a brain tumor is being cared for in the neurological intensive care unit (ICU) because of a sudden exacerbation of the signs and symptoms of his neoplasm. Which of the following would be indicative of increased intracranial pressure (ICP)? A)Epistaxis B)Severe headache C)Increased jugular venous pressure (JVP) D)Electrocardiographic (ECG) changes

B)Severe headache

A nurse instructs a client to refrain from blinking after administering eye drops based on which rationale? Blinking causes the eye drop to be expelled from the conjunctival sac. Blinking slows absorption of the instilled eye drops. Blinking limits the size of the conjunctival sac for the needed amount of eye drop. Blinking keeps substances from entering the eye.

Blinking causes the eye drop to be expelled from the conjunctival sac. Blinking expels an instilled eye drop from the conjunctival sac, which interferes with the efficacy of the medication. Blood-ocular barriers keep foreign substances from entering the eye. The size of the conjunctival sac does change with blinking. It can hold only 50 uL.

An ophthalmologist diagnoses a patient with myopia. The nurse explains that this type of impaired vision is a refractive error characterized by: Eyes that are shallow. A shorter depth to the eyeball. Farsightedness. Blurred distance vision.

Blurred distance vision. People who have myopia are said to be nearsighted. They have deeper eyeballs; thus, the distant visual image focuses in front of, or short of, the retina. Myopic people experience blurred distance vision.

A client who had a corneal transplant a few months ago arrives at the emergency department reporting eye discomfort. When assessing the client, which of the following would lead the nurse to suspect graft failure? Halos around lights Blurred vision Reduced tearing Pale conjunctiva

Blurred vision Signs and symptoms of graft failure include eye discomfort, blurred vision, tearing, and redness of the eye. Halos around lights are associated with glaucoma.

The nurse is administering an ophthalmic ointment to a patient with conjunctivitis. What disadvantage of the application of an ointment does the nurse explain to the patient? It does not work as rapidly as eye drops do. It has more side effects than eye drops. It has a lower concentration than eye drops. Blurred vision results after application.

Blurred vision results after application. Ophthalmic ointments have extended retention time in the conjunctival sac and provide a higher concentration than eye drops. The major disadvantage of ointments is the blurred vision that results after application. In general, eyelids and eyelid margins are best treated with ointments.

The nurse is preparing to discharge a patient home. During discharge teaching, the nurse realizes that the patient is not able to read the medication bottles and has not been taking her medications at home. How should the nurse intervene appropriately in this situation? By informing the patient that she will need to go to a rehabilitation facility for vision training By asking the social worker to investigate nursing home placement By informing that patient that surgery will correct the vision problem By asking the social worker to investigate community support agencies

By asking the social worker to investigate community support agencies Managing low vision involves magnification and image enhancement through the use of low-vision aids and strategies and referrals to social services and community agencies serving those with visual impairment. Community agencies, such as The Lighthouse National Center for Vision and Aging, offer services to low vision patients that include training in independent living skills and a variety of assistive devices for vision enhancement, and orientation and mobility to keep patients from needing to enter a nursing facility. A rehabilitation facility is generally not needed by patients to learn to use assistive devices or gain a greater degree of independence. Surgery is also generally not a necessary component to independent living for this group of patients.

A client reports pain in the right knee, stating, "My knee got twisted when I was going down the stairs." The client was diagnosed with an injury to the ligaments and tendons of the right knee. Which terminology, documented by the nurse, best reflects the injury? A Subluxation B Strain C Sprain D Dislocation

C

Which of the following musculoskeletal injuries is manifested by acromioclavicular joint pain? A Acute compartment syndrome B Heterotopic ossification C Rotator cuff tears D Epicondylitis

C

Which type of fracture involves a break through only part of the cross-section of the bone? A Comminuted B Open C Incomplete D Oblique

C

The nurse is assessing a patient's right knee, and the assessment reveals edema, tenderness, muscle spasms, and ecchymosis. The patient states that 2 days ago he ran a half marathon and now it is painful to stand up. Based upon these symptoms, the nurse should plan care based upon the fact that the patient has likely experienced what? A. 1st degree strain B. 1st degree sprain C. 2nd degree strain D. 2nd degree sprain

C A second-degree strain involves tearing of muscle fibers and is manifested by notable loss of load-bearing strength with accompanying edema, tenderness, muscle spasm, and ecchymosis. A first-degree strain reflects tearing of few muscle fibers and is accompanied by minor edema, tenderness, and mild muscle spasm, without noticeable loss of function

When is it advisable for the nurse to apply heat to a sprain or a contusion? A Only after a week B Do not apply at all C After 2 days D Immediately

C It is advisable to apply heat on a sprain or a contusion 2 days after a sprain or a contusion has occurred. This is because after 2 days swelling is not likely to increase and as a result heat application reduces pain and relieves local edema by improving circulation. Delaying the application of heat prolongs the pain and increased the risk of local edema

A patient has sustained a long bone fracture. The nurse is preparing a care plan for this patient. Which nursing action should the nurse include in the care plan to enhance fracture healing? A Administer high doses of corticosteroids. B Avoid prolonged immobilization of the fracture fragments. C Monitor color, temperature, and pulses of the affected extremity. D Limit weight-bearing and exercising during the recovery.

C The nurse should include monitoring for sufficient blood supply by assessing the color, temperature, and pulses of the affected extremity as adequate blood supply enhances the healing of a fracture. Factors that inhibit fracture healing include inadequate or lack of immobilization of the fracture fragments and administration of corticosteroids. Weight-bearing exercises are encouraged for patients with long bone fracture

A patient's recent stroke has had a profound effect on his communication. To foster an atmosphere that is conducive to communication and the recovery of speech the nurse should: A)Speak to the patient using simplified vocabulary and short sentences. B) Disregard the patient's gestures in an effort to encourage speech. C) Adhere to predictable schedules whenever possible. D) Ensure that the patient receives constant stimulation during daytime hours.

C) Adhere to predictable schedules whenever possible.

20. A community health nurse is conducting a home visit to a male patient who has Parkinson's disease (PD). The nurse has been working closely with the patient and his wife to prevent the many possible complications of the disease. What recommendation should the nurse make to improve nutrition? A) Have the patient drink nutritional supplements in lieu of solid food B) Replace complex carbohydrates with simple carbohydrates C) Allot a large amount of time for each meal of the day D) Have the patient drink fluids before and after meals but not during meals

C) Allot a large amount of time for each meal of the day - Eating becomes a very slow process, requiring concentration due to a dry mouth from medications and difficulty chewing and swallowing. An electric warming tray keeps food hot and allows the patient to rest during the prolonged time that it may take to eat.

The nurse educator is discussing neoplasms with a group of recent graduates. The educator explains that the effects of neoplasms are caused by the compression and infiltration of normal tissue. The physiologic changes that result can cause what pathophysiologic events? Select all that apply. A)Intracranial hemorrhage B)Infection of cerebrospinal fluid C)Increased ICP D)Focal neurologic signs E)Altered pituitary function

C)Increased ICP D)Focal neurologic signs E)Altered pituitary function Feedback: The effects of neoplasms are caused by the compression and infiltration of tissue. A variety of physiologic changes result, causing any or all of the following pathophysiologic events: increased ICP and cerebral edema, seizure activity and focal neurologic signs, hydrocephalus, and altered pituitary function.

A patient is given atropine to dilate the pupil of her right eye to treat an acute iritis. The nurse is aware that a major consideration is to: Assess for cardiac dysrhythmias. Observe for signs of disorientation or delirium. Check for signs and symptoms of glaucoma. Monitor the patient for indicators of hypertension.

Check for signs and symptoms of glaucoma. The nurse should watch the patient for signs and symptoms of glaucoma. Refer to Table 49-4 in the text.

Which of the following medication classifications increases aqueous fluid outflow in the patient with glaucoma? Alpha-adrenergic agonists Beta blockers Cholinergics Carbonic anhydrase inhibitors

Cholinergics Cholinergics increase aqueous fluid outflow by contracting the ciliary muscle, causing miosis and opening the trabecular meshwork. Beta blockers decrease aqueous humor production. Alpha-adrenergic agonists decrease aqueous humor production. Carbonic anhydrase inhibitors decrease aqueous humor production.

A client with rheumatoid arthritis arrives at the clinic for a checkup. Which statement by the client refers to the most overt clinical manifestation of rheumatoid arthritis? "My legs feel weak." "My finger joints are oddly shaped." "I have pain in my hands." "I have trouble with my balance."

Correct response: "My finger joints are oddly shaped." Explanation: Joint abnormalities are the most obvious manifestations of rheumatoid arthritis. A systemic disease, rheumatoid arthritis attacks all connective tissue. Although muscle weakness may occur from limited use of the joint where the muscle attaches, such weakness isn't the most obvious sign of rheumatoid arthritis; also, it occurs only after joint abnormalities arise. Subcutaneous nodules in the hands, although common in rheumatoid arthritis, are painless. The disease may cause gait disturbances, but these follow joint abnormalities.

A client asks the nurse what the difference is between osteoarthritis (OA) and rheumatoid arthritis (RA). Which response is correct? A. "OA is a noninflammatory joint disease. RA is characterized by inflamed, swollen joints." B. "OA and RA are very similar. OA affects the smaller joints and RA affects the larger, weight-bearing joints." C. "OA affects joints on both sides of the body. RA is usually unilateral." D. "OA is more common in women. RA is more common in men."

Correct response: "OA is a noninflammatory joint disease. RA is characterized by inflamed, swollen joints." Explanation: OA is a degenerative arthritis, characterized by the loss of cartilage on the articular surfaces of weight-bearing joints with spur development. RA is characterized by inflammation of synovial membranes and surrounding structures. OA may occur in one hip or knee and not the other, whereas RA commonly affects the same joints bilaterally. RA is more common in women; OA affects both sexes equally. Aging is the risk factor most strongly correlated with OA.

A client with osteoarthritis tells the nurse she is concerned that the disease will prevent her from doing her chores. Which suggestion should the nurse offer?

Correct response: "Pace yourself and rest frequently, especially after activities." Explanation: A client with osteoarthritis must adapt to this chronic and disabling disease, which causes deterioration of the joint cartilage. The most common symptom of the disease is deep, aching joint pain, particularly in the morning and after exercise and weight-bearing activities. Because rest usually relieves the pain, the nurse should instruct the client to rest frequently, especially after activities, and to pace herself during daily activities. Telling the client to do her chores in the morning is incorrect because the pain and stiffness of osteoarthritis are most pronounced in the morning. Telling the client to do all chores after performing morning exercises or in the evening is incorrect because the client should pace herself and take frequent rests rather than doing all chores at once.

A client diagnosed with arthritis doesn't want to take medications. Physical therapy and occupational therapy have been consulted for nonpharmacologic measures to control pain. What might physical and occupational therapy include in the care plan to help control this client's pain? Acupuncture An exercise routine that includes range-of-motion (ROM) exercises Heat therapy and nonsteroidal anti-inflammatory medications (NSAIDs) Cold therapy

Correct response: An exercise routine that includes range-of-motion (ROM) exercises Explanation: Physical and occupational therapy will most likely develop an exercise routine that includes ROM exercises to control the client's pain. Acupuncture may help relieve the client's pain; however, it isn't within the scope of practice for physical and occupational therapists. Heat therapy may help the client, but it's coupled with NSAIDs in this option, which goes against the client's wishes. Cold therapy aggravates joint stiffness and causes pain.

Which of the following terms refers to fixation or immobility of a joint? Hemarthrosis Diarthrodial Arthroplasty Ankylosis

Correct response: Ankylosis Explanation: Ankylosis may result from disease or scarring due to trauma. Hemarthrosis refers to bleeding into the joint. Diarthrodial refers to a joint with two free moving parts. Arthroplasty refers to replacement of a joint.

What is the priority intervention for a patient who has been admitted repeatedly with attacks of gout?

Correct response: Assess dietary diet and activity at home Explanation: Patients with gout need teaching about diet restrictions in order to prevent repeated attacks. Foods high in purine need to be avoided, and alcohol intake has to be limited. Stressful activities should also be avoided. The nurse should assess to see what is stimulating the repeated attacks of gout. The other interventions are not appropriate for a patient with this problem.

A client is receiving treatment for an acute episode of gout with colchicine. The nurse is administering the medication every 2 hours. What should the nurse be sure the client communicates so that the drug can be temporarily stopped? Select all that apply.

Correct response: Diarrhea Intestinal cramping Nausea and vomiting Explanation: Colchicine is administered every 1 or 2 hours until the pain subsides or nausea, vomiting, intestinal cramping, and diarrhea develop. When one or more of these symptoms occurs, the drug should be stopped temporarily. Tingling in the arms and increase in pain are not normal adverse reactions that are seen with this drug.

The nurse teaches the patient that the presence of crystals in his or her synovial fluid obtained from an arthrocentesis confirms which disease process? Gout Infection Inflammation Degeneration

Correct response: Gout Explanation: The presence of crystals is indicative of gout, and the presence of bacteria is indicative of infective arthritis. Gout is caused by hyperuricemia (increased serum uric acid).

Which findings best correlate with a diagnosis of osteoarthritis? A.Joint stiffness that decreases with activity B.Erythema and edema over the affected joint C.Anorexia and weight loss D.Fever and malaise

Correct response: Joint stiffness that decreases with activity Explanation: A characteristic feature of osteoarthritis (degenerative joint disease) is joint stiffness that decreases with activity and movement. Erythema and edema over the affected joint, anorexia, weight loss, and fever and malaise are associated with rheumatoid arthritis, a more severe and destructive form of arthritis.

A nursing student asks the instructor how to identify rheumatoid nodules in a client with rheumatoid arthritis. Which of the following characteristics would the instructor include? Tender to touch Reddened Nonmovable Located over bony prominence

Correct response: Located over bony prominence Explanation: Rheumatoid nodules usually are nontender, movable, and evident over bony prominences, such as the elbow or the base of the spine. They are not reddened.

A client with rheumatoid arthritis has experienced increasing pain and progressing inflammation of the hands and feet. What would be the expected goal of the likely prescribed treatment regimen? A.Minimizing damage B. Eradicating pain C.Eliminating deformities D.Promoting sleep

Correct response: Minimizing damage Explanation: Although RA cannot be cured, much can be done to minimize damage. Treatment goals include decreasing joint inflammation before bony ankylosis occurs, relieving discomfort, preventing or correcting deformities, and maintaining or restoring function of affected structures. Early treatment leads to the best results.

Which of the following are usually the first choice in the treatment of rheumatoid arthritis (RA)? A. Nonsteroidal anti-inflammatory drugs (NSAIDs) B .Disease-modifying antirheumatic drugs (DMARDS) C. Tumor necrosis factor (TNF) blockers D. Glucocorticoids

Correct response: Nonsteroidal anti-inflammatory drugs (NSAIDs) Explanation: In most patients NSAIDs usually are the first choice in the treatment of RA. The use of traditional NSAIDs and salicylates inhibit the production of prostaglandins and provide anti-inflammatory effects as well as analgesic. In RA, if joint symptoms persist despite use of NSAIDs, the second major drug group known as DMARDs is initiated early in the disease. TNF blockers interfere with the action of tumor necrosis factor (TNF). Oral glucocorticoids, such as prednisone and prednisolone, are indicated for patients with generalized symptoms.

Which of the following is the leading cause of disability and pain in the elderly? A. Osteoarthritis B.Rheumatoid arthritis (RA) C.Systemic lupus erythematous (SLE) D.Scleroderma

Correct response: Osteoarthritis Explanation: Osteoarthritis is the leading cause of disability and pain in the elderly. RA, SLE, and scleroderma are not leading causes of disability and pain in the elderly. A client with osteoarthritis has increased stiffness in the morning upon awakening. Exercise should be scheduled at least 1 hour after awakening. Exercising in the evening interferes with the client's ability to rest at bedtime.

Nursing care for the patient with fibromyalgia should be guided by what assumption?

Correct response: Patients with fibromyalgia may feel as if their symptoms are not taken seriously. Explanation: Because patients present with widespread symptoms that are often vague in nature, health care providers may misdiagnose patients. Patients feel as though people are not listening to them. Nurses need to provide support and encouragement. Symptoms of disease vary from patient to patient, and respond to different treatments. Patients do not lose their ability to walk.

Which diagnostic study is decreased in patient diagnosed with rheumatoid arthritis? ESR Red blood cell count Uric acid Creatinine

Correct response: Red blood cell count Explanation: There is a decreased red blood cell count in patients diagnosed with rheumatic diseases. EDR increases inflammatory connective tissue disease. Uric acid is increased in gout. Increased creatinine may indicate renal damage in SLE, scleroderma, and polyarteritis.

A nurse is poviding care for a client with progressive systemic sclerosis. For a client with this disease, the nurse is most likely to formulate which nursing diagnosis? A.Risk for impaired skin integrity B.Constipation C.Ineffective thermoregulation D.Risk for imbalanced nutrition: More than body requirements

Correct response: Risk for impaired skin integrity Explanation: Progressive systemic sclerosis is a connective tissue disease characterized by fibrosis and degenerative changes of the skin, synovial membranes, and digital arteries. Therefore, the nurse is most likely to formulate a nursing diagnosis of Risk for impaired skin integrity. Because clients with the disease are prone to diarrhea from GI tract hypermotility (caused by pathologic changes), Constipation is an unlikely nursing diagnosis. Progressive systemic sclerosis doesn't cause Ineffective thermoregulation. GI hypermotility may lead to malabsorption, and esophageal dysfunction may cause dysphagia; these conditions put the client with the disease at risk for inadequate nutrition, making Risk for imbalanced nutrition: More than body requirements an improbable nursing diagnosis.

After teaching a group of students about systemic lupus erythematosus, the instructor determines that the teaching was successful when the students state which of the following?

Correct response: The belief is that it is an autoimmune disorder with an unknown trigger. Explanation: Systemic lupus erythematosus is believed to be an autoimmune disorder but the triggering mechanism is not known. The disorder is more common in women than in men, most with the disorder in the 3rd or 4th decade of life. The disease is considered the "great imitator" because the clinical signs resemble many other conditions. SLE is a diffuse connective tissue disease that affects multiple body systems. Sunlight tends to exacerbate the disease. Because fatigue is a major issue, allow for adequate rest, along with regular activity to promote mobility and prevent joint stiffness. Maintain a well-balanced diet and increase fluid intake to raise energy levels and promote tissue healing.

A nurse is assessing a client with possible osteoarthritis. The most significant risk factor for primary osteoarthritis is: congenital deformity. age. trauma. obesity.

Correct response: age. Explanation: Age is the most significant risk factor for developing primary osteoarthritis. Development of primary osteoarthritis is influenced by genetic, metabolic, mechanical, and chemical factors. Secondary osteoarthritis usually has identifiable precipitating events such as trauma.

A client who has a pituitary adenoma would report which symptoms related to the presence of this type of tumor? Select all that apply. - Fever - Chiasmal syndrome - Morning headaches - Anorexia - Polydipsia

Correct response: Morning headaches Chiasmal syndrome Polydipsia Anorexia Explanation: Common symptoms reported in association with the diagnosis of a pituitary adenoma include headaches in the morning, and changes in the visual field resulting from pressure on the optic nerves, optic chiasm and optic tracts. It is the pressure on the optic chiasm that can lead to a condition called chiasmal syndrome, which is correlated with pituitary adenomas. Polydipsia is just one of the symptoms of diabetes insipidus that accompanies the presence of this type of tumor. Low appetite resulting from the pressure of the tumor can lead to anorexia. Fever is not associated with the presence of a pituitary adenoma. This finding may be associated with other serious neurological conditions if accompanied by the same symptoms (i.e., headaches, visual impairments). These conditions include meningitis or encephalitis.

A female patient has presented to the emergency department (ED) with complaints of a high fever and severe headache. The patient states that acetaminophen has had no appreciable effect on either symptom. The triage nurse recognizes the need to perform a rapid assessment for possible meningitis and should ask which of the following questions: - "Have you done any travelling in the last few weeks?" - "Have you had a nosebleed since this problem started?" - "Have you noticed any tremors in your hands or arms?" - "Are you having stiffness or pain in your neck-?

Correct response: "Are you having stiffness or pain in your neck?" Explanation: Nuchal rigidity is an early sign of meningitis that is seen in 30% to 70% of patients. Nosebleed and tremors are not characteristic of meningitis, and the disease is not commonly preceded by travel.

A patient has been undergoing rehabilitation for the past 3 weeks following a hemorrhagic stroke. The nurse has observed that the patient is motivated to perform his activities of daily living independently, but rarely attempts tasks with his affected arm. How should the nurse address this observation with the patient? - "You should be doing a lot more with your affected arm and a lot less with your strong arm." - "Even though it requires a huge effort, using your affected arm will help you build its strength." - "I've noticed that you're being very conscientious about protecting your affected arm, which is very important." - "If you don't push through the challenge of using your affected arm again it might never recover."

Correct response: "Even though it requires a huge effort, using your affected arm will help you build its strength." Explanation: The nurse must be sure that the patient does not neglect the affected side. However, this must be communicated in an empathic and nonthreatening way.

A 64-year-old woman has been admitted to the neurological unit after being diagnosed with a brain tumor. The woman is slated for neurosurgery the following day, and the woman's daughter has come to the hospital from out of state. After visiting her mother for the first time since she was diagnosed, the daughter is distraught and tells the nurse, "My mother has always been one of the kindest, most considerate people, and I don't remember her ever being so abrasive." How should the nurse best respond to the daughter's statement? - "It's most helpful if you can accept her the way she is now, even if it's more difficult than in the past." - "Your mother has received a very stressful diagnosis, and she's likely still processing it." - "Your mother may have been experiencing a partial seizure when she was speaking to you." - "Those personality changes are probably a result of the tumor's effect on her brain."

Correct response: "Those personality changes are probably a result of the tumor's effect on her brain." Explanation: Personality changes are a common manifestation of brain tumors and result from organic brain changes. Stress and seizures are less likely to underlie the behaviors the daughter noticed. It would be inappropriate to reprimand the daughter or to downplay her observations.

The nurse is caring for a patient newly diagnosed with a primary brain tumor. The patient asks the nurse where his tumor came from. What would be the nurse's best response? - "Your tumor is from nerve tissue somewhere in your body." - "Your tumor originated from cells and structures within the brain." - "Your tumor is pituitary in origin." - "Your tumor originated from somewhere outside the central nervous system."

Correct response: "Your tumor originated from cells and structures within the brain." Explanation: Primary brain tumors originate from cells and structures within the brain. Secondary brain tumors are metastatic tumors that originate somewhere else in the body. The scenario does not indicate that the patient's tumor is a pituitary tumor or a neuroma.

An older adult patient exhibiting clinical manifestations of a brain tumor is admitted to the hospital for testing. What tumor types does the nurse know are commonly seen in the older adult? - Anaplastic astrocytoma - Ependymoma - Glioblastoma - Cerebral metastasis from other sites - Medulloblastoma

Correct response: - Anaplastic astrocytoma - Cerebral metastasis from other sites - Glioblastoma Explanation: The most frequent tumor types in the older adult are anaplastic astrocytoma, glioblastoma, and cerebral metastases from other sites.

A patient is diagnosed with an intracerebral tumor. The nurse knows that the diagnosis may include which of the following? Select all that apply. - Ependymoma - Acoustic neuroma - Medulloblastoma - Astrocytoma - Meningioma

Correct response: - Astrocytoma - Ependymoma - Medulloblastoma Explanation: Glial tumors, the most common type of intracerebral brain neoplasm, are divided into many categories, including astrocytomas, ependymomas, and medulloblastomas. Meningiomas occur on the meninges and acoustic neuromas on the eight cranial nerve, and thus are not intracerebral tumors.

Corticosteroids are used in the treatment of brain tumors for which of the following clinical manifestations? Select all that apply. - Personality changes - Cerebral edema - Seizures - Headache - Altered level of consciousness

Correct response: - Cerebral edema - Headache - Altered level of consciousness Explanation: Corticosteroids are used during treatment to reduce cerebral edema and reduce side effects of treatment, such as nausea and vomiting. They are also helpful in relieving headache and alterations in level of consciousness. Antiseizure agents are used to treat seizures if they occur. Corticosteroids are not used for personality changes associated with brain tumors.

A client with a brain tumor is exhibiting papilledema. When reviewing the client's history, which information would the nurse correlate with this finding? Select all that apply. - Double vision - Aphasia - Swaying gait - Visual field deficit - Enhanced visual acuity

Correct response: - Double vision - Visual field deficit Explanation: Papilledema is associated with visual disturbances, such as decreased visual acuity, diplopia (double vision), and visual field deficits. It is not associated with a swaying gait or aphasia.

A client with an incurable brain tumor is experiencing nausea and vomiting and has little interest in eating. His family states, "We don't know how to help him." Which of the following would be appropriate for the nurse to suggest to help improve the client's nutritional intake? Select all that apply. - Place the client near the sounds and smells of meals being prepared. - Ensure that the client is free of pain for meals. - Plan meals for times when the client is rested. - Prepare the client for the insertion of a feeding tube. - Provide the client with foods that he likes.

Correct response: - Ensure that the client is free of pain for meals. - Plan meals for times when the client is rested. - Provide the client with foods that he likes. Explanation: Suggestions to improve nutrition include making sure that the client is comfortable, free of pain, and rested. This may require family members to adjust meal times. Additionally, they should eliminate offensive sights, sounds, and odors. Therefore, placing the client near sites of meal preparation may be too overwhelming. If the client has difficulty with or shows disinterest in usual foods, the family should offer foods that the client prefers, rather than attempting to get the client to eat as previously. If the client shows marked deterioration, then some other form of nutritional support such as a feeding tube or parenteral nutrition may be indicated, but only if this measure is consistent with the client's choices for care.

The nurse is conducting a preoperative assessment of a client who is scheduled for surgical removal of a primary spinal tumor. What should the nurse include in the preoperative session? Select all that apply. - Ensuring privacy of client information from family members - Methods of pain control after surgery - Question about current bowel and bladder control - Adjusting to changes in daily activities - Preparing to transition to palliative care

Correct response: - Methods of pain control after surgery - Adjusting to changes in daily activities - Question about current bowel and bladder control Explanation: The objectives of preoperative care include recognition of neurologic changes through ongoing assessments, pain control, and the management of altered activities of daily living. The nurse assesses for weakness, muscle wasting, spasticity, sensory changes, bowel and bladder dysfunction, and potential respiratory problems, especially if a cervical tumor is present. The client is also evaluated for coagulation deficiencies. Postoperative pain management strategies are discussed with the client before surgery. Discussing the transition to palliative care would only be part of an individualized plan of care where the client is already aware the surgical procedure is life-prolonging rather that curative. It would be inappropriate to have this discussion with the client at this time. Family members should be encouraged to be part of the preoperative discussions, because they will be providing care and supporting the client after the surgery. Anticipatory guidance should be offered to the family as much as possible, because they will need to be prepared for changes in the client's ability to perform activities of daily living.

A nurse begins an assessment interview of a patient with a brain tumor. The patient describes her headaches. The nurse expects to hear specific terms that describe her symptoms. Which of the following best describe the symptoms of her headache? Select all that apply. - Is made worse with coughing and sudden movement - May improve with vomiting - Is intermittent and diminishes with rest - Is unrelenting - Occurs most frequently in the early morning

Correct response: - Occurs most frequently in the early morning - Is unrelenting - Is made worse with coughing and sudden movement - May improve with vomiting Explanation: Characteristic indicators of an intracerebral headache are a headache that occurs in the early morning; is made worse by coughing, straining, or sudden movement; and may improve with vomiting. It is described as deep, dull, and unrelenting.

The nurse is caring for a client who is currently under medical investigation for a pituitary adenoma. The nurse anticipates the client will likely report which symptoms that are consistent with this type of brain tumor? Select all that apply. - Impairment of visual field - Seizures - Polyuria - Disturbed sleep - Polydipsia

Correct response: - Polydipsia - Polyuria - Disturbed sleep - Impairment of visual field Explanation: Pressure from a pituitary adenoma may be exerted on the optic nerves, optic chiasm, optic tracts, hypothalamus, or the third ventricle. Headache is a common symptom; there can also be visual dysfunction including loss of visual field, the development of diabetes insipidus including symptoms such as excessive thirst and urination. Sleep disturbances are reported and result from the development of diabetes insipidus. Seizures are a common finding with angioma brain tumors.

Which of the following are clinical manifestations associated with increased intracranial pressure (ICP)? Select all that apply. - Headache - Angina - Nausea with or without vomiting - Papilledema - Seizures

Correct response: - Seizures - Nausea with or without vomiting - Papilledema - Headache Explanation: Symptoms of increased intracranial pressure include headache, nausea with or without vomiting, and papilledema. Angina is not associated with increased ICP.

The nurse educator is testing a group of nursing students about various types of brain tumors and their clinical manifestations. The students are correct when stating tumors located in the cerebellar region of the brain produce which symptoms? Select all that apply. - Staggering gait - Visual hallucinations - Abnormal eye movements - Muscle incoordination - Apathetic mental attitude

Correct response: - Staggering gait - Muscle incoordination - Abnormal eye movements Explanation: A cerebellar tumor causes dizziness; an ataxic or staggering gait with a tendency to fall toward the side of the lesion; marked muscle incoordination; and nystagmus (involuntary rhythmic eye movements), usually in the horizontal direction. Visual hallucination are associated with occipital lobe tumors. An apathetic mental attitude can manifest from a tumor in the motor cortex of the frontal lobe.

What nursing intervention will best help the client with Huntington disease to increase nutrition? Select all that apply. - Take phenothiazine prior to meals - Increase high carbohydrate foods - Maintain a pureed diet - Use Relaxation techniques - Eliminate foods high in fat

Correct response: - Use Relaxation techniques - Take phenothiazine prior to meals Explanation: Talking to the client before meals will help to promote relaxation, and phenothiazines help to calm some clients. Eliminating foods high in fat, increasing carbohydrates, and pureeing food will not assist in relaxing muscles during choreiform movements. The nurse should wait for the client to chew and swallow, which can be a slow process.

The nurse is preparing to administer tissue plasminogen activator (t-PA) to a patient who weighs 132 lb. The order reads 0.9 mg/kg t-PA. The nurse understands that 10% of the calculated dose is administered as an IV bolus over 1 minute, and the remaining dose (90%) is administered IV over 1 hour via an infusion pump. How many milligrams IV bolus over 1 minute will the nurse initially administer?

Correct response: 5.4 Explanation: The patient is weighed to determine the dose of t-PA. Typically two or more IV sites are established prior to administration of t-PA (one for the t-PA and the other for administration of IV fluids). The dosage for t-PA is 0.9 mg/kg, with a maximum dose of 90 mg. Of the calculated dose, 10% is administered as an IV bolus over 1 minute. The remaining dose (90%) is administered IV over 1 hour via an infusion pump. First, the nurse must convert the patient's weight to kilograms (132/2.2 = 60 kg), then multiply 0.9 mg × 60 kg = 54 mg. Next, the nurse figure out that 10% of 54 mg is 5.4 (54 ×.10). The nurse will initially administer 5.4 mgs IV bolus over 1 minute.

A 37-year-old male is brought to the clinic by his wife because the patient is experiencing loss of motor function and sensation. After initial neurological assessment, the health care provider suspects the patient has a spinal cord tumor and hospitalizes him for diagnostic testing. In preparation for diagnostic studies, the nurse will inform the patient that the most commonly used study to diagnosis spinal cord compression from a tumor is what? - A magnetic resonance imaging (MRI) scan - An ultrasound - A computed tomography (CT) scan - An X-ray

Correct response: A magnetic resonance imaging (MRI) scan Explanation: The MRI scan is the most commonly used diagnostic procedure. It is the most sensitive diagnostic tool that is particularly helpful in detecting epidural spinal cord compression and vertebral bone metastases.

A patient with herpes simplex virus (HSV) encephalitis has been admitted to the ICU. What medication would the nurse expect the health care provider to order for the treatment of this disease process? - Cyclosporine (Neoral) - Acyclovir (Zovirax) - Cyclobenzaprine (Flexeril) - Ampicillin (Principen)

Correct response: Acyclovir (Zovirax) Explanation: Acyclovir (Zovirax), an antiviral agent, is the medication of choice in HSV treatment. The mode of action is the inhibition of viral DNA replication. To prevent relapse, treatment would continue for up to 3 weeks. Cyclosporine is an immunosuppressant and antirheumatic. Cyclobenzaprine is a centrally acting skeletal muscle relaxant. Ampicillin is an antibiotic.

A nurse who provides care at a community clinic is in contact with a diverse group of patients. Which of the following individuals most clearly displays risk factors for stroke? - An obese woman with a history of atrial fibrillation and type 2 diabetes - A man who is receiving oral antibiotics for the treatment of a chlamydial infection - A woman who has osteoporosis, a history of fractures, and a family history of stroke - A 70-year-old man who has benign prostatic hyperplasia and early stage Alzheimer's disease

Correct response: An obese woman with a history of atrial fibrillation and type 2 diabetes Explanation: Obesity, atrial fibrillation, and type 2 diabetes are all highly significant risk factors for stroke. None of the other listed individuals displays multiple risk factors for stroke.

A 33-year-old man who is in healthy physical condition has been admitted to the emergency room with unilateral weakness and slurred speech. A computed tomography (CT) scan of the man's head reveals a hemorrhagic stroke despite the fact that the man is in good health with no history of hypertension, thromboembolism, or drug use. What type of brain tumor has the potential to cause this man's health problem? - Acoustic neuroma - Pituitary adenoma - Angioma - Meningioma

Correct response: Angioma Explanation: Because the walls of the blood vessels in angiomas are thin, affected patients are at risk for hemorrhagic stroke. In fact, cerebral hemorrhage in people younger than 40 years of age should suggest the possibility of an angioma.

A patient has been admitted to a unit at a primary stroke center after experiencing an ischemic stroke. The nurse on the unit is aware of the vital importance of rehabilitative efforts and knows that an active rehabilitation program should begin at what point? - As soon as the patient is able to independently identify goals for rehabilitation - As soon as moderate motor activity is regained on the affected side - As soon as sensory ability is regained on the affected side - As soon as the patient regains consciousness

Correct response: As soon as the patient regains consciousness Explanation: Usually an active rehabilitation program is started as soon as the patient regains consciousness. It would be erroneous to wait until the affected side recovers. Patients may benefit from rehabilitation before they are able to independently set goals.

While reviewing the nursing documentation on a patient on the neurological unit, the nurse notes that the patient complained of a headache several times over the previous shift. How can the nurse differentiate between a headache that is caused by a brain tumor and a headache that is caused by meningitis or encephalitis? - Assess the active and passive range of motion of the patient's neck. - Assess for the presence of a fever. - Assess the patient's carotid pulses bilaterally. - Assess the patient's orientation to person, place, and time.

Correct response: Assess for the presence of a fever. Explanation: When the patient complains of a headache, the nurse assesses the patient's temperature. The nurse knows that fever with headache is associated with an infectious process such as meningitis or encephalitis, whereas headache without fever is associated with a tumor or intracerebral bleeding.

A 76-year-old male client is brought to the clinic by his daughter. The daughter states that her father has had two transient ischemic attacks (TIAs) in the past week. The physician orders carotid angiography, and the report reveals that the carotid artery has been narrowed by atherosclerotic plaques. What treatment option does the nurse expect the physician to offer this client to increase blood flow to the brain? Select all that apply. - Percutaneous transluminal coronary artery angioplasty - Carotid endarterectomy - Administration of tissue plasminogen activator - Removal of the carotid artery - Balloon angioplasty of the carotid artery followed by stent placement

Correct response: Balloon angioplasty of the carotid artery followed by stent placement Carotid endarterectomy Explanation: If narrowing of the carotid artery by atherosclerotic plaques is the cause of the TIAs, a carotid endarterectomy (surgical removal of atherosclerotic plaque) could be performed. A balloon angioplasty of the carotid artery, a procedure similar to a percutaneous transluminal coronary artery angioplasty, may be performed alternatively to dilate the carotid artery and increase blood flow to the brain, followed by stent placement. The other options are not options to increase blood flow through the carotid artery to the brain.

Extensive diagnostic testing has resulted in a patient's diagnosis of a benign brain tumor. When providing care for this patient, the nurse should be cognizant of which of the following characteristics of benign brain tumors? - Benign brain tumors have no physiological effect but should be closely monitored. - Benign brain tumors can slowly grow into an area of vital brain function. - Benign brain tumors constitute a risk factor for possible metastasis. - Benign brain tumors typically become malignant within 1 to 2 years.

Correct response: Benign brain tumors can slowly grow into an area of vital brain function. Explanation: Benign tumors are slow growing but can occur in a vital area, where they can grow large enough to cause serious effects. They do not necessarily develop to malignancy, and they are not primarily understood as simply a risk factor for further cancer.

A client is following up after a visit to the emergency department where testing indicated that the client had suffered a transient ischemic attack. What lifestyle changes would the nurse include in teaching to prevent further reoccurrence? Select all that apply. - High-dose aspirin - Smoking cessation - Weight loss - Blood pressure control - Physical activity limitations

Correct response: Blood pressure control Weight loss Smoking cessation Explanation: Primary prevention of ischemic stroke remains the best approach. A healthy lifestyle including not smoking, engaging in physical activity (at least 40 minutes a day, 3 to 4 days a week), maintaining a healthy weight, and following a healthy diet (including modest alcohol consumption) can reduce the risk of having a stroke. Specific diets that have decreased risk of stroke include the Dietary Approaches to Stop Hypertension (DASH) diet (high in fruits and vegetables, moderate in low-fat dairy products, and low in animal protein), the Mediterranean diet (supplemented with nuts), and overall diets that are rich in fruits and vegetables. Research findings suggest that low-dose aspirin may lower the risk of a first stroke for those who are at risk.

A patient who just suffered a hemorrhagic stroke is brought to the emergency department by ambulance. What should be the nurse's primary assessment focus? - Cardiac and respiratory status - Urinary output - Seizure activity - Fluid and electrolyte balance

Correct response: Cardiac and respiratory status Explanation: Acute care begins with managing the ABC's. Patients may have difficulty keeping an open and clear airway secondary to decreased level of consciousness. Neurological assessment with close monitoring for signs of increased neurological deficit and seizure activity occurs next. Fluid and electrolyte balance must be controlled carefully, with the goal of adequate hydration to promote perfusion and decrease further brain damage.

A patient has been brought to the emergency department (ED) with signs and symptoms that are characteristic of an ischemic stroke. The ED nurse is aware that a disruption in cerebral blood flow will have initiated the ischemic cascade. The mitochondria in the patient's affected neurons respond to impaired blood supply by: - Changing from aerobic respiration to anaerobic metabolism - Metabolizing stored glucose contained in the cytoplasm - Releasing cytokines that signal the brain to shunt blood flow to alternative routes - Signaling the cell nucleus to synthesize oxygen

Correct response: Changing from aerobic respiration to anaerobic metabolism Explanation: The ischemic cascade begins when cerebral blood flow decreases to less than 25 mL/100 g/min. At this point, neurons are no longer able to maintain aerobic respiration. The cell's mitochondria must then switch to anaerobic metabolism, generating large amounts of lactic acid and causing a change in pH.

Magnetic resonance imaging has confirmed a diagnosis of glioblastoma multiforme (GBM) in a 56-year-old male patient. The nurse who is planning this patient's care should prioritize which of the following nursing actions? - Liaising with community agencies to organize long-term rehabilitation - Choosing psychosocial interventions that are relevant to the patient's poor prognosis - Teaching the patient about the importance of healthy lifestyle in recovery from GBM - Teaching the patient about the pharmacological interventions relevant to his treatment

Correct response: Choosing psychosocial interventions that are relevant to the patient's poor prognosis Explanation: GBM is the most common and aggressive malignant brain tumor, and the overall prognosis is poor. Consequently, interventions should be chosen in light of this difficult reality. Pharmacological interventions are not the central treatments of GBM.

A client with a brain tumor is experiencing changes in cognition that require the nurse to reorient the client frequently. When performing this task, which devices would be appropriate for the nurse to use? Select all that apply. - Client's clothing - Common words - Calendar - Picture of the client's family - Clock

Correct response: Client's clothing Picture of the client's family Clock Calendar Explanation: Clients with changes in cognition caused by their lesions require frequent reorientation and the use of orienting devices (e.g., personal possessions, photographs, lists, and a clock). Words would not be as helpful as items that are familiar to the client.

A 66-year-old woman with a recent history of headaches and agitation has been found to have a meningioma. The neurological nurse should understand that the symptoms of this woman's tumor are attributable to what pathophysiological process? - Invasion of brain tissue by the tumor - Impaired cerebrospinal fluid (CSF) synthesis - Compression of brain tissue - Intracerebral hemorrhage

Correct response: Compression of brain tissue Explanation: Manifestations of meningiomas depend on the area involved and are the result of compression rather than invasion of brain tissue. These tumors do not normally result in alterations in CSF production or bleeding.

A patient with generalized seizure disorder has just had a seizure. The nurse would assess for what characteristic associated with the postictal state? - Epileptic cry - Confusion - Urinary incontinence - Body rigidity

Correct response: Confusion Explanation: In the postictal state (after the seizure), the patient is often confused, hard to arouse, and may sleep for hours. The epileptic cry occurs from the simultaneous contractions of the diaphragm and chest muscles that occur during the seizure. Urinary incontinence and intense rigidity of the entire body are followed by alternating muscle relaxation and contraction (generalized tonic-clonic contraction) during the seizure.

A community health nurse is conducting a workshop for unlicensed care providers who work in a chain of long-term care facilities. The nurse is teaching the participants about the signs and symptoms of stroke. What signs and symptoms should the nurse identify? Select all that apply. - Visual disturbances - Sudden ear pain - Sudden numbness - Confusion - Epistaxis (nosebleed)

Correct response: Confusion Sudden numbness Visual disturbances Explanation: The most common symptoms of stroke include numbness or weakness of the face, arm, or leg, especially on one side of the body; confusion or change in mental status; trouble speaking or understanding speech; visual disturbances; difficulty walking, dizziness, or loss of balance or coordination; and sudden, severe headache.

A female patient who is recovering from a stroke has begun eating a minced and pureed diet after passing the speech pathologist's swallowing assessment. This morning, the nurse set up the patient with her breakfast tray and later noticed that the woman was swallowing her food well but dribbling small amounts of food out of affected side of her mouth. How should the nurse follow up this observation? - Provide oral suctioning after each bite that the patient swallows. - Remove the patient's tray because of the risk of aspiration. - Cue the patient to the fact that she is dribbling food while commending her for eating. - Make the patient NPO and encourage the care provider to consider enteral nutrition.

Correct response: Cue the patient to the fact that she is dribbling food while commending her for eating. Explanation: Dribbling of food should be noted and addressed but does not necessarily constitute an acute risk of aspiration. Close observation is warranted but enteral feeding and NPO status are not likely necessary. Suctioning after each bite of food is not necessary.

The nurse is caring for a patient with MS who is having spasticity in the lower extremities that decreases physical mobility. What interventions can the nurse provide to assist with relieving the spasms? Select all that apply. - Have the patient take a hot tub bath to allow muscle relaxation. - Apply warm compresses to the affected areas. - Assist with a rigorous exercise program to prevent contractures. - Demonstrate daily muscle stretching exercises. - Allow the patient adequate time to perform exercises

Correct response: Demonstrate daily muscle stretching exercises. Apply warm compresses to the affected areas. Allow the patient adequate time to perform exercises Explanation: Warm packs may be beneficial for relieving spasms, but hot baths should be avoided because of risk of burn injury secondary to sensory loss and increasing symptoms that may occur with elevation of the body temperature. Daily exercises for muscle stretching are prescribed to minimize joint contractures. The patient should not be hurried in any of these activities, because this often increases spasticity.

A nurse has developed a strong therapeutic rapport with a male patient during the patient's recovery in hospital from a stroke. The patient has had a largely successful recovery but has admitted to the nurse that he has concerns about how his sexual relationship will be affected upon discharge. The nurse should respond to the patient's statement by: - Encouraging him to focus on his achievements rather than his perceived deficits - Encouraging him to consider alternative forms of sexual expression with his partner - Encouraging him to prioritize the emotional aspects of his relationship rather than the physical aspects - Encouraging him with the fact that this aspect of his life is not likely to have been affected by his stroke

Correct response: Encouraging him to consider alternative forms of sexual expression with his partner Explanation: Encouraging the patient to explore alternative methods of sexual expression acknowledges the patient's concerns and provides a realistic and empathic response. It is inappropriate to divert the patient's concerns or to provide unrealistic expectations.

A patient is being treated in hospital for St. Louis encephalitis. When planning this patient's care, the nurse should be aware that this specific variant of encephalitis creates a potential for what nursing diagnosis? - Risk for deficient fluid volume - Excess fluid volume - Risk for unstable blood glucose - Imbalanced nutrition: less than body requirements

Correct response: Excess fluid volume Explanation: A unique clinical feature of St. Louis encephalitis is the development of syndrome of inappropriate antidiuretic hormone secretion (SIADH) with hyponatremia in 25% to 33% of affected patients; SIADH often results in profound fluid overload. Impaired nutrition and unstable blood glucose levels may occur.

The nurse is conducting a neurological assessment with a client who has increased intracranial pressure secondary to growth of brain tumor mass. What assessment tools can the nurse use to determine the client's neurological status? Select all that apply. - Beck Depression Inventory (BDI) - Glasgow coma scale (GCS) - Urinalysis - Chest auscultation - Mini mental status examination (MMSE)

Correct response: Glasgow coma scale (GCS) Mini mental status examination (MMSE) Explanation: Included within the neurological examination to determine deficits, the nurse should use the GCS, an assessment tool that can help identify the severity of brain injury for clients who have had surgery to remove a brain tumor. The MMSE can assist in evaluating the client's orientation to person, place and time. This tool can also assist the nurse is identifying changes to the client's cognitive functioning that may result from brain injury. A urinalysis would not provide any information on the client's neurological status. This test can provide information about the appearance, concentration and content of urine, but this is non-specific to issues related to neurological status. Chest auscultation is useful when conducting a respiratory assessment. The BDI is used to quantify an assessment of a client's mood and the severity of depression. This tool can be helpful when screening for mood disorders, but it does not provide any useful information about neurological status.

A patient diagnosed with a pituitary adenoma has arrived on the oncology unit. Based upon the nurse's initial assessment, the patient is most likely to exhibit: - Headache - Decreased level of consciousness - Decreased intracranial pressure (ICP) - Restlessness

Correct response: Headache Explanation: Pressures from pituitary adenomas may cause headaches, visual dysfunction, and hypothalamic disorders. Restlessness is not a typical manifestation of pituitary adenomas. Pressure from pituitary adenomas would increase ICP, and these tumors would not usually cause decreased LOC.

A nurse is assisting with the assessment of a client with suspected brain abscess. Which of the following findings would be consistent with such an abscess in the frontal lobe of the brain? Select all that apply. - Changes in vision - Seizures - Hemiparesis - Expressive aphasia - Nystagmus

Correct response: Hemiparesis Seizures Expressive aphasia Explanation: Signs and symptoms of a frontal lobe abscess include hemiparesis, expressive aphasia, seizures, and frontal headache. Vision changes are associated with a temporal lobe abscess. Nystagmus is a sign of a cerebellar abscess.

An emergency department (ED) nurse has administered an ordered bolus of tissue plasminogen activator (tPA) to a male patient who was diagnosed with stroke. During the administration of tPA, the nurse should prioritize assessments related to what problem? - Peripheral edema - Fluid overload - Hemorrhage - Acute pain

Correct response: Hemorrhage Explanation: Bleeding is the most common side effect of tPA. The patient is closely monitored for bleeding (at IV insertion sites, gums, urine/stools, and intracranially by assessing changes in level of consciousness). Edema, fluid overload, and pain are not likely to result from tPA.

The nurse is caring for a patient on the neurological unit who is in status epilepticus. What medication does the nurse anticipate being given to halt the seizure? - IV lidocaine - IV phenobarbital - IV diazepam - Oral phenytoin

Correct response: IV diazepam Explanation: Status epilepticus (acute prolonged seizure activity) is a series of generalized seizures that occur without full recovery of consciousness between attacks. Medical management of status epilepticus includes IV diazepam (Valium) and IV lorazepam (Ativan), given slowly in an attempt to halt seizures immediately. Other medications (phenytoin, phenobarbital) are given later to maintain a seizure-free state.

A 55-year-old male patient has been admitted to the hospital with a gastrointestinal bleed, and the patient has just experienced a generalized seizure that may be attributable to alcohol withdrawal. When providing immediate care during the patient's seizure, what nursing diagnosis should be prioritized? - Acute confusion - Risk for impaired skin integrity - Acute pain - Impaired gas exchange

Correct response: Impaired gas exchange Explanation: Airway and breathing are priorities in any emergency situation, including seizures. These considerations would be prioritized over confusion, pain, and skin integrity.

A patient has just returned to the unit from the PACU after surgery for a tumor within the spine. The patient complains of pain. When positioning the patient for comfort and to reduce injury to the surgical site, the nurse will position the patient in what position? - In the reverse Trendelenburg position - In the Trendelenburg position - In a flat side-lying position - In the lithotomy position

Correct response: In a flat side-lying position Explanation: After spinal surgery, the bed is usually kept flat initially. The side-lying position is usually the most comfortable because this position imposes the least pressure on the surgical site. The lithotomy position, the Trendelenburg position, and the reverse Trendelenburg position are inappropriate for this patient.

A middle-aged woman has just received word that her recent diagnostic testing has resulted in a diagnosis of myasthenia gravis. The nurse who is contributing to this woman's care should be aware that she is experiencing signs and symptoms that are the result of what pathophysiological process? - Nerve demyelination - Inadequate action of acetylcholine - Loss of upper and lower motor neurons - Decreased levels of dopamine

Correct response: Inadequate action of acetylcholine Explanation: In myasthenia gravis, there is a reduction in the number of acetylcholine receptor sites because antibodies directed at the acetylcholine receptor sites impair transmission of impulses across the neuromuscular junction. Demyelination causes multiple sclerosis, whereas low dopamine levels are implicated in Parkinson's disease. Amyotrophic lateral sclerosis (ALS) results from the loss of upper and lower motor neurons.

A 44-year-old woman has been admitted to the medical unit because of a recent exacerbation of multiple sclerosis. Which of the following nursing actions should be prioritized in this patient's care? - Reorient the patient to time and place during interactions. - Institute measures to reduce the patient's risk of falls. - Provide a liquid diet. - Implement universal infection control precautions.

Correct response: Institute measures to reduce the patient's risk of falls. Explanation: Involvement of the cerebellum or basal ganglia can produce ataxia (impaired coordination of movements) and tremor. This phenomenon, combined with muscle weakness, creates a risk of falls. Severe confusion is less common, and the patient does not normally have a greatly increased risk of infection. A liquid diet is not necessary

Which are contraindications for the administration of tissue plasminogen activator (t-PA)? Select all that apply. - Major abdominal surgery within 10 days - Age 18 years or older - Intracranial hemorrhage - Ischemic stroke - Systolic BP less than or equal to 185 mm Hg

Correct response: Intracranial hemorrhage Major abdominal surgery within 10 days Explanation: Intracranial hemorrhage, neoplasm, aneurysm, and major surgical procedures within 14 days are contraindications to t-PA. Clinical diagnosis of ischemic stroke, being 18 years of age or older, and a systolic BP less than or equal to 185 mm Hg are eligibility criteria.

A family member comes to the clinic to talk to the nurse about a client who has had a stroke on the right side of the brain. The family member is concerned because of the deficits the client is exhibiting. The nurse knows that when a client experiences a stroke on the right side of the brain, common deficits include what? Select all that apply. - Hyperaware of deficits - Impairment of long-term memory - Left-sided hemiplegia - Tendency to distractibility - Neglect of objects and people on the left side

Correct response: Left-sided hemiplegia Tendency to distractibility Neglect of objects and people on the left side Explanation: Left-sided hemiplegia (stroke on right side of brain) may have the following neurologic deficits: spatial-perceptual defects; disregard for the deficits of the affected side require special safety considerations; tendency to distractibility; impulsive behavior, unaware of deficits; poor judgment; defects in left visual fields; misjudge distances; difficulty distinguishing upside-down and right-side up; impairment of short-term memory; and neglect left side of body, objects and people on left side.

The nurse is called to attend to a patient having a seizure in the waiting area. What nursing care is provided for a patient who is experiencing a convulsive seizure? Select all that apply. - Opening the patient's jaw and inserting a mouth gag - Loosening constrictive clothing - Positioning the patient on his or her side with head flexed forward - Providing for privacy - Restraining the patient to avoid self injury

Correct response: Loosening constrictive clothing Positioning the patient on his or her side with head flexed forward Providing for privacy Explanation: During a patient's seizure, the nurse should do the following. Loosen constrictive clothing. If possible, place the patient on one side with head flexed forward, which allows the tongue to fall forward and facilitates drainage of saliva and mucus. If suction is available, use it if necessary to clear secretions. Provide privacy, and protect the patient from curious onlookers. (The patient who has an aura [warning of an impending seizure] may have time to seek a safe, private place.) The nurse should not attempt to pry open jaws that are clenched in a spasm or attempt to insert anything. Broken teeth and injury to the lips and tongue may result from such an action. No attempt should be made to restrain the patient during the seizure, because muscular contractions are strong and restraint can produce injury.

A 13-year-old patient is admitted to the pediatric unit with a suspected brain tumor. The nurse should understand that which diagnostic test is the most helpful in the diagnosis of brain tumors? - Computed tomography (CT) scan - Brain biopsy - Magnetic resonance imaging (MRI) - Blood work with adrenocorticotropic hormone (ACTH) levels

Correct response: Magnetic resonance imaging (MRI) Explanation: An MRI is the most helpful in the diagnosis of brain tumors. Its use has resulted in the detection of smaller lesions; it is particularly helpful in detecting tumors in the brainstem and pituitary regions, where bone interferes with CT. A brain biopsy or blood work with ACTH levels does not diagnose brain tumors.

The community health nurse is preparing to conduct a home visit to a client in the community who was recently discharged from hospital after treatment of a metastatic brain lesion. What should the community health nurse plan to include within the time allotted for the home visit? Select all that apply. - Mental status examination - Skin integrity - Mobility - Use of pain medication - Cranial nerve functioning

Correct response: Mental status examination Skin integrity Mobility Use of pain medication Explanation: The community health nurse should ensure a comprehensive assessment is conducted to note any new deficits that may have developed since the client was discharged from the hospital. The mental status examination would uncover any neurological deficits that have developed. Assessment of skin integrity would uncover any possible impairments that could become portals for infection. This is especially important for clients who have bowel and/or bladder dysfunction. Changes in mobility could be related to pain management or new neurological deficits caused by compression of brain structures. It is always important to conduct a pain assessment to ensure client has adequate pain relief to engage in activities of daily living, rehabilitative activities and, overall, for the client's quality of life. Reports of increased pain and in different regions can be indicative of progression of the metastatic lesion. Cranial nerve function should be assessed during a clinic visit as the nurse may be limited in the community care setting (the client's home) with regards to the extent of a cranial nerve assessment that can be conducted.

A client diagnosed with a brain tumor is exhibiting focal symptoms. Which assessment findings are the nurse likely to note? Select all that apply. - Vomiting - Sensory loss - Visual changes - Aphasia - Muscle weakness

Correct response: Muscle weakness Sensory loss Aphasia Visual changes Explanation: Common focal, or localized, symptoms include muscle weakness, sensory loss, aphasia, and visual changes. When specific regions of the brain are affected, additional local signs and symptoms occur, such as motor abnormalities, changes in hearing, alterations in cognition, and language disturbances. Vomiting would be considered a generalized symptom.

The nurse educator is facilitating a class on neurological function with a group of nursing students. When discussing problems that can result from growing brain tumors, the nurse educator should include that clients can experience which neurologic deficits even after surgical resection? Select all that apply. - Respiratory infection - Aphasia - Incontinence - Fever - Paralysis

Correct response: Paralysis Incontinence Aphasia Explanation: Although fever and respiratory infection can result from various factors that influence the hospitalized client, these are not categorized as neurologic deficits. The nurse educator is correct in stating that paralysis, incontinence and aphasia are potential neurological deficits that can result from pressure of growing tumors on surrounding brain structures. The arise from a decreased sensory motor response of the central and peripheral nervous system.

A patient, diagnosed with cancer of the lung, has just been told that she has metastases to the brain. The family should be aware that the neurologic signs and symptoms of metastatic brain disease are most often what? - Bradycardia - Increase in diastolic blood pressure - Temperature greater than 100.5°F - Personality changes

Correct response: Personality changes Explanation: Neurologic signs and symptoms include headache, gait disturbances, visual impairment, personality changes, altered mentation (memory loss and confusion), focal weakness, paralysis, aphasia, and seizures. These problems can be devastating to both patient and family. Bradycardia, elevated temperature, and an increase in diastolic blood pressure are not typical neurologic signs and symptoms of metastatic brain disease.

A 30-year-old female patient has been referred to a specialist by her primary care provider because she has recently developed fat pads at the base of her neck, an increasingly round face, and striae at various locations. The patient's signs of illness have been attributed to a brain tumor. What type of brain tumor is most likely to result in these changes to the woman's physical appearance? - Meningioma - Pituitary adenoma - Glioma - Acoustic neuroma

Correct response: Pituitary adenoma Explanation: Adrenocorticotropic hormone (ACTH)-producing pituitary adenomas result in Cushing's disease, which is characterized by signs and symptoms that include a "buffalo hump," a rounded face, and striae.

A patient has had an ischemic stroke and has been admitted to the unit. The nurse knows the importance of the principles of body alignment and correct positioning to stroke victims. How should the nurse position the patient to prevent joint deformities? - Place the patient flat in the prone position for 30 minutes per day. - Place patient's hand in pronation. - Place a pillow in the axilla when there is limited external rotation. - Assist the patient in acutely flexing the thigh to promote movement.

Correct response: Place a pillow in the axilla when there is limited external rotation. Explanation: A pillow in the axilla prevents adduction of the affected shoulder and keeps the arm away from the chest. The prone position with a pillow under the pelvis, not flat, promotes hyperextension of the hip joints, essential for normal gait. To promote venous return and prevent edema, the upper thigh should not be flexed acutely. The hand is placed in slight supination, not pronation, which is its most functional position.

After having a stroke, a patient has cognitive deficits. What are the cognitive deficits the nurse recognizes the patient has as a result of the stroke? Select all that apply. - Paresthesias - Expressive aphasia - Short- and long-term memory loss - Poor abstract reasoning - Decreased attention span

Correct response: Poor abstract reasoning Decreased attention span Short- and long-term memory loss Explanation: Cognitive deficits associated with stroke include short- and long-term memory loss, decreased attention span, and poor abstract reasoning. Expressive aphasia is a verbal deficit, not a cognitive deficit. Paresthesias are sensory deficits, not cognitive deficits.

A male patient presents to the clinic complaining of a headache. The nurse notes that the patient is guarding his neck, and he tells the nurse that he has stiffness in the neck area. The nurse suspects the patient may have meningitis. What is another well-recognized sign of this infection? - Negative Brudzinski's sign - Positive Kernig's sign - Hyperactive patellar reflex - Sluggish pupil reaction

Correct response: Positive Kernig's sign Explanation: Meningeal irritation results in a number of well-recognized signs commonly seen in meningitis, such as a positive Kernig's sign, a positive Brudzinski's sign, and photophobia. Hyperactive patellar reflex and a sluggish pupil reaction are not common signs of meningitis.

The nurse is caring for a client diagnosed with a subarachnoid hemorrhage resulting from a leaking aneurysm. The client is awaiting surgery. Which nursing interventions would be appropriate for the nurse to implement? Select all that apply. - Elevate the head of bed 30 degrees. - Provide a dimly lit environment. - Ambulate the client every hour. - Administer docusate per order. - Permit friends to visit often.

Correct response: Provide a dimly lit environment. Elevate the head of bed 30 degrees. Administer docusate per order. Explanation: Cerebral aneurysm precautions are implemented for the patient with a diagnosis of aneurysm to provide a nonstimulating environment, prevent increases in intracranial pressure (ICP), and prevent further bleeding. The patient is placed on immediate and absolute bed rest in a quiet, nonstressful environment, because activity, pain, and anxiety elevate blood pressure, which increases the risk for bleeding. Visitors, except for family, are restricted. Dim lighting is helpful because photophobia (visual intolerance of light) is common. The head of the bed is elevated 15 to 30 degrees to promote venous drainage and decrease ICP. No enemas are permitted, but stool softeners (Colace) and mild laxatives are prescribed. Both prevent constipation, which would cause an increase in ICP, as would enemas.

A nurse is caring for a client with a cerebral aneurysm. Which nursing interventions would be most useful to the nurse to avoid bleeding in the brain? Select all that apply. - Report changes in neurologic status as soon as a worsening trend is identified. - Follow the healthcare provider's orders to increase fluid volume. - Maintain the head of the bed at 30 degrees. - Use a well-lighted room for assessments every 2 hours. - Avoid any activities that cause a Valsalva maneuver.

Correct response: Report changes in neurologic status as soon as a worsening trend is identified. Maintain the head of the bed at 30 degrees. Avoid any activities that cause a Valsalva maneuver. Explanation: Cerebral aneurysm precautions are implemented for the patient with a diagnosis of aneurysm to provide a nonstimulating environment, prevent increases in intracranial pressure, and prevent further bleeding. The patient is placed on bed rest in a quiet, nonstressful environment, because activity, pain, and anxiety are thought to elevate the blood pressure, which may increase the risk for bleeding. The head of the bed is elevated 30 degrees to promote venous drainage and decrease intracranial pressure. Any activity that suddenly increases the blood pressure or obstructs venous return is avoided. This includes the Valsalva maneuver, straining, forceful sneezing, pushing oneself up in bed and acute flexion or rotation of the head and neck (which compromises the jugular veins). Stool softeners and mild laxatives are prescribed to prevent constipation, which can cause an increase in intracranial pressure. Dim lighting is helpful for photophobia. Increasing fluid volume does not affect brain bleeding.

The nurse is caring for a client postoperatively from a spinal tumor resection. The nurse assesses that the client has partial paralysis. What anticipated problems should the nurse include in the client's care plan? Select all that apply. - Risk for sexual dysfunction - Risk for knowledge deficit - Risk for impaired physical mobility - Risk for powerlessness - Risk for injury

Correct response: Risk for impaired physical mobility Risk for injury Risk for powerlessness Risk for knowledge deficit Explanation: The change in the client's muscle strength will effect the client's ability to carry out activities that he or she was once used to being able to perform independently. Due to the partial paralysis, the client is now at risk for impaired physical mobility related to a decreased range of motion. The client is at risk for injury due to a possible unsteady gait. The development of partial paralysis is a loss for the client, and there is the potential for feelings of powerlessness related to inability to control situation and being dependent on others. It is not likely that the nurse has been able to accurately assess sexual dysfunction. Not all clients with partial paralysis experience sexual dysfunction because this is dependent on the extent of spinal injury or nerve compression. Given that teaching is needed, it implies that there is a knowledge deficit.

A patient with Parkinson's disease is undergoing a swallowing assessment because she is experiencing difficulties when swallowing. What consistency is most appropriate for this patient, to reduce the risk of aspiration? - Semisolid food with thick liquids - Pureed food with water - Thin liquids only - Solid food with thin liquids

Correct response: Semisolid food with thick liquids Explanation: A semisolid diet with thick liquids is easier to swallow for a patient with swallowing difficulties than a solid diet. Thin liquids should be avoided. Pureed foods with water are not indicated for this patient.

The nurse is providing morning care for a male patient who is recovering from a head injury. The patient's right hand has begun twitching, and he is speaking unintelligibly. This patient has most likely experienced what type of seizure? - Tonic-clonic seizure - Simple partial - Complex partial - Absence seizure

Correct response: Simple partial Explanation: In simple partial seizures, only a finger or hand may shake, or the mouth may jerk uncontrollably. The person may talk unintelligibly, may be dizzy, and may experience unusual or unpleasant sights, sounds, odors, or tastes, but without loss of consciousness. In complex partial seizures, the person either remains motionless or moves automatically but inappropriately for time and place, or he or she may experience excessive emotions of fear, anger, elation, or irritability. Tonic-clonic seizures begin with rigidity (tonic phase), followed by repetitive clonic activity of all extremities characterized by stiffening or jerking of the body. Absence (petit mal) seizures involve short episodes of staring and loss of awareness.

The nurse is providing education to a client who is being discharged with an outpatient treatment plan that includes taking a chemotherapeutic agent. What instructions should the nurse include? Select all that apply. - The client should seek emergency care if he or she develops a fever. - Hair loss should be expected when taking the medication. - The client should seek emergency help if nausea or vomiting occur. - The client should ensure no one else handles the medication. - If a dose is missed, the client should take double the amount at the regular time the following day.

Correct response: The client should ensure no one else handles the medication. The client should seek emergency care if he or she develops a fever. Hair loss should be expected when taking the medication. Explanation: The client should be the only person to handle the medication. Because it is a chemotherapy agent, it is cytotoxic and can have a harmful effect on anyone who does not have a tumor. It is unsafe to take a double dose of the medication if it is missed the previous day. The client should be instructed to take the medication at the same time each day and, if a dose is missed, the client should be instructed to take it as soon as possible and then get back on the regular schedule again. Some clients taking this medication experience gastrointestinal side effects such as nausea and vomiting. Although this is not considered an emergency, the client should be instructed to discuss this side effect with the health care provider, because prolonged symptoms can lead to nutritional deficit and/or dehydration. Immunosuppression caused by the medication can lead to a white blood cell count too low to fight off an infection. A fever is a sign of infection and can be life-threatening for a person taking a chemotherapeutic agent. Care should be sought immediately in this case. A common side effect of this medication is alopecia or hair loss. The client should be made aware to anticipate that this is a possibility while taking the drug.

A patient's recent diagnostic workup has resulted in a diagnosis of a glioma, and a treatment plan is being promptly created by the multidisciplinary care team. The patient's oncologist has recommended chemotherapy, which is to be administered by the intrathecal route. The nurse should understand that the rationale for choosing this administration route involves which of the following considerations? - The patient will require weekly, rather than daily, drug administration. - The drug will bypass the blood-brain barrier. - The patient will not require IV access. - The drug can be administered on an outpatient basis.

Correct response: The drug will bypass the blood-brain barrier. Explanation: Chemotherapy that is given by intrathecal injection bypasses the blood-brain barrier. The rationale for choosing this route does not involve foregoing IV access, less frequent administration, or the possibility of outpatient administration.

A patient has been admitted to the neurological ICU with a diagnosis of a brain tumor. The patient is scheduled to have a tumor resection/removal in the morning. What is a priority part of the nurse's preoperative assessment of this patient? - Ability to chew - Sensory perception - The gag reflex - Corneal reflex

Correct response: The gag reflex Explanation: Preoperatively, the gag reflex and ability to swallow are evaluated. In patients with diminished gag response, care includes teaching the patient to direct food and fluids toward the unaffected side, having the patient sit upright to eat, offering a semisoft diet, and having suction readily available. The ability to chew and the corneal reflex would be assessed, and so would sensory perception on the face, but none of them is more important than the gag reflex.

A patient with a suspected brain tumor has been scheduled for a positron emission tomography (PET) scan. The nurse should explain to the patient that this test is being performed to assess: - The metabolic activity taking place in the patient's brain - The tissue characteristics of the patient's brain - The distribution patterns of cerebrospinal fluid (CSF) in the patient's central nervous system - The blood flow in the patient's brain

Correct response: The metabolic activity taking place in the patient's brain Explanation: PET, which measures the brain's activity rather than simply its structure, is useful in differentiating tumor from scar tissue or radiation necrosis. PET is not primarily used to assess blood flow, CSF flow, or structural characteristics.

A patient has severe shoulder pain from subluxation of the shoulder is being cared for on the unit. To prevent further injury and pain, the nurse caring for this patient is aware of what? - Passively exercising the affected extremity is avoided to minimize pain. - Use of a sling should be avoided due to adduction of the affected shoulder. - Elevation of the arm and hand can lead to further complications associated with edema. - The patient should be taught to interlace fingers, place palms together, and slowly bring scapulae forward to avoid excessive force to shoulder.

Correct response: The patient should be taught to interlace fingers, place palms together, and slowly bring scapulae forward to avoid excessive force to shoulder. Explanation: To prevent shoulder pain, the nurse should never lift a patient by the flaccid shoulder or pull on the affected arm or shoulder. The patient is taught how to move and exercise the affected arm/shoulder through proper movement and positioning. The use of a properly worn sling when the patient is out of bed prevents the paralyzed upper extremity from dangling without support. Range of motion exercises are still vitally important in preventing a frozen shoulder and ultimate atrophy of subcutaneous tissues, which can cause more pain. Elevation of the arm and hand is also important in preventing dependent edema of the hand.

The nurse is liaising with the physical therapist and occupational therapist to create an activity management plan for a patient who has multiple sclerosis. What principle should be integrated into guidelines for exercise and activity that the team will provide to this patient in anticipation of discharge? - The patient should attempt to maintain prediagnosis levels of activity and mobility. - The patient should perform frequent physical activity but avoid becoming fatigued. - The patient should prioritize energy conservation and remain on bed rest if possible. - The patient should perform exercises that are brief but high-intensity.

Correct response: The patient should perform frequent physical activity but avoid becoming fatigued. Explanation: The patient is encouraged to work and exercise to a point just short of fatigue. Very strenuous physical exercise is not advisable because it raises the body temperature and may aggravate symptoms. The patient is advised to take frequent short rest periods, preferably lying down. Extreme fatigue may contribute to the exacerbation of symptoms. It is unrealistic to expect the patient to maintain prediagnosis levels of activity.

The nurse is writing a care plan for a patient with brain metastases. Following a thorough assessment, the nurse decides that an appropriate nursing diagnosis is "Anxiety related to lack of control over the health care needs and situation." In establishing this plan of care for the patient, the nurse will identify what measure as appropriate for the care of this patient? - The patient will receive antianxiety medications every 4 hours. - The patient will be encouraged to verbalize concerns related to the disease and its treatment. - The patient will begin a busy schedule of therapy, so that he will forget about the anxiety. - The patient's family will be instructed on measures to implement when providing care for the patient.

Correct response: The patient will be encouraged to verbalize concerns related to the disease and its treatment. Explanation: Patients need the opportunity to exercise some control over their situation. A sense of mastery can be gained as they learn to understand the disease and its treatment and how to deal with their feelings. Distraction, assuming care responsibilities, and administering medications will not allow the patient to gain some control over his situation or discuss his feelings.

11) ** The nurse is seeing the mother of a client who states, "I'm so relieved because my son's doctor told me his brain tumor is benign." The nurse knows what is true about benign brain tumors? - They are all metastatic. - The prognosis is very poor. - They can affect vital functioning. - They do not require surgical removal.

Correct response: They can affect vital functioning. Explanation: Benign tumors are usually slow growing but can occur in a vital area, where they can grow large enough to cause serious effects. Surgical removal of a benign tumor is dependent on many factors; even if the tumor is slow growing or not growing at all, the location of the tumor in the brain factors into the decision for surgical removal. The prognosis for all brain tumors is not necessarily poor. Treatment is individualized and can have varying prognostic outcomes. Benign tumors are not metastatic, meaning they do not grow rapidly or spread into surrounding tissue, but they can still be considered life-threatening.

An infusion of phenytoin (Dilantin) has been ordered for a patient whose brain tumor has just caused a seizure. The patient has been receiving D5W at 100 mL/hour to this point and has only one IV access site at this point. How should the nurse prepare to administer this drug to the patient? - Administer the drug orally due to the risk of precipitation. - Thoroughly flush the patient's IV with normal saline. - Mix the phenytoin in a 50 mL mini-bag of D5W. - Saline lock the patient's IV and wait 15 minutes before administering phenytoin.

Correct response: Thoroughly flush the patient's IV with normal saline. Explanation: The rate of Dilantin administration is no faster than 50 mg/min in normal saline solution, since the drug precipitates in D5W. If the preexisting solution contained dextrose, the nurse flushes the IV line with normal saline before administering the medication.

A patient diagnosed with transient ischemic attacks (TIAs) is scheduled for a carotid endarterectomy. The nurse explains that this procedure will be done for what purpose? - To prevent a stroke by removing atherosclerotic plaques blocking cerebral flow - To determine the cause of the TIA - To prevent seizure activity that is common following a TIA - To decrease cerebral edema

Correct response: To prevent a stroke by removing atherosclerotic plaques blocking cerebral flow Explanation: The main surgical procedure for select patients with TIAs is carotid endarterectomy, the removal of an atherosclerotic plaque or thrombus from the carotid artery to prevent stroke in patients with occlusive disease of the extra cranial arteries. An endarterectomy does not decrease cerebral edema, prevent seizure activity, or determine the cause of a TIA.

Nursing students are reviewing information about Parkinson's disease in preparation for class the next day. The students demonstrate understanding of the material when they identify which of the following as a cardinal sign of this disorder? Select all that apply. - Rigidity - Postural instability - Intellectual decline - Bradykinesia - Tremor

Correct response: Tremor Rigidity Bradykinesia Postural instability Explanation: Cardinal signs of Parkinson's disease are tremor, rigidity, bradykinesia, and postural instability. Although mental status changes can occur over the course of the disease, intellect is usually not affected.

A 70-year-old woman is being treated at home for Parkinson's disease (PD), a health problem that she was diagnosed with 18 months ago. The nurse who is participating in the woman's care should be aware that her initial symptoms most likely consisted of: - Visual disturbances and muscle weakness - Increasing forgetfulness and confusion - Tremors and muscle rigidity - Fatigue and respiratory difficulties

Correct response: Tremors and muscle rigidity Explanation: The cardinal signs of PD are tremor, rigidity, akinesia/bradykinesia, and postural disturbances.

A client with a malignant brain tumor comes to the clinic for a follow up. During the visit, the client asks the nurse, "Why am I so tired all the time?" When responding to the client, which information would the nurse include as possible causes? Select all that apply. - Tumor - Metastasis - Stress - Treatment being used - Effects of increased intracranial pressure

Correct response: Tumor Treatment being used Stress Explanation: Fatigue is a symptom experienced by clients with both malignant and nonmalignant brain tumors. Etiology of fatigue can be multifactorial. The tumor itself, surgery, medications, chemotherapy, and radiation may all contribute to increased fatigue. Clients may report a constant feeling of exhaustion, weakness, and lack of energy. It is also important to identify underlying conditions, such as stress, anxiety, and depression, which may play a role in fatigue. Metastasis and increased intracranial pressure are not usually associated with fatigue.

A client with meningitis has a history of seizures. Which should the nurse do to safely manage the client during a seizure? Select all that apply. - Turn the client to the side. - Physically restrain the client's movements. - Provide verbal reassurance. - Inspect the oral cavity and teeth.

Correct response: Turn the client to the side. Provide verbal reassurance. Explanation: Turning client to the side will allow accumulated saliva to drain from the mouth. The person may not be able to hear you while unconscious, but verbal assurances will help as the person is regaining consciousness. Physically restraining a client during a seizure increases the potential for injuries. Inspection of oral cavity occurs after a generalized seizure and not during a seizure.

A client is diagnosed with a tumor of the temporal lobe. When developing the client's plan of care the nurse would plan interventions to address problems with which areas of functioning? Select all that apply. - Writing - Understanding language - Emotions - Reading - Memory

Correct response: Understanding language Emotions Memory Explanation: Tumors of the temporal lobe may cause problems with language comprehension, behavior, memory, hearing and emotion. Problems with reading and writing would be associated with a tumor of the parietal lobe.

A middle-aged patient has undergone emergency neurosurgery for the treatment of spinal cord compression (SCC) that was detected by magnetic resonance imaging (MRI). Which of the following signs and symptoms is considered an early sign of spinal compression? - Intermittent claudication - Urinary incontinence - Personality changes - Muscle spasticity

Correct response: Urinary incontinence Explanation: Early symptoms associated with SCC include back pain, and bladder and bowel dysfunction (urinary incontinence or retention; fecal incontinence or constipation). Later symptoms include evidence of motor weakness and sensory deficits progressing to paralysis. Personality changes and intermittent claudication are not associated with SCC.

What interventions will best help the client with Huntington disease relieve anxiety and increase communication? Select all that apply. - Use an interpreter. - Consult with a speech therapist. - Always have family present. - Talk as little as possible. - Use biofeedback.

Correct response: Use biofeedback. Consult with a speech therapist. Explanation: Using biofeedback and relaxation therapy may help to decrease stress and help with communication. A speech therapist can help maintain and prolong communication abilities as well. An interpreter is not needed and the client should be encouraged to talk. Family presence is not essential, but the nurse should learn how the client expresses needs and wants, especially if the client is nonverbal.

The nurse is developing a plan of care for a patient who has stabilized after the emergency treatment of Guillain-Barré syndrome (GBS). What nursing intervention would receive priority for this patient? - Reorienting the patient to person, time, and place - Using the incentive spirometer as prescribed - Limiting free water to 1 L per day - Maintaining the patient on bed rest

Correct response: Using the incentive spirometer as prescribed Explanation: Respiratory function can be maximized in GBS with incentive spirometry and chest physiotherapy. Nursing interventions toward enhancing physical mobility should be utilized. Nursing interventions are aimed at preventing a deep vein thrombosis. Guillain-Barré does not affect cognitive function or level of consciousness. Fluid restriction is not indicated.

The nurse plays a critical role in the initial work-up of a patient with acute stroke symptoms. An immediate decision is to determine if the stroke is ischemic or hemorrhagic. Although there is overlap in some motor, sensory, and cognitive changes, hemorrhagic strokes can be identified by some specific signs. Which of the following signs are consistent with a hemorrhagic stroke? Select all that apply. - Loss of balance - Sudden, severe headache - Vomiting - Numbness or weakness of an extremity - Seizures

Correct response: Vomiting Sudden, severe headache Seizures Explanation: These three signs are usually diagnostic of a hemorrhagic stroke. The other signs can occur with both hemorrhagic and ischemic stroke.

The nurse is performing stroke risk screenings at a hospital open house. Identification of high-risk individuals is the goal of the screenings. The nurse has identified four patients who might be at risk for a stroke. Which patient is likely at highest risk for a stroke? - White man, age 60 with history of uncontrolled hypertension - Black man, age 60, with history of diabetes - Black man, age 50 with history of smoking - White woman, age 60 with history of excessive alcohol intake

Correct response: White man, age 60 with history of uncontrolled hypertension Explanation: Uncontrolled hypertension is the primary cause of a hemorrhagic stroke. Control of hypertension, especially in individuals over 55 years of age, clearly reduces the risk for hemorrhagic stroke. Additional risk factors are increased age, male gender, and excessive alcohol intake. Another high-risk group is African Americans, where the incidence of first stroke is almost twice that in Caucasians. Modifiable risk factors for ischemic stroke include hypertension, atrial fibrillation, hyperlipidemia, obesity, smoking, and diabetes.

Nurses on a neurological unit have been conscientious about positioning a patient to maximize mobility and preserve function following the patient's stroke 2 weeks ago. How should the patient's hands be best positioned to meet these goals? - With her hands slightly flexed and her palms facing up - With her fists closed and her wrists in a neutral position - With her wrists flexed and fingers straight - With her fingers straight and her palms facing down

Correct response: With her hands slightly flexed and her palms facing up Explanation: The fingers are positioned so that they are barely flexed. The hand is placed in slight supination (palm faces upward), which is its most functional position.

21) ** J - A nurse is reading a journal article about stroke and the underlying causes associated with this condition. The nurse demonstrates understanding of the information when identifying which subtype of stroke as being due to atrial fibrillation? large-artery thrombotic small, penetrating artery thrombotic cardio embolic cryptogenic

Correct response: cardio embolic Explanation: Ischemic strokes are further divided into five subtypes, according to a mechanism-based classification system: large-artery thrombotic strokes (representing 20% of ischemic strokes); small, penetrating artery thrombotic strokes (25%); cardio embolic strokes (20%); cryptogenic strokes (strokes that cannot be attributed to any specific cause) (30%); and "other" (5%). Large-artery thrombotic strokes are caused by atherosclerotic plaques in the large blood vessels of the brain. Thrombus formation and occlusion can occur at the site of the atherosclerosis and result in ischemia and infarction (tissue death). Small, penetrating artery thrombotic strokes that affect one or more vessels and cause reduced blood flow are the most common type of ischemic stroke, typically caused by longstanding hypertension, hyperlipidemia, or diabetes. Cardio embolic strokes are associated with cardiac dysrhythmias, such as atrial fibrillation, but can also be associated with valvular heart disease or left ventricular thrombus. The last two classifications of ischemic strokes are cryptogenic strokes, which have no identified cause, and strokes from other causes, such as illicit drug use (cocaine), coagulopathies, migraine, or spontaneous dissection of the carotid or vertebral arteries.

A nurse is teaching a client with multiple sclerosis (MS). When teaching the client how to reduce fatigue, the nurse should tell the client to: - rest in an air-conditioned room. - increase the dose of muscle relaxants. - take a hot bath. - avoid naps during the day.

Correct response: rest in an air-conditioned room. Explanation: Fatigue is a common symptom in clients with MS. Lowering the body temperature by resting in an air-conditioned room may relieve fatigue; however, extreme cold should be avoided. A hot bath or shower can increase body temperature, producing fatigue. Muscle relaxants, ordered to reduce spasticity, can cause drowsiness and fatigue. Frequent rest periods and naps can relieve fatigue. Other measures to reduce fatigue in the client with MS include treating depression, using occupational therapy to learn energy-conservation techniques, and reducing spasticity.

Which of the following medications needs to be withheld for 5 to 7 days prior to cataract surgery? Coumadin Glucophage Lasix Prednisone

Coumadin It has been common practice to withhold any anticoagulant therapy such as Coumadin to reduce the risk for retrobulbar hemorrhage (after retrobulbar injection) for 5 to 7 days before surgery.

What type of medication would the nurse use in combination with mydriatics to dilate the patient's pupil? NSAIDs Corticosteroids Anti-infectives Cycloplegics

Cycloplegics Mydriasis, or pupil dilation, is the main objective of the administration of mydriatics and cycloplegics (Table 63-3). These two types of medications function differently and are used in combination to achieve the maximal dilation that is needed during surgery and fundus examinations to give the ophthalmologist a better view of the internal eye structures.

A client presents to the emergency department with an open fracture. What is the first action the nurse should take? A Assist the physician with reduction of the fracture. B Assess the client's vital signs and determine allergies. C Perform a neurovascular assessment of the affected extremity. D Cover the exposed bone with sterile dressing.

D

A client with a traumatic amputation of the right lower leg is refusing to look at the leg. Which action by the nurse is most appropriate? A. Encourage the client to perform range-of-motion (ROM) exercises to the right leg. B Provide wound care without discussing the amputation. C Request a referral to occupational therapy. D Provide feedback on the client's strengths and available resources.

D

Which of the following is a term used to describe a soft tissue injury produced by a blunt force? A Hematoma B Strain C Sprain D Contusion

D A contusion is a soft tissue injury produced by blunt force, such as a blow, kick, or fall, that results in bleeding into soft tissues (ecchymosis, or bruising). A hematoma develops when the bleeding is sufficient to form an appreciable solid swelling. A strain, or a "pulled muscle," is an injury to a musculotendinous unit caused by overuse, overstretching, or excessive stress. A sprain is an injury to the ligaments and supporting muscle fibers that surround a joint often caused by a trauma, wrenching or twisting motion.

Which term refers to an injury to ligaments and other soft tissues surrounding a joint? A Strain B Dislocation C Subluxation D Sprain

D A sprain is caused by a wrenching or twisting motion. Dislocation refers to the separation of joint surfaces. Subluxation refers to partial separation or dislocation of joint surfaces. Strain refers to a muscle pull or tear.

A patient has stepped in a hole in the yard, causing an ankle injury. The ankle is edematous and painful to palpation. How long should the nurse inform the patient that the acute inflammatory stage will last? A About 72 hours B At least 1 week C Less than 24 hours D Between 24 and 48 hours

D After the acute inflammatory stage (e.g., 24 to 48 hours after injury), intermittent heat application (for 15 to 30 minutes, four times a day) relieves muscle spasm and promotes vasodilation, absorption, and repair.

Radiographs were ordered for a 10-year-old boy who had his right upper arm injured. The radiographs show that the humerus appears to be fractured on one side and slightly bent on the other. What type of fracture is this an example of? A Compound B Impacted C Compression D Greenstick (incomplete)

D Greenstick fractures are a result of the bone being broken on one side, while the other side is bent.

The primary nursing intervention that will control swelling while treating a musculoskeletal injury is: A Immobilize the injured area. B Apply cold (moist or dry). C Apply an elastic compression bandage. D Elevate the affected area.

D The acronym RICE—Rest, Ice, Compression, Elevation—is helpful for remembering treatment interventions for musculoskeletal injuries. Rest prevents additional injury and promotes healing. Intermittent application of moist or dry cold packs for 20-30 minutes during the first 24-48 hours after injury produces vasoconstriction, which decreases bleeding, edema, and discomfort. Ensure care to avoid skin and tissue damage from excessive cold. An elastic compression bandage controls bleeding, reduces edema, and provides support for the injured tissues. Elevation controls the swelling.

A client is treated in the emergency department for acute muscle strain in the left leg caused by trying a new exercise. During discharge preparation, the nurse should provide which instruction? A "Apply ice packs for the first 12 to 18 hours." B "Apply heat packs for the first 24 to 48 hours." C "Apply heat packs for the first 24 hours, then apply ice packs for the next 48 hours." D "Apply ice packs for the first 24 to 48 hours, then apply heat packs."

D The nurse should instruct the client to apply ice packs to the injured area for the first 24 to 48 hours to reduce swelling and then apply heat to increase comfort, promote reabsorption of blood and fluid, and speed healing. Applying ice for only 12 to 18 hours may not keep swelling from recurring. Applying heat for the first 24 to 48 hours would worsen, not ease, swelling. Applying ice 48 hours after the injury would be less effective because swelling already has occurred by that time.

A client is being discharged from the Emergency Department after being diagnosed with a sprained ankle. Which client statement indicates the client understands the discharge teaching? A "I'll get the prescription filled for the narcotic pain reliever." B "I need to stay off my ankle for at least the next 3 to 4 weeks." C "I'll start with ice for the first couple of hours and then apply heat." D "I'll make sure to keep my ankle elevated as much as possible."

D Treatment consists of applying ice or a chemical cold pack to the area to reduce swelling and relieve pain for the first 24 to 48 hours. Elevation of the part and compression with an elastic bandage also may be recommended. After 2 days, when swelling no longer is likely to increase, applying heat reduces pain and relieves local edema by improving circulation.

An older adult man has been diagnosed with Parkinson's disease and has begun treatment with levodopa and carbidopa. When providing health education about his new medication regimen, what should the nurse teach the man? A) "If you're consistent with taking your medication, you might not experience symptoms for several more years." B) "This medication can cure Parkinson's disease, but this is not necessarily the case for everyone." C) "The beneficial effects of this medication usually increase over time, so you may not get maximum relief for a few years." D) "This medication helps significantly but the benefits tend to decrease over time."

D) "This medication helps significantly but the benefits tend to decrease over time." The beneficial effects of levodopa are most pronounced in the first few years of treatment. Benefits begin to wane and adverse effects become more severe over time. Confusion, hallucinations, depression, and sleep alterations are associated with prolonged use. Levodopa is usually administered in combination with carbidopa (Sinemet), an amino acid decarboxylase inhibitor, which helps to maximize the beneficial effects of levodopa. Within 5 to 10 years, most patients develop a response to the medication characterized by dyskinesia (abnormal involuntary movements), including facial grimacing, rhythmic jerking movements of the hands, head bobbing, chewing and smacking movements, and involuntary movements of the trunk and extremities.

23. An elderly female resident of a long-term care facility has a diagnosis of Alzheimer's disease (AD). The resident is visibly anxious and is insisting to the nurse that she needs to "take care of my babies." How should the nurse respond to the resident's statement? A) Reorient the resident to the fact that she does not have young children. B) Ask the resident questions about her children to help her realize that her children are now adults. C) Ignore the resident's statement and return to the room later. D) Engage the resident in a conversation about a different topic.

D) Engage the resident in a conversation about a different topic. Trying to reason with people with AD and using reality orientation only increases their anxiety without increasing function.

The nurse is planning the care of a patient who has been admitted to the hospital for surgical treatment of prostate cancer that has metastasized to his spine. When planning nursing care that is specific to this patient's spinal metastasis, what outcome should the nurse identify? A)The patient will explain the importance of exercise in his recovery. B)The patient will maintain full range of motion. C)The patient will perform activities of daily living comparable to his prediagnosis abilities. D)The patient will state that pain control is adequate.

D)The patient will state that pain control is adequate.

A male patient with a metastatic brain tumor is having a generalized seizure and begins vomiting. What should the nurse do first? A)Perform oral suctioning. B)Page the physician. C)Insert a tongue depressor into the patient's mouth. D)Turn the patient on his side.

D)Turn the patient on his side. Feedback: The nurse's first response should be to place the patient on his side to prevent him from aspirating emesis. Inserting something into the seizing patient's mouth is no longer part of a seizure protocol. Obtaining supplies to suction the patient would be a delegated task. Paging or calling the physician would only be necessary if this is the patient's first seizure.

A patient diagnosed with multiple sclerosis (MS) has been admitted to the medical unit for treatment of an MS exacerbation. Included in the admission orders is baclofen (Lioresal). What would be the expected outcome of this medication? -Reduction in the appearance of new lesions on magnetic resonance imaging (MRI) -Decreased muscle spasms in the lower extremities -Increased muscle strength in the upper extremities -Promotion of urinary continence

Decreased muscle spasms in the lower extremities Feedback: Baclofen, a GABA agonist, is the medication of choice in treating spasms. It can be administered orally or by intrathecal injection. It is not used to promote continence or to increase strength. Avonex and Betaseron reduce the appearance of new lesions on the MRI.

The nurse is providing care to a client who has been admitted to the hospital for treatment of an infection. The client is visually impaired. Which of the following would be most appropriate for the nurse to do when interacting with the client? Touch the client before identifying himself or herself. Talk to the client in a loud tone of voice. Avoid using the terms "see" or "look." Face the client when speaking directly to him.

Face the client when speaking directly to him. When interacting with a client with a visual impairment, the nurse should face the client and speak directly to the client using a normal tone of voice. It is not necessary to raise the voice unless the client asks the nurse to do so and it is not necessary to avoid the terms, "see" or "look" when interacting with the client. The nurse should identify himself or herself when approaching the client and before making any physical contact.

The nurse should monitor for which manifestation in a client who has had LASIK surgery? Halos and glare Cataract formation Excessive tearing Stye formation

Halos and glare After LASIK surgery, symptoms of central islands and decentered ablations can occur that include monocular diplopia or ghost images, halos, glare, and decreased visual acuity. These procedures do not cause excessive tearing or result in cataract or stye formation.

The nurse should monitor for which manifestation in a client who has undergone LASIK? Cataract formation Halos and glare Excessive tearing Stye formation

Halos and glare Symptoms of central islands and decentered ablations can occur after LASIK surgery; these include monocular diplopia or ghost images, halos, glare, and decreased visual acuity. These procedures do not cause excessive tearing or result in cataract or stye formation.

A client with chronic open-angle glaucoma is now presenting with eye pain and intraocular pressure of 50 mm Hg. An immediate iridotomy is scheduled. Which of the following describes the desired effects of this procedure? Restore vision To relieve pain Reverse optic nerve damage Improve outflow drainage

Improve outflow drainage Laser iridotomy or standard iridotomy is a surgical procedure that provides additional outlet drainage of aqueous humor. This is done to lower the IOP as quickly as possible since permanent vision loss can occur in 1 to 2 days. Once optic nerve damage occurs, it cannot be reversed, and vision is not restored. Pain that occurs with rising IOP will be controlled once pressure is lowered through improved outflow drainage.

A client with chronic open-angle glaucoma is now presenting with eye pain and intraocular pressure of 50 mm Hg. An immediate iridotomy is scheduled. Which of the following describes the desired effects of this procedure? Reverse optic nerve damage To relieve pain Improve outflow drainage Restore vision

Improve outflow drainage Laser iridotomy or standard iridotomy is a surgical procedure that provides additional outlet drainage of aqueous humor. This is done to lower the IOP as quickly as possible since permanent vision loss can occur in 1 to 2 days. Once optic nerve damage occurs, it cannot be reversed, and vision is not restored. Pain that occurs with rising IOP will be controlled once pressure is lowered through improved outflow drainage.

The nurse is planning the care of a 28-year-old female hospitalized with a diagnosis of myasthenia gravis. What schedule would be most appropriate for the organization of diagnostic procedures for this patient? - All at one time, to provide a longer rest period - Before meals, to stimulate her appetite - In the morning, with frequent rest periods - Before bedtime, to promote rest

In the morning, with frequent rest periods. Feedback: Myasthenia gravis, an autoimmune disorder affecting the myoneural junction, is characterized by varying degrees of weakness of the voluntary muscles. Procedures should be spaced to allow for rest in between. Procedures should be avoided before meals, or the patient may be too exhausted to eat. Procedures should be avoided at bedtime.

A client who has come to the clinic for an evaluation is diagnosed with glaucoma. The client asks the nurse, "What is this disease?" Which information would the nurse most likely include in the response? Increased pressure in the eye causes damage to the optic nerve. The conjunctiva becomes inflamed and irritated. The lens becomes cloudy, causing vision to be impaired. The retina separates from the sensory layers of the eye.

Increased pressure in the eye causes damage to the optic nerve. Glaucoma is a group of ocular conditions characterized by optic nerve damage that is related to the increased intraocular pressure caused by congestion of the aqueous humor in the eye. A cataract is a clouding of the lens. Retinal detachment occurs when the retina separates from the sensory layer. Conjunctivitis refers to an inflammation of the conjunctiva.

A male patient with a history of poorly controlled type 1 diabetes has experienced an accelerated deterioration in his vision and is now considered to be legally blind. When planning this patient's care, the nurse should recognize the possibility of what nursing diagnosis? Ineffective coping Unilateral neglect Decisional conflict Moral distress

Ineffective coping Although every patient will respond differently to a decrease in visual acuity, ineffective coping has been identified as a common phenomenon. This is more likely than unilateral neglect, moral distress, or decisional conflict.

Intracerebral tumors - There is 5 - A - G - O - E - M -

Intracerebral tumors Gliomas—infiltrate any portion of the brain: - Astrocytomas (grades I and II) - Glioblastoma multiforme (astrocytoma grades III and IV) - Oligodendrocytoma (low and high grades) - Ependymoma (grades I to IV) - Medulloblastoma

The nurse is preparing a presentation for a local community group comparing photorefractive keratectomy and LASIK refractive surgeries. Which of the following would the nurse include? PRK requires that a thin flap be made to allow access to the cornea. PRK is used primarily for people without an astigmatism. LASIK is appropriate for people with very thin corneas. LASIK involves working with the cornea on a deeper level.

LASIK involves working with the cornea on a deeper level. LASIK involves the creation of a corneal flap to allow access to the corneal stroma at a deeper level. PRK is used to treat myopia and hyperopia with or without an astigmatism and is now reserved for clients unsuitable for LASIK, such as those with very thin corneas.

A female patient is admitted for evaluation of a cerebral metastasis from a primary site. When reviewing her history, what would be the MOST likely primary site? A) Lung B) Prostate C) renal D) Uterus

LUNG

Which surgical procedure involves flattening the anterior curvature of the cornea by removing a stromal lamella? Photorefractive keratectomy (PRK) Laser-assisted in situ keratomileusis (LASIK) Keratoplasty Keratoconus

Laser-assisted in situ keratomileusis (LASIK) LASIK involves flattening the anterior curvature of the cornea by removing a stromal lamella or layer. PRK is used to treat myopia and hyperopia with or without astigmatism. Keratoconus is a cone-shaped deformity of the cornea. Keratoplasty involves replacing abnormal host tissue with healthy donor (cadaver) corneal tissue.

A nurse is teaching a client about medications for glaucoma. What is the main marker of glaucoma control with medication? Reducing the appearance of optic nerve head Changing the opacity of the lens Lowering intraocular pressure to target pressure Increasing the visual field

Lowering intraocular pressure to target pressure The main marker of the efficacy of the medication in glaucoma control is lowering of the intraocular pressure to the target pressure. Opacity of the lenses relates to cataract formation. The appearance of the optic nerve head and the visual field are not goals with glaucoma medication.

During an initial assessment, the nurse notes a symptom of a mild case of bacterial conjunctivitis and documents in the electronic medical record that the client is displaying which of the following ? Blurred vision Elevated intraocular pressure Severe pain Mucopurulent ocular discharge

Mucopurulent ocular discharge Bacterial conjunctivitis manifests with an acute onset of redness, burning, and discharge. Purulent discharge occurs in severe acute bacterial infections, whereas mucopurulent discharge appears in mild cases.

To avoid the side effects of corticosteroids, which medication classification is used as an alternative to treat inflammatory conditions of the eyes? Mydriatics NSAIDs Cycloplegics Miotics

NSAIDs NSAIDs are used as an alternative in controlling inflammatory eye conditions and postoperatively to reduce inflammation. Miotics are used to cause the pupil to constrict. Mydriatics cause the pupil to dilate. Cycloplegics cause paralysis of the iris sphincter.

A patient is being discharged home from the ambulatory surgical center after cataract surgery. In reviewing the discharge instructions with the patient, the nurse instructs the patient to immediately call the office if he experiences what? Mild redness of the eye New floater in vision Scratchy feeling in the eye Sight morning discharge from the eye

New floater in vision Cataract surgery increases the risk of retinal detachment, and the patient must be instructed to notify the surgeon if new floaters in vision, flashing lights, decrease in vision, pain, or increase in redness occurs. Slight morning discharge, some redness, and a scratchy feeling may be expected for a few days after surgery.

When an impaled object is in the eye, which of the following steps should be taken to ensure that no further damage occurs? Select all that apply. Apply a patch to the eye Apply gentle pressure to the eye Protect object from jarring No attempt should be made to remove the object Use metal shield

No attempt should be made to remove the object Protect object from jarring Use metal shield With a foreign body, no attempt should be made to remove it. The object should be protected from jarring or movement to prevent further ocular damage. No pressure or patch should be applied to the affected eye. All traumatic eye injuries should be protected with a metal shield if available.

A legally blind client is in pre-op area prior to an appendectomy. What steps does the nurse take to effectively communicate with this client ? Inform the client that the nurse will be working nearby. Sit near the client to provide reassurance of the strange surroundings. Make direct eye contact with the client when communicating. Notify the client prior to touching the client.

Notify the client prior to touching the client. The nurse should announce upon arrival the bedside every time because many voices sound similar. The nurse should use the client's name initially so the client knows the nurse is communicating with the client directly. The nurse should speak before touching the client as not to startle the client. The nurse should notify the client when approaching and leaving the bedside each time. Orient the client to their surroundings using verbal descriptions and directions such as left, or right.

Which term refers to swelling of the optic disc due to increased intracranial pressure? Photophobia Papilledema Chemosis Ptosis

Papilledema Papilledema is swelling of the optic disc due to increased intracranial pressure. Chemosis is edema of the conjunctiva. Ptosis is a drooping eyelid. Photophobia is ocular pain on exposure to light.

A 71-year-old man has made an appointment with his primary care provider at the urging of his wife, who states that he has occasionally had episodes of weakness and slurring of words over the past several weeks. The care provider recognizes the possibility that the man has been experiencing transient ischemic attacks (TIAs). TIAs have which of the following characteristics? -TIAs result in motor symptoms rather than sensory symptoms. -TIAs are a result of minor cerebral hemorrhages that spontaneously resolve. -TIAs cause irreversible, but minor, neurological damage -TIAs cause symptoms that last less than 1 hour.

TIAs cause symptoms that last less than 1 hour. Feedback: A TIA is defined as a brief episode of neurologic dysfunction caused by focal brain or retinal ischemia, with clinical symptoms typically lasting less than 1 hour, and without evidence of acute infarction. They may cause sensory symptoms and are not a result of hemorrhage.


Set pelajaran terkait

Chapter 1: Introduction to Systems Analysis and Design

View Set

Chapter 6 Sampling Distributions and Confidence Intervals

View Set

PrepU 20: Assessment of Respiratory Function

View Set

CompTIA SY0-401 - Topic 1, Network Security

View Set

Mgmt 493 Final - Woo (Chapter 1)

View Set

Module 4: Protista and the Origin of Eukaryotes

View Set

Erik Erikson's stages of development

View Set

Assessment and Evaluations: Aptitude

View Set